You are on page 1of 278

JENIC CRNGANU

ANALIZ MATEMATIC

EDITURA FUNDAIEI UNIVERSITARE Dunrea de Jos Galai

JENIC CRNGANU

ANALIZ MATEMATIC

Conf.dr. JENIC CRNGANU

ANALIZ MATEMATIC

EDITURA FUNDAIEI UNIVERSITARE Dunrea de Jos Galai, 2008

Referent tiinific: Conf.univ.dr. Petru V Universitatea Dunrea de Jos Galai

PREFA
Lucrarea de fa reprezint o variant mbuntit a cursului publicat anterior de catre autor. Ea prezint noiuni fundamentale ale analizei matematice cum ar fi cele de limit, continuitate, difereniabilitate, integrabilitate, etc. Acest material reprezint rodul activitii universitare din ultimii ani, cursuri inute la diferite faculti ale Universitii Dunrea de Jos din Galai, de ctre autor. Pentru o mai bun nelegere a chestiunilor teoretice, cartea are numeroase observaii, exemple, probleme propuse spre rezolvare i probleme rezolvate. Prezentul curs se adreseaz studenilor din anul I ai facultilor cu profil tehnic i universitar, profesorilor din licee care i matematica modern a zilelor noastre. Tuturor cititorilor mei, studeni, profesori de matematica, ingineri, interesai de rezolvarea unor probleme matematice, autorul le ureaz lectur interesant i s gseasc n paginile lucrrii ceea i doresc. n ncheiere, in s exprim mulumiri domnilor conf.dr. Petru V i conf.dr. Ion Miric, pentru rbdarea i atenia cu care au citit ntregul manuscris, fcnd observaii utile, de care am inut seama n redactarea final a lucrrii. Autorul rmne ndatorat tuturor acelora care i vor trimite sugestii, alte puncte de vedere sau vor avea amabilitatea de a-i semnala eventualele erori structurate n lucrare. pregtesc examenele de definitivare sau grad, ct i tuturor celor care doresc s nvee i s aprofundeze

Galai, septembrie, 2008

J. Crnganu

Refereni tiinifici :
Conf. univ. dr. Petru V Conf. univ. dr. Ion Miric Universitatea Dunrea de Jos Galai

CUPRINS

PREFA..7 CAP. 1. MULIMI. RELAII. FUNCII. 9 CAP.2. SPAII METRICE. IRURI N SPAII METRICE...17 2.1.Spaii metrice. Definiie. Exemple..17 2.2. iruri n spaii metrice.19 2.3. iruri n spaii metrice particulare..22 2.4. Spaii metrice complete..31 2.5. Elemente de topologie n spaii metrice...33 CAP.3. SERII DE NUMERE REALE...44 3.1. Serii convergente. Serii divergente...44 3.2. Serii cu termeni pozitivi..48 3.3. Serii cu termeni oarecare..59 3.4. Serii alternate...61 CAP.4. FUNCII NTRE SPAII METRICE...65 4.1. Limita unei funcii ntr-un punct.65 4.2. Funcii continue71 4.3. Proprieti ale funciilor continue...75 4.4. Funcii uniform continue.79 CAP.5. DERIVABILITATEA I DIFERENIABILITATEA FUNCIILOR REALE DE VARIABIL REAL .83 5.1.Proprieti de baz ale derivatei.83 5.2. Difereniala unei funcii87 5.3. Derivate i difereniale de ordin superior.88 CAP.6. DERIVABILITATEA I DIFERENIABILITATEA FUNCILOR DE MAI MULTE VARIABILE.96 6.1.Derivata dup o direcie. Derivate pariale de ordinul nti96 6.2. Difereniabilitatea funciilor reale de mai multe variabile...99

6.3. Difereniabilitatea funciilor compuse.105 6.4. Derivate pariale i difereniale de ordin superior.108 6.5. Funcii i sisteme de funcii implicite..118 6.6. Extreme locale pentru funcii reale de mai multe variabile.127 CAP. 7. INTEGRALA RIEMANN......143 7.1. Primitiva unei funcii reale de variabil real...143 7.2. Integrala definit......153 7.3. Aplicaii ale integralei definite.158 CAP. 8. INTEGRALE IMPROPRII........166 8.1. Integrale improprii de spea nti...166 8.2. Integrale improprii de spea a doua..173 8.3. Integralele lui Euler..178 CAP. 9. IRURI I SERII DE FUNCII.......183 9.1.iruri de funcii...183 9.2. Serii de funcii......191 9.3 Serii de puteri....196 CAP. 10. INTEGRALE CU PARAMETRU......211 CAP. 11. INTEGRALE CURBILINII..222 11.1. Integrale curbilinii de spea nti.....222 11.2. Integrale curbilinii de spea a doua.....232 CAP. 12. INTEGRALE MULTIPLE..........244 12.1.Integrale duble....244 12.2. Integrale de suprafa...262 12.3. Integrale triple....271 BIBLIOGRAFIE279

-9-

CAPITOLUL 1
MULIMI. RELAII. FUNCII
Fie X, Y dou mulimi nevide i X x Y = { (x,y): xX, yY }, produsul lor cartezian. Definiia 1.1. Se numete relaie binar ntre elementele mulimilor X i Y tripletul = (X,Y,G), unde G X x Y se numete graficul relaiei . Dac (x,y) G spunem c x este n relaia cu y i scriem xy. Se numete domeniu al relaiei multimea D() = { xX : ()yY astfel nct (x,y)G }. Se numete codomeniu al relaiei mulimea Im() = { yY : ()xX astfel nct (x,y)G }. Dac X = Y atunci se numete relaie binar pe X (sau relaie pe X). Dintre relaiile definite ntre elementele aceleiai mulimi se disting relaiile de echivalena i de ordine. Definiia 1.2. Fie X . O relaie pe X se numete relaie de echivalen dac (E1) este reflexiv: () xX xx; (E2) este simetric: () x, y X astfel nct xy yx; (E3) este tranzitiv: () x, y, z X astfel nct xy i yz xz. Dac xX definim Cx = { yX : yx } clasa de echivalen a lui x. Din definiie rezult imediat urmtoarele proprieti ale claselor de echivalen: 1. ()xX Cx ; 2. dac x, yX, xy Cx Cy = ; 3. xy Cx = Cy ; 4. U Cx = X.
xX

- 10 -

Mulimea notat X/ = { Cx : xX } se numete mulimea claselor de echivalen. Definiia 1.3. O relaie pe X, notat se numete relaie de ordine pe X dac (O1) este reflexiv: () xX x x; (O2) este antisimetric: () x,yX astfel nct x y i y x x = y; (O3) este tranzitiv: () x,y,zX astfel nct x y i y z x z. Perechea ( X, ) se numete mulime ordonat. Dac () x,yX avem x y sau y x, relaia de ordine se numete relaie de ordine total, iar ( X, ) mulime total ordonat. Exemple. 1. Dac X atunci relaia de incluziune este o relaie de ordine pe mulimea prilor lui X, mulime notat cu P(X). 2. (N*,I) ( I relaia de divizibilitate) este ordonat dar nu este total ordonat. Definiia 1.4. Fie ( X , ) o mulime ordonat i A X, nevid. Un element mX se numete minorant pentru A dac m x, ()xA. Un element MX se numete majorant pentru A dac x M, ()xA. Dac mX este minorant i m A atunci m se numete cel mai mic element al mulimii A i se noteaz m = minA. Dac MX este majorant i M A atunci M se numete cel mai mare element al mulimii A i se noteaz M = maxA. Observaie. Dac A are un cel mai mic (respectiv cel mai mare)

element, din proprietatea de antisimetrie rezult c acesta este unic. Dac submulimea A X are minorani (respectiv majorani) spunem c A este minorat sau mrginit inferior (respectiv majorat sau mrginit superior). Mulimea A se numete mrginit dac este minorat i majorat.

- 11 -

Definiia 1.5. Dac A admite minorani (respectiv majorani) i mulimea minoranilor (respectiv a majoranilor) lui A are un cel mai mic (respectiv cel mai mare) element, acesta se numete margine inferioar sau infimum (respectiv margine superioar sau supremum) i se noteaz infA (respectiv supA). Observaii. 1. Dac exist infA (respectiv supA), din proprietatea de antisimetrie rezult c acesta este unic. 2. Dac A are un cel mai mic (respectiv cel mai mare) element atunci exist infA = minA (respectiv supA = maxA).

Mulimea numerelor reale


Pornind de la mulimea numerelor naturale N = { 0,1,2,} se poate face o construcie riguroas a mulimilor Z i Q (vezi [7]): Z = { 0,1, 2,} - mulimea numerelor ntregi. Q={
m : m,nZ, n 0 } - mulimea numerelor raionale. n

Pentru mulimea numerelor reale, notat cu R vom prezenta o construcie axiomatic. Definiia 1.6. Numim mulimea numerelor reale o mulime R nzestrat cu dou operaii algebrice: + adunarea, (x,y) x + y R; nmulirea, (x,y) xy R; i cu o relaie de ordine total care satisfac urmtoarele grupe de axiome (R,+,.) este corp comutativ, adic 1) () x,y,zR (x+y)+z = x+(y+z); 2) () x,yR x+ y = y+ x; 3) () 0R astfel nct 0+ x = x, ()xR; 4) () xR, ()x = -xR astfel nct x+ (-x) = 0; 5) () x,y,zR (xy)z = x(yz);

- 12 -

6) () x,yR xy = yx. 7) () 1R, 1 0 astfel nct 1.x = x, ()xR; 8) () xR, x 0, ()x-1 = R astfel nct x . x-1 =1; 9) () x,y,zR x(y+z) = xy+ xz. (R, ) este total ordonat i relaia este compatibil cu structura de corp, adic: 10) 11) 12) 13) 14) 15) () xR x x; () x,yR astfel nct x y i y x x = y; () x,y,zR astfel nct x y i y z x z; () x,yR avem x y sau y x; () x,y,zR cu x y x + z y + z; () x,y,zR cu x y i 0 z xz yz.
1 x

Axioma de completitudine a lui Cantor-Dedekind: Pentru orice submulime A R, nevid i majorat exist margine superioar, supAR. Observaii. 1. Pentru ()x,yR definim x-y = x+(-y) iar dac, n plus, y 0 atunci
x = xy-1. y

2. Din axiomele lui R, cum 1R rezult c i elementele 2=1+1, 3=(1+1)+1, vor aparine lui R. Aceste elemente le vom numi numere naturale iar mulimea lor o vom nota cu N. Dac nN atunci nR i mulimea elementelor 0,1,-1,2,-2, o vom nota cu Z i se numete mulimea numerelor ntregi. Dac x,yZ, y 0, atunci xy-1R iar mulimea elementelor de forma xy-1 cu x,yZ, y 0 o vom numi mulimea numerelor raionale i o vom nota cu Q. S observm c N Z Q R. 3. Relaiei de ordine i se poate ataa relaia <, numit de ordine strict i definit astfel: x < y <=> x y i x y, ()x,yR. De asemenea, dac x < y(respectiv x y), mai scriem y > x (respectiv y x).

- 13 -

Numerele reale x pentru care x 0( respectiv x > 0 ) se vor numi numere pozitive(respectiv strict pozitive), iar numerele reale x pentru care x 0 (respectiv x < 0), se vor numi numere negative (respectiv strict negative). Vom nota prin R+ - mulimea numerelor reale pozitive; R * - mulimea numerelor reale strict pozitive; + R- - mulimea numerelor reale negative; R * - mulimea numerelor reale strict negative. Atunci R = R+ R -, R+ R- = {0}. Dac a, bR, a < b definim mulimile (a,b) = { xR : a < x < b }; [a,b] = { xR : a x b }; [a,b) = { xR : a x < b }; (a,b] = { xR : a < x b }, numite i intervale mrginite. Prin definiie (a,a ) = [a,a) = (a,a] = , [a,a] = {a}. Pe R se mai definesc i urmtoarele tipuri de intervale nemrginite: (-,a ) = { xR : x < a} , (-,a] = { xR : x a}; ( a, ) = { xR : x > a}, [a, ) = { xR : x a}. O mulime IR se numete interval dac pentru () a,bI i a c b c I. Pentru orice numr real x definim modulul sau valoarea absolut a lui x, notat IxI, prin
x, daca x 0 x = x, daca x < 0

Din axioma lui Cantor-Dedekind i definiia marginii superioare (respectiv inferioare) obinem urmatoarele dou teoreme: Teorema1.1. Fie A R nevid i mrginit. Atunci () M = supA R i este caracterizat astfel : i) ii) x M, () xA; () > 0, () x A asfel nct x > M-.

- 14 -

Teorema1.2. Fie A R nevid i minorat. Atunci () m = infA R i este caracterizat astfel: i) ii) Arhimede: Teorema1.3. Dac x > 0 i y > 0 atunci exist n N* asfel nct nx > y. Demonstraie. Presupunem prin reducere la absurd ca nx y, () nN*. Fie A={nx: nN*}. Atunci A este nevid i majorat i din teorema 1.1 exist M = supA R. Pentru = x > 0 exist xA astfel nct M-x < x M. Cum xA, exist n0N* astfel nct x= n0x i atunci vom avea M-x < n0x M, de unde M < ( n0+1)xA, care contrazice faptul c M = supA. m x, () xA; () > 0, () x A astfel nct x < m + .

Urmtoarea teorem este cunoscut i sub numele de proprietatea lui

Observaie. Mulimea numerelor reale se mai numete i dreapta real, deoarece se poate stabili o corespondenta bijectiv (bijecia lui Descartes) ntre elementele lui R i mulimea punctelor de pe o dreapt. Fie A R nevid. Dac A nu admite majorani, prin definiie supA = +. Dac A nu admite minorani, prin definiie infA = -. Definim R = R {-,+} i vom numi aceasta mulime dreapta real ncheiat (sau mulimea extins a numerelor reale). Relaia de ordine uzual din R se extinde pe R prin conveniile - < + , - < x , x < +, ()xR. In felul acesta R devine total ordonat i orice submulime nevid A R are att supremum ct si infimum. Operaiile algebrice ale mulimii R se extind la R fr a fi ns peste tot definite. Astfel se definesc: +x = x+ = , ()x R , x -;

- 15 -

- + x = x+ (-) = -, () x R , x +;
+ , daca x > 0 x = x = , daca x < 0

Nu sunt definite operaiile: - ,


, 0. , 0 , etc

Dac a,b R ,a < b se definesc n R urmtoarele tipuri de intervale: (a,b) = { x R : a < x < b }; [a,b] = { x R : a x b }; [a,b) = { x R : a x <b }; (a,b] = { x R : a < x b }. Funcii Fie X i Y dou mulimi nevide. Definiia 1.7. Se numete funcie de la X la Y o relaie f = (X, Y, Gf ) cu proprietile i) ii) D(f) =X; () xX si () (x,y1), (x, y2) Gf y1 = y2 (deci oricrui element

xX i se asociaz un unic element yY astfel nct (x,y) Gf ) . Pentru o funcie se folsesc notaiile : f : X Y sau x f(x), xX. Mulimea X se numete mulime de definiie a funciei f, Y codomeniu (sau domeniul valorilor) iar Gf X Y se numete graficul funciei f. Observaie. n locul termenului de funcie se mai folosesc i termenii de aplicaie, transformare, operaie, operator sau reprezentare. Fie f : X Y o funcie, A X, B Y.

- 16 -

Mulimea f(A) = { f(x) : x A} Y se numete imaginea direct a lui A prin funcia f iar mulimea f-1(B) = { xX : f(x) B} se va numi imaginea invers a lui B prin funcia f. O funcie f : X Y se numete injectiv dac pentru ()x1,x2 X cu x1 x2 f(x1) f(x2). O funcie f : X Y se numete surjectiv dac pentru ()yY, ()xX astfel nct f(x) = y (sau echivalent Imf =Y). O funcie f :X Y se numete bijectiv dac este injectiv i surjectiv. Dac f :X Y i g : Y Z definim funcia compus h : X Z, h = g f, unde h(x) = g(f(x)), () xX. Fie 1X : X X funcia identic pe X, 1X(x) = x, oricare ar fi xX i 1Y:YY funcia identic pe Y. O funcie f :X Y se numete inversabil dac () g :Y X astfel nct f g = 1Y i g f = 1X. Se arat c funcia g, n ipoteza c exist, este unic i prin definiie g se numete inversa funciei f i se noteaz g = f -1. Teorema 1.4. O funcie f : X Y este inversabil dac i numai dac este bijectiv.

- 17 -

CAPITOLUL 2
SPAII METRICE. IRURI N SPAII METRICE
2.1. Spaii metrice. Definiie. Exemple
Definiia 2.1.1. Fie X . O funcie d : X X R se numete metric (sau distana) pe X dac: (M1) d(x,y) 0, ()x,yX si d(x,y) = 0 x = y; (M2) d(x,y) = d(y,x), () x, y X; (M3) d(x,z) d(x,y)+ d(y,z), () x,y,z X. Observaii. 1. Elementele unui spaiu metric se numesc puncte. 2. Inegalitatea (M3) se mai numete i inegalitatea triunghiului. 3. Pe orice mulime X se poate defini o structur de spaiu metric.In acest sens fie d : X X R, d(x,y) =
1 , daca x y 0 , daca x = y.

4. Pe aceeai mulime se pot defini mai multe metrici deci mai multe structuri de spaiu metric. 5. Dac x1, x2 ,, xnX, n 3, se arat prin inducie matematic c d(x1, xn) d(x1, x2)+ d(x2, x3)++ d(xn-1, xn). Exemple. 1. X=R, d:RRR, d(x,y)=|x-y|, ()x,yR, numit i distana euclidian. S observm c d(x,y) reprezint distana obinuit (n sensul geometriei euclidiene) ntre dou puncte M i N de pe axa real, de coordonate x si respectiv y. 2. X=Rn,n1, d:Rn RnR, d(x,y)=

(x
i =1

yi )2 , ()x,yR , x=(x1,x2,, xn),

y=(y1, y2,, yn), numit i distana euclidian. S observm c, dac n = 2

- 18 -

atunci d(x,y) =

( x1 y1)2 + ( x 2 y 2 )2 reprezint distana obinuit (n sensul

geometriei euclidiene) dintre dou puncte din plan M(x1, x2) i N(y1, y2). Pe Rn se pot defini i alte distane, de exemplu: d1(x,y) =
n

x
i =1

yi , d2(x,y) = max xi yi .
i =1,n

O clas important de spaii metrice sunt spaiile vectoriale normate. Definiia 2.1.2. Fie X/K (K = R sau C) spaiu vectorial. Se numete norma pe X o funcie real notat |||| : X R cu proprietile: (N1) ||x|| 0, () xX i ||x|| = 0 x = ; (N2) ||x|| = || ||x||, () K, () x X; (N3) ||x+y|| ||x|| + ||y||, () x, y X. Un spaiu vectorial nzestrat cu o norm se numete spaiu vectorial normat. Observaie. Un spaiu vectorial normat este un spaiu metric cu distana indus de norma astfel: d(x,y) = ||x-y||, () x, yX. Exemple. 1. X = R, ||x|| = |x|, () xR; 2. X = Rn, n 1, ||x|| =

x
i =1

2 i

, () xRn, x = (x1, x2,, xn).

Astfel ( Rn, ||.|| ) este un spaiu vectorial normat. Pe Rn se pot defini i alte norme, de exemplu: ||x||1 =
n

x , ||x|| 2 = max xi . i =1,n


i =1 i

S observm c, pentru n = 1, ||x||1 = ||x||2 = ||x|| = |x|, () x R.

- 19 -

2.2. iruri n spaii metrice


Definiia 2.2.1. Fie (X,d) spaiu metric, x X si r > 0. Mulimea notat B(x,r) = { y X: d(y,x) < r } se numete bil (sau sfer) deschis de centru x si raza r, iar B[x,r] = { yX :d(y,x) r } bil (sau sfer) nchis de centru x i raz r. Exemple. 1. X = R, d(y,x) = |x-y|, B(x,r) = {y R : |y-x| < r} = (x-r, x + r), deci bila deschis este intervalul deschis (x - r, x + r) centrat n x. Evident B[x, r] = [x - r, x + r]. 2. X = R2, d(x,y) = (x1 y1 )2 + (x 2 y 2 )2 , ()x,yR2 ,x = (x1,x2), y = (y1,y2), B(x,r) = {yR2:d(y,x)<r} ={yR2: (x1 y1 )2 + (x 2 y 2 )2 <r} ={y =(y1,y2)R2: (y1-x1)2 + + (y2-x2)2 < r2} =interiorul cercului de centru C(x1,x2) i raz r. O proprietate important a spaiilor metrice este aceea de separabilitate, adic () x,yX, x y, () r > 0 astfel nct B(x, r) B(y, r) = . ntr-adevr, fie x,yX,x y, d = d(x,y) > 0 i r = . Atunci B(x,r)B(y,r) = . Dac () z B(x, r) B(y, r), atunci 0 < d(x,y) = d d(x, z) + d(z, y) < r+ r = 2r = 2 , contradicie. Definiia 2.2.2. O mulime A X se numete mrginit dac () xX i r > 0 astfel nct A B(x, r) . O mulime V X se numete vecintate pentru xX dac () r > 0 astfel nct B(x,r) V. Fie (x) = { V X : V vecintate pentru x } = mulimea vecintilor lui x. Definiia 2.2.3. Fie (X,d) spaiu metric. Se numete ir de puncte n
d 3
d 3

spaiul metric X o funcie f :N X .Valorile acestei funcii le notm cu xn = f(n)X i pentru un ir mai folosim notaia (xn )nN sau (xn).

- 20 -

Dac (xn) este un ir n spaiul metric (X,d) si ko<k1<...<kn<... este un ir strict cresctor de numere naturale atunci irul (yn) , unde subir al irului (xn) i scriem ( xk ) (xn).
n

yn = xk se numete n

Observaie. Cum (kn) este strict cresctor rezult c kn n, () n N. Definiia 2.2.4. Un ir (xn) X este convergent ctre xX i scriem xn x sau lim xn = x dac ()V (x), () nvN astfel nct xnV, () n nv.
n

n caz contrar irul (xn) este divergent. Observaie. Dac (xn) X este un ir convergent ctre xX atunci n orice vecintate V a lui x se afl toi termenii irului (deci o infinitate) exceptnd un numr finit. Propoziia 2.2.1. Fie (X,d) un spaiu metric, (xn) X si xX.Urmtoarele afirmaii sunt echivalente: 1. lim xn = x ;
n

2. () > 0, () n N astfel nct () n n, d(xn,x) < ; 3. irul de numere reale (d(xn,x))nN este convergent ctre zero. Demonstraie. 1. 2. Fie > 0 si V = B(x, ) (x). Din 1) () n = nN astfel nct () n n, xn V = B(x, ), sau echivalent d(xn,x)< . 2. 1. Fie V (x) ; atunci ()r > 0 astfel nct B(x,r) V i folosind 2) () n = nvN astfel nct () n nv, d(xn,x) < r, sau echivalent xnB(x,r) V ()nnv. 2. 3. evident.

Definiia 2.2.5. Un ir (xn) din spaiul metric (X,d) se numete ir Cauchy (sau fundamental) dac () > 0, ()nN astfel nct ()m,nn, d(xm,xn) < , sau echivalent (lund m = n+p, cu pN): () > 0, () n N astfel nct () n n i () p 1, d(xn+p,xn) < .

- 21 -

Teorema 2.2.1. Fie (X,d) un spaiu metric. Atunci: 1. Dac un ir este convergent, limita este unic. 2. Orice ir convergent este ir Cauchy. 3. Orice ir Cauchy este mrginit. 4. Orice subir al unui ir convergent este convergent ctre aceeasi limit. Demonstraie. 1. Fie (xn) X. Presupunem prin absurd c () x, y X, x y astfel nct xn x, xn y. Cum X este separat () r > 0 astfel nct B(x,r) B(y,r) = . Cum xn x, () n1N astfel nct xnB(x, r), () n n1.Cum xn y, () n2N astfel nct xnB(y,r), () n n2. Pentru n max(n1,n2) avem xnB(x,r) B(y,r)= , contradicie. 2. Fie (xn) X, xn xX i > 0. Atunci ()n N astfel nct

() n n,d(xn,x) < . 2 Pentru m, n n, d(xm,xn) d(xm,x) + d(x,xn) < + = , deci (xn) este ir Cauchy. 2 2

3. Fie (xn) X ir Cauchy i =1. Atunci () n0 N astfel nct () m, n n0

d(xm,xn) < 1.Pentru n = n0 obinem d(xm,xn 0 ) <1, () m n0,adic xmB(xn 0 ,1), () m n0 . Lund r = max {1,d(xn 0 ,x0),...,d( xn 0 , xn 0 -1)}, rezult xn B(xn 0 ,r), () n N, deci (xn) este ir marginit. n particular, dac (xn)este convergent atunci (xn) este mrginit.
4. Fie (xn) X, xn x i (xk n ) (xn) un subir al su.

Pentru () > 0, () n N astfel nct d(xn, x) < , () n n. Cum kn n, () n N rezult c d(xk n ,x) < , () n n, adic xk n x.

- 22 -

2.3 iruri n spaii metrice particulare


Proprietile generale ale irurilor n spaii metrice rmn valabile i n spaiile metrice particulare R, Rk, k 2. n aceste spaii, pe lng proprietile generale irurile au i proprieti specifice.
A. iruri de numere reale

Fie spaiul metric X = R i d(x, y) = |x - y|, distana euclidian. Conform definiiei 2.2.2 o mulime V R este vecintate pentru xR dac ( ) r > 0 astfel nct B(x,r) = (x-r,x+r) V. Din propoziia 2.2.1 un ir de numere reale (xn) R este convergent ctre xR () > 0, () n N astfel nct | xn-x | < , () n n . Conform definiiei 2.2.5 un ir (xn) R este ir Cauchy dac () > 0, () nN astfel nct ()m,n n, | xm-xn | < , sau echivalent (lund m = n+p, cu pN) : () > 0, () n N astfel nct () n n, () p 1, | xn+p-xn | < .
Definiia 2.3.1. Un ir (xn) R se numete monoton cresctor (respectiv

descresctor) dac xn+1 xn (respectiv ), () nN. Un ir (xn) R se numete monoton dac este monoton cresctor sau monoton descresctor. Dac inegalitile sunt stricte, (xn) se numete strict cresctor (respectiv strict descresctor).
Definiia 2.3.2. Un ir (xn) R este marginit inferior (respectiv superior)

dac mulimea termenilor si este minorat (respectiv majorat), adic dac () R (respectiv R) astfel nct xn (respectiv xn ), () n N. Un ir (xn) R este mrginit dac este mrginit inferior i superior, adic dac () , R astfel xn , () n N, sau echivalent: () M > 0 astfel nct |xn| M, () n N. Un ir (xn) R este nemrginit dac nu este mrginit.

- 23 -

Prezentm n continuare cteva rezultate cunoscute referitoare la convergena irurilor:


Teorema 2.3.1. Fie (xn), (yn) dou iruri de numere reale astfel nct

xnxR, yn yR. Atunci irurile (xn + yn), ( xn), cu R, (xnyn), yn 0 , sunt convergente i
1. lim ( xn + yn ) = x + y;
n

xn , cu yn

2. lim (xn ) = x ;
n n

3. lim ( xn yn ) = xy ; 4. lim
xn x = , daca y 0. n yn y

Teorema 2.3.2. (Teorema cletelui). Dac (xn), (yn), (zn) sunt trei iruri de

numere reale astfel nct xn yn zn, () n N i lim x n = lim zn = x R , atunci


n n

(yn) este convergent i lim yn = x .


n

Teorema 2.3.3. (Cesro). Orice ir mrginit de numere reale are cel puin

un subir convergent.
Teorema 2.3.4. (Weierstrass). 1. Orice ir de numere reale monoton cresctor i mrginit superior este

convergent iar limita sa este marginea superioar a mulimii termenilor si.


2. Orice ir de numere reale monoton descresctor i mrginit inferior este

convergent iar limita sa este marginea inferioar a mulimii termenilor si. n continuare vom arta c noiunile de ir numeric convergent i ir Cauchy sunt echivalente.
Teorema 2.3.5. (Cauchy). Un ir de numere reale este convergent dac i

numai dac este ir Cauchy.

- 24 -

Demonstraie. i) Fie (xn) R, xn x R. Vom arata ca (xn) este sir Cauchy.


i atunci 2

Fie > 0; cum xn x, () nN astfel incat () n n, | xn-x | <

pentru m,n n vom avea | xm - xn | = | xm - x + x - xn | | xm x | + | xn x | < < + = , deci (xn) este sir Cauchy. 2 2
ii) Reciproc, s presupunem c (xn) este ir Cauchy i vom arta c

()xR astfel ncat xn x. Cum (xn) este sir Cauchy, din teorema 2.2.1 (xn) este mrginit i din teorema 2.3.3 irul (xn) conine un subir convergent (xk n ); fie xR, limita sa. Vom arta ca xn x. Fie > 0; cum xk n x, () nN astfel nct | xk n -x | < Pentru n n, cum kn n n vom avea | xn-x | = | xn - xk n + xk n -x | | xn - xk n | + | xk n - x | < i demonstraia este ncheiat.
Exemplu. Fie sirul cu termenul general x n =
n

, () n n. 2
+ = 2 2

sin kx , xR. Vom arta ) k =1 k (k + 1

c (xn) este ir Cauchy, deci convergent. Fie > 0, n, p N. Vom avea:


xn +p xn = + sin(n + 1)x sin(n + p)x 1 + ...... + + ... (n + 1)(n + 2) (n + p)(n + p + 1) (n + 1)(n + 2)

1 1 1 1 1 1 1 = + + ...... + = n + p n + p +1 (n + p)(n + p + 1) n + 1 n + 2 n + 2 n + 3 1 1 1 1 1 = < < , daca n + 1 > n > 1. n +1 n + p +1 n +1

1 Fie n = [ ] i atunci () n n i () p 1 vom avea (xn) este ir Cauchy.

x n + p xn < , deci

- 25 -

iruri cu limita + sau - Definiia 2.3.3. O mulime V R se numete vecintate pentru + dac

()aR astfel nct (a,+ ) V. O mulime V R se numete vecintate pentru - dac () a R astfel

nct (- , a) V.
Definiia 2.3.4. Un ir (xn) R are limita +,
lim xn = + , dac

() V(+), () nv N astfel nct xnV, () n nv. Analog definim iruri cu limita - .


Observaie. Lund vecinti pentru + de forma ( ,+), cu > 0 rezult

c lim x n = + () > 0, () n N astfel nct xn > , () n n.


n

Analog lim xn = () > 0, () n N astfel nct xn < -, () n n.


n

Prezentm n continuare cteva rezultate referitoare la irurile cu limita + sau -.


Teorema 2.3.6. 1. Dac (an) R este un ir cu limita + iar (xn) este astfel nct an xn,

() nN, atunci () lim xn = + .


n

2. Dac (bn) R este un ir cu limita - iar (xn) este astfel nct xn bn,

() n N, atunci () lim xn = .
n

Teorema 2.3.7. 1. Dac lim xn = + i lim y n = y R \ {} atunci () lim ( xn + y n ) = + .


n n n

2. Dac lim xn = i lim y n = y R \ {+} atunci () lim ( xn + y n ) = .


n n
n

3. Dac lim xn = + i lim yn = y > 0 atunci () lim ( xn yn ) = + .


n n

4. Dac lim xn = + i lim yn = y < 0 atunci () lim ( xn yn ) = .


n n n

- 26 -

Teorema 2.3.8. 1. Dac (xn) este un ir cresctor i nemrginit superior atunci


n

lim xn = + .

2. Dac (xn) este un ir descresctor i nemrginit inferior atunci


n

lim xn = .

Observaie. Din teroremele 2.3.4 i 2.3.8 rezult c orice ir monoton de

numere reale are limit n R .


Limit superioar i limit inferioar

Fie (xn) un ir de numere reale i An = {xn, xn+1,}, nN. S observm c An An+1, () nN. Fie yn = supAn = sup xk, zn = infAn = inf xk .
k n k n

Se observ c irul (yn) este descresctor n timp ce irul (zn) este cresctor i atunci (yn) i (zn) au limit n R .
Definitia 2.3.5. 1. Elementul y R , y = lim yn se numete limit superioar a irului (xn) i
n

se noteaz y = lim sup xn sau y = lim xn .


n n

2. Elementul z R , z = lim zn se numete limit inferioar a irului (xn) i


n

se noteaz z = lim inf xn sau z = lim xn .


n
n

Observaie. Cum lim yn = inf yn rezult c


n

nN

lim sup xn = inf sup xk i analog


nN k n nN k n

lim inf xn = sup inf xk . n . 2

Exemplu. Fie irul cu termenul general xn = cos

- 27 -

Atunci zn = inf { cos


n

k k : k n} = -1, () n N i yn = sup { cos : k n} = 1, 2 2


n

() nN deci lim inf xn = -1 , lim sup xn = 1.


Observaie. Cum zn yn , () nN rezult c lim zn lim yn i atunci
n

lim inf x n lim sup xn .


n

Teorema 2.3.9. Un ir (xn) R are limit n R dac i numai dac


n

lim inf xn = lim sup xn .


n

n acest caz lim xn = lim inf xn = lim sup xn .


n n n

Demonstraie. 1. Presupunem c (xn) are limit n R i fie x = lim xn . Deosebim cazurile:


n

a) xR; atunci pentru () > 0, () n N astfel nct () n n

| xn-x | <

, de unde xn < x + , () n n . 2 2 i cum (yn) este descresctor, 2

De aici rezult c yn = sup {xn : n n} x+


pentru () n n , yn yn x + . 2

Trecnd la limit cu n obinem


n

lim xn = lim yn x+
n

< x + , deci lim xn < x + . n 2

Cum > 0 este arbitrar rezult c lim xn x.


n

Analog se arat c lim x n x i cum lim x n lim xn va rezulta c x = lim x n = lim xn .


n n

b) lim xn = +; atunci () > 0, () n N astfel nct xn > , () n n.


n

De aici rezult c zn = inf { xn


xn

+1, }

i cum (zn) este cresctor


n

zn zn ,

() n n, de unde lim zn = + , adic lim inf xn = + .


n

- 28 -

Cum lim xn lim xn , rezult c lim xn = + i deci lim x n = lim xn =+ .


n

c) Cazul lim xn = se trateaz analog.


n

2. Reciproc, presupunem c lim xn = lim x n = x


n n

i vom arta c

() lim xn = x . Deosebim cazurile:


n

a)

x R; n acest caz avem lim yn = x i deci () > 0, () nN


n

astfel nct |yn-x| < , () n n . Cum xn sup x k = y n , rezult c


k n

xn < x + , () n n .
n

(2.1)
n

Pe de alt parte, cum lim x n = x rezult c lim zn = x , deci () > 0, () nN astfel nct |zn-x| < , () n n . Cum xn zn, () n N, rezult c xn > x - , () n n .
lim xn = x .

(2.2)

Pentru n n = max{n,n}, din (2.1) i (2.2) vom avea |xn-x| < , () n n , deci
n

b) x = +; atunci din lim zn = + rezulta ca pentru () > 0,


n

() n N astfel nct zn > , () n n . Cum xn zn , () n rezulta ca xn > , () n n , deci lim xn = + .


n

c) Cazul x = - se trateaza analog.


_

Definitia 2.3.6. Fie (xn) un sir de numere reale.Un punct x R se numeste

punct limita pentru sirul (xn) daca in orice vecinatate V a lui x se afla o infinitate de termeni ai sirului (xn). Fie A = { x R : x punct limita pentru sirul (xn)}.
_

- 29 -

Observatie. Din definiie rezult imediat c x R este punct limita pentru

irul (xn) dac i numai dac exista un subir xkn (xn ) astfel nct lim xkn = x .
n

( )

Exemplu. Fie x n = cos

1 n , () n N . Atunci A = 0, ,1 . 2 3

Observaie. Dac (xn) este un ir de numere reale se arat c


n

lim inf xn = inf A i lim sup xn = sup A .


n

Teorema 2.3.10. Fie (xn) un ir de numere reale strict pozitive. Atunci


n

lim inf

x n +1 x lim inf n xn lim sup n xn lim sup n +1 . n n n xn xn

Demonstraie. Inegalitatea din mijloc este evident. O vom demonstra pe

prima, iar a treia se demonstraz analog. Fie


= lim inf
n

x n +1 , = lim n x n . Vom arta c . Dac = 0, n xn

inegalitatea este evident. S presupunem deci > 0 i fie > 0 astfel nct > 0 . Cum = lim inf
n

x n +1 x = sup inf k +1 > , exist un rang n0 astfel nct xn n k n x k

k n0

inf

xk +1 > , de unde rezult c xk xk +1 > , () k n0 xk Fie n n0; vom avea x n = (2.3)

xn +1 xn xn 1 0 xn0 i folosind (2.3) rezult xn 1 xn 2 xn0


n n0

xn > ( )
n

xn0 , () n n0 ,
n0

sau echivalent x n > ( ) a , unde a = ( ) Obinem atunci


n n

xn0 > 0 .

xn > ( )n a, () n n0
n

i prin trecerea la limit

inferioar = lim inf n xn lim inf ( )n a = Cum > 0 este arbitrar rezult i demonstaia este ncheiat.

- 30 -

Observaie. Din teoremele 2.3.9. si 2.3.10. rezult c dac (xn) este un ir

de numere reale strict pozitive astfel nct () lim

x n +1 = L ,unde 0 L + atunci xn

() nlim n xn

=L.

B. iruri n spaiul metric Rk , k 2

Fie spaiul metric Rk cu distana euclidian


d(x, y ) = x y =
2 (xi yi ) , ()x, y Rk , x = (x1,x2,,xk), y =(y1,y2,,yk). i =1 k

2 xn = x1 , xn ,..., xk Rk , deci unui ir (xn) din Rk i corespunde k iruri de numere n n


2 reale (x1 ) , (x n ) ,..., (x k ) numite iruri componente. Conform propoziiei 2.2.1. un ir n n

Dac (xn) este un ir in Rk atunci termenul su general xn este de forma

(xn) din Rk este convergent ctre xRk dac i numai dac () > 0, () nN astfel nct || xn x|| < , () n n. Un ir (xn) din Rk este ir Cauchy (sau fundamental) dac i numai dac () > 0, () nN astfel nct () m, n n avem xm xn < .
2 Teorema 2.3.11. Un ir (xn) din Rk, x n = x1 , xn ,..., xk este convergent n n n
2 Rk dac i numai dac irurile componente (x1 ), (xn ), ..., (xk ) sunt convergente n n

n R.
Demonstraie. Presupunem c (xn) este convergent n Rk i fie
n

lim xn = x Rk , x = (x1, x 2 ,..., xk ) .Vom arta c x in xi, () i = 1, k.

Fie i 1, k si > 0; cum x n x, () n astfel nct Cum

x n x < , () n n .

xin xi x n x , () n rezult c () n n , xin xi xn x < ,

adic xin xi.


Presupunem c x in xi R, () i = 1, k i vom arta c

xnx, unde x = (x1,x2,,xk)Rk.

- 31 -

Fie > 0; cum x in xi , () ni astfel nct () n ni , x in xi < Lund n = max { n1, n2,..., nk } rezult c () n n vom avea
xn x =

(x
k i =1

i n

xi

<

2 = , deci xnx. k

Observaie. Din aceast teorema rezult c n cazul convergenei avem


2 lim xn = lim x1 , lim xn ,..., lim xk . n n n n n n
n n n , n ,n 2 . Exemplu. Fie (xn) R , x n = n + 1

Avem xn =
n

2 x1 , xn n

), unde
1 1 1 + n

x1 n

= n,

2 xn

Cum lim x1 = lim n n = 1 rezult c x1 este convergent. n n


n 2 Cum lim xn = lim n n

( )

n = . n + 1

1 2 rezult c xn este convergent. e

( )

1 n concluzie (xn) este convergent n R2 i lim xn = 1, . n e

Observaie. Raionnd asemntor ca n demonstraia teoremei 2.3.11. se


2 arat c un ir (xn) din Rk, xn = x1 , xn ,..., xk este ir Cauchy n Rk dac i numai n n
2 dac irurile (x1 ) , (xn ) , ..., (xk ) sunt iruri Cauchy n R. n n

2.4. Spaii metrice complete


Fie (X,d) un spaiu metric. Din teorema 2.2.1. rezult c, dac (xn) este un ir de puncte din X, convergent, atunci el este ir Cauchy. Reciproca nu este n general adevrat.
Exemplu. Fie X = (0,1], d(x,y) = |x-y|, xn =
1 , () n 1. Cum (xn) este n

convergent n R rezult c (xn) este ir Cauchy n R, deci n X.

- 32 -

Cum lim xn = 0 X rezult c (xn) nu este convergent n X.


n

Definiia 2.4.1.

Un spaiu metric (X,d) n care orice ir Cauchy este

convergent se numete spaiu metric complet.


Definiia 2.4.2. Un spaiu vectorial normat i complet se numete spaiu

Banach.
Observaie. Folosind teorema lui Cauchy rezult c R, Rk, k 2 sunt spaii

Banach relativ la distana euclidian.


Definiia 2.4.3. Fie (X,d) un spaiu metric. O funcie f : X X se numete

contracie dac exist [0,1) astfel nct d(f(x),f(y)) d(x,y), () x,y X.


Teorema 2.4.1.

(Principiul contraciei sau teorema de punct fix a lui

Banach). Fie (X,d) un spaiu metric complet i f : X X o contracie. Atunci f are un unic punct fix, adic () X, unic, astfel nct f() = .
Demonstraie. Unicitatea. Presupunem prin absurd c exist 1,2 X,

1 2 astfel nct f(1) = 1 i f(2) = 2. Atunci 0 < d(1, 2) = d(f(1),f(2)) d(1, 2), de unde rezult 1, contradicie.
Existena. Fie x0 X, fixat i construim irul (xn) astfel :

x1 = f(x0), x2 = f(x1),, xn = f(xn-1), irul (xn) se mai numete i irul aproximaiilor succesive. Vom arta c irul (xn) este ir Cauchy. Fie d = d(x1, x0). Observm c: d(x1,x2) = d(f(x0), f(x1)) d(x0,x1) = d; d(x2,x3) = d(f(x1), f(x2)) d(x1,x2) 2d. Prin inducie se arat c: d(xn,xn+1) nd, () nN. Pentru nN i p 1 avem d(xn,xn+p) d(xn,xn+1) + d(xn+1,xn+2)++ d(xn+p-1,xn+p)

- 33 -

nd + n+1d++n+p-1d = nd

1 p n d. 1 1

(2.4)

Dac d = 0 rezult x1 = x0, deci f(x0) = x0 i demonstraia este ncheiat cu = x0 . Dac d > 0 , cum [0,1) rezult c lim n = 0 i atunci () >0,
n

() nN astfel nct n <

1 , () n n. d

Folosind (2.4) rezult c () n n i () p 1 avem d(xn,xn+p)< , deci (xn) este ir Cauchy. Cum (X,d) este complet () X astfel nct xn . Vom arta c f() = . Evident avem d(,f()) d(,xn+1)+d(xn+1,f()) = d(,xn+1)+ d(f(xn),f()) d(,xn+1)+ d(xn,) i cum lim d( xn , ) = 0 , prin trecere la limit cu n obinem d(,f()) = 0, adic
n

f() = .

2.5. Elemente de topologie n spaii metrice


Fie (X,d) un spaiu metric.
Definiia 2.5.1. O mulime nevid D X se numete deschis dac D este

vecintate pentru orice punct al su, adic dac pentru ()xD, () r > 0 astfel nct B(x,r) D. Prin definiie este o mulime deschis. O mulime F X se numete nchis dac CF = X\F este deschis.
Exemple. 1. Dac (X,d) este un spaiu metric atunci i X sunt i deschise i

nchise.
2. Dac (X,d) este un spaiu metric atunci orice bila deschisa este o

multime deschisa si orice bila inchisa este o multime inchisa. Intr-adevar, fie D = B(x,r), x X, r > 0 i yD. Fie 0 < r1 < r-d(y,x). Atunci B(y,r1) D, deoarece daca zB(y,r1) d(z,y) <r1
d(z,x) d(z,y)+d(y,x) < r1+d(y,x) < r, deci z B(x,r) = D.

- 34 -

Analog se arat c B[x,r] este o mulime nchis . n particular, dac X = R, d distana euclidiana, atunci orice interval de forma (x-,x +) cu > 0 este multime deschisa si orice interval de forma [x-,x+] este o mulime nchis.
3. Dac X = R cu metrica euclidiana, intervalele de forma [a,b], [a,),

(-,b] sunt multimi inchise.


4. Mulimea A = {xR :1 < x 2} nu este nici nchis nici deschis. 5. Mulimea A = {(x,y)R2 : 1< x < 3, -1 < y < 1} este deschis iar

B = {(x, y) R2: x2 - y2 1} este nchis.


Teorema 2.5.1. Fie (X,d) spaiu metric. Atunci 1. O reuniune oarecare de mulimi deschise este o mulime deschisa iar o

intersecie finit de mulimi deschise este o mulime deschis.


2. O reuniune finita de multimi inchise este o multime inchisa iar o

intersectie oarecare de multimi inchise este o multime inchisa.


Demonstraie. 1. Fie (Di)iI o familie oarecare de submulimi deschise ale

lui X i D =

U Di .
iI

Dac D = atunci D este deschisa. Dac D , fie xD; atunci () i0 I astfel incat x D i 0 i conform definitiei () r > 0 astfel incat B(x,r) D i 0 D, deci D este deschis. Fie D1, D2,, Dn, nN*, mulimi deschise ale lui X i D = Dac D = atunci D este deschis. Dac D , fie xD; atunci xDi, () I = 1, n i conform definitiei ()ri > 0, i= 1, n astfel ncat B(x,ri) Di, () i = 1, n . Fie r = min ri. Atunci B(x,r) B(x,ri) Di,
i=1,n

ID
i=1

() i = 1, n , de unde B(x,r)

ID
i=1

=D, deci D este deschisa.

2. Rezult imediat din 1) folosind formulele lui de Morgan.

- 35 -

Observaii. 1. Dac (X,d) este un spaiu metric, submulimile lui X formate

dintr-un singur element sunt mulimi nchise i folosind teorema 2.5.1 rezult c mulimile finite sunt nchise, fiind o reuniune finit de mulimi nchise.
2. O intersectie oarecare de multimi deschise nu este in general o multime
1 1 deschisa.In acest sens fie X = R cu metrica euclidiana si Dn = (- , ), nN*. n n

Evident Dn este deschisa () n N* si

n =1

I Dn = {0}, care nu este deschisa.

3. Fie (X,d) spatiu metric si d = {D X: D multime deschisa} P(X).

Atunci d are urmatoarele proprietai (ce rezulta din teorema 2.5.1): (T1) X, d; (T2) Orice reuniune de multimi din d apartine lui d; (T3) Orice intersectie finita de multimi din d apartine lui d. Spunem in acest caz ca d este o topologie pe X numita topologia indusa de metrica d iar perechea (X, d) se numeste spatiu topologic. In particular daca X = Rn, n 1 si d este distanta euclidiana atunci topologia indusa de d o vom nota cu 0 si o vom numi topologia uzuala sau naturala a lui Rn. In continuare, ori de cate ori vom vorbi de Rn fara a face vreo mentiune speciala il vom considera inzestrat cu topologia naturala. Fie (X,d) spatiu metric si A X, nevid.
Definiia 2.5.2. Un punct xA se numeste punct interior multimii A daca

() V(x) astfel incat V A, sau echivalent () r > 0 astfel incat B(x,r) A. Notam cu A (sau IntA) = {xA: x punct interior multimii A} si o numim interiorul lui A.
Definiia 2.5.3. Un punct xX se numete punct de aderen pentru
0

mulimea A dac ()V(x), AV , sau echivalent ()r > 0, AB(x,r) . Notm cu A = {xX: x punct de aderen pentru A} i o numim aderen sau nchiderea lui A.
_

- 36 -

Definiia 2.5.4. Un punct xX se numete punct de acumulare pentru A

daca () V(x), A (V\{x}) , sau echivalent () r > 0, A (B(x,r)\{x}) . Notm cu A = {xX :x punct de acumulare pentru A} si o numim multimea derivata a lui A. Un punct xA care nu este punct de acumulare pentru A se numeste izolat pentru A. Notam cu IzA = {xA : x punct izolat pentru A} si o numim multimea punctelor izolate ale lui A. Se observa ca un punct xA este izolat daca si numai daca ()V(x) astfel incat A V = {x}. Definim multimea FrA = A CA , numita si frontiera lui A.
Exemple. 1. Fie X = R i A X, A = (-,1) U (2,3] U {5}. Atunci
A = (-,1) U (2,3), A = (-,1] U [2,3] U {5}, A = (-,1] U [2,3],
0 _

IzA = {5}, FrA = {1,2,3,5}.


2. X = R, A = (0,1) Q. Atunci A =, A =[0,1], A=[0,1], IzA = , FrA =[0,1] 3. (X,d) spaiu metric i A X, finit. Atunci
A =, A =A, A = , IzA = A, FrA = A.
0 0

Observaie.

Din definiiile date rezult imediat c, dac (X,d) este un

spaiu metric i A, B X sunt nevide atunci:


1. A A A , A A ; 2. A B A B , A B , A B;
6o8 7 3. A U B A B , A B = A U B , (AUB) = A U B;
o o
0 0

6o8 o o 7 4. A B = A B , A B A B .

Cum primele dou proprieti sunt evidente, s artm de exemplu a doua egalitate de la proprietatea 3); celelalte se demonstreaz analog.

- 37 -

Fie x A B ; dac x A U B atunci x A i x B , deci ()r1,r2 > 0 astfel nct A B(x,r1) = , B B(x,r2) = . Lund r = min(r1,r2) > 0 obinem (AUB) B(x,r) = (A B(x,r)) U (BB(x,r)) = , care contrazice faptul c x A B . Reciproc, dac x A U B , cum A AUB, B AUB, rezult A A B , B A B , deci x A B .
Teorema 2.5.2. Fie (X,d) spaiu metric i A X, nevid. Atunci
o } 1. C A = CA i C A = CA .

2. A este deschis i A este nchis. 3. A este deschis A = A . 4. A este nchis A = A . Demonstraie. 1. x C A x A () r > 0, B(x, r) A () r > 0, B(x, r) CA
o } x CA . Analog se arat c C A = CA .

2. Fie x A ; atunci () r > 0 astfel nct B(x, r) A. Vom arta c B(x,r)

A ; fie yB(x,r) i 0 < r1 < r - d(y,x). Atunci B(y,r1) B(x,r) A, deci y A .

Rezult deci c A este deschis.


o } Cum C A = CA , care este deschis rezult c A este nchis.

3. evident; Presupunem c A este deschis i vom arta c A = A . Cum A A


o o

este suficient s artm c A A . Fie xA; cum A este deschis () r > 0 astfel nct B(x,r) A, adic x A .
o

- 38 -

4. evident; Presupunem c A este nchis i vom arta c A = A . Cum A A

este suficient s artm c A A. Fie x A ; dac xA rezult xCA i cum CA este deschis () r > 0 astfel nct B(x,r) CA, deci B(x,r) A = , care contrazice faptul c x A .

Teorema 2.5.3.(de caracterizare a punctelor aderente cu ajutorul irurilor).

Fie (X,d) spaiu metric i A X, A . Atunci x A exist un ir (xn) A astfel nct xn x.


1 Demonstraie. Fie x A ; atunci () nN*, A B(x, ) , deci n 1 1 () xnA B(x, ), () nN*. Obinem astfel un ir (xn) A cu d(xn,x)< , n n

() nN*, adic xn x. Reciproc, dac (xn) A, xn x, atunci () > 0, () nN astfel nct () n n, xnB(x,). Astfel () > 0, B(x, ) A , adic x A .

innd cont de definiia punctului de acumulare, analog cu teorema 2.5.3. obinem:


Teorema 2.5.4.(de caracterizare a punctelor de acumulare cu ajutorul

irurilor). Fie (X,d) spaiu metric i A X, A . Atunci xA exist un ir (xn) A, xn x, () nN astfel nct lim xn = x .
n

Definiia 2.5.5. Fie (X, d) spaiu metric i A X, A . O familie (Di) iI de

pri ale lui X se numete acoperire a mulimii A dac A U Di.


iI

Dac J I i A U Di spunem c (Di)


iJ

iJ

este o subacoperire a lui (Di)

iI

pentru A. Dac toate mulimile Di, iI, sunt deschise spunem c (Di)iI este o acoperire deschis.

- 39 -

Definiia 2.5.6. O submulime K a unui spaiu metric (X,d) se numete

compact dac din orice acoperire deschis se poate extrage o subacoperire finit.
Exemple. 1. X = R, K = (0,1).Atunci K nu este compact. Este evident c

1 familia (In)nN*, unde In = ( ,1) formeaz o acoperire deschis pentru K. n Daca K ar fi compacta () J N* finita astfel incat K U In. Luand m = maxJ
nJ

rezulta ( 0,1) U In = (
nJ

1 ,1) contradictie. m

2. (X,d) spatiu metric, K = {x1, x2,, xn} X finita. Atunci K este compacta.

Intr-adevar, fie (Di) iI o acoperire deschis pentru K, deci K U Di.


iI

Cum xiK, exist ki I, astfel nct xi D k i , i = 1, n . Atunci (D k i ) i=1,n este o subacoperire finit pentru K.
Teorema 2.5.5. Orice submultime compacta a unui spatiu metric este

inchisa si marginita.
Demonstratie. Fie K X compact i xK. Evident avem K U B(x,n),
nN*

deci (B(x,n))n1 este o acoperire deschisa pentru K. Cum K este compacta exista J N*, finita astfel incat K U B(x,n). Fie n0 = maxJ si atunci vom avea
nJ

K B(x,n0), deci K este marginita. Vom arata in continuare ca multimea K este inchisa sau echivalent K = K . Cum K K este suficient sa aratam ca K K. Presupunem prin absurd ca
K K, deci exista x K si xK.

Cum xK rezulta ca () yK avem y x i cum X este separat, pentru () yK, () ry > 0 astfel incat B(x, ry) B(y, ry) = . Familia (B(y, ry))yk formeaza o acoperire deschisa pentru K, deci ()y1, y2,, ynK astfel inct K
n

U B( yi,ryi ) . Dar B(x,r


i =1

yi

) B(y,r y ) = , () i = 1, n .
i

- 40 -

Lund r = min r y obtinem B(x,r) B(yi,r y ) = , () i = 1, n , de unde


i =1,n
i i

B(x,r) K = , contradictie.

Observaie. Reciproca acestei teoreme nu este in general adevarata. Se

poate arata ca, daca X este finit dimensional atunci K X este compacta daca si numai daca K este inchisa si marginita. Rezulta ca o multime K Rn este compacta daca si numai daca este inchisa si marginita.
Teorema 2.5.6. Fie (X,d) spatiu metric. Atunci K X este compacta daca

si numai daca orice sir de puncte (xn) din K are un subsir convergent la un punct din K.
Demonstraie. Presupunem mai intai ca K este compacta si fie (xn) un sir

de puncte din K. Presupunem prin absurd ca (xn) nu contine nici un subsir convergent; deci multimea termenilor sai nu are nici un punct de acumulare. Atunci pentru orice xK, () rx > 0 astfel incat B(x,rx) contine cel mult un numar finit de termeni ai sirului (xn) (in caz contrar ar exista un subsir al lui (xn) care sa convearga la x). Famlia (B(y,ry))yk constituie o acoperire deschisa pentru K si cum K este compacta se poate extrage o subacoperire finita, deci exista y1, y2,, ymK si r1, r2,, rm > 0 astfel incat K
m

U B( yi, ri ) . Cum (xn) K


i =1

U B( yi,ri )
i =1

iar fiecare

dintre bilele B(yi,ri) contine cel mult un numar finit de termeni ai sirului (xn) rezult ca sirul (xn) are un numar finit de termeni, contradictie. Pentru a demonstra reciproca acestei teoreme se poate consulta [11].

Teorema 2.5.7. Fie (X,d) spatiu metric, K X, compacta si A K, inchisa.

Atunci A este compacta.


Demonstratie. Fie (Di)iI o acoperire deschisa a lui A. Cum X\A este

deschisa si K U Di (X\A) rezult c familia ((Di)iI, X\A) este o acoperire


iI

- 41 -

deschisa pentru K. Cum K este compacta () i1, i2,, inI astfel incat K

k =1

U Dik (X\A)

i atunci A U Dik deci ( Di k ) k1,n este o subacoperire finita


k =1

pentru A.
Probleme propuse 1. Fie X = N* i d: X X R, d(x,y) =
ln

x . Sa se arate ca ( X,d ) este y

un spaiu metric i s se determine B(3,

1 1 ), B[3, ]. 2 2

2. S se arate c distana euclidiana d:Rn RnR, d(x,y) =

(x
i =1

yi ) ,
2

()x,yRn, x = (x1, x2,, xn), y = (y1, y2,, yn) este o distanta pe Rn. Pentru n = 2 sa se determine imaginea geometrica a bilei B((2,-1),3) relativ la d si la distanta d1, unde d1(x,y) = |x1-y1| + | x2-y2|, () x,yR2, x = (x1, x2), y = (y1,y2).
3. Fie X = L = {(xn) R : (xn) marginit} i d : X X R, d(x,y) =

= sup xn yn , ()x,yX, x = (xn), y = (yn). Sa se arate ca (X,d) este un spatiu


n1

metric.
4. Fie (X1,d1), (X2,d2) spatii metrice, X = X1 X2, d: XXR,
2 d(x,y) = d1 ( x1, y1) + d2 ( x 2 , y 2 ) , () x, y X, x = (x1,x2), y = (y1,y2). S se arate c 2

(X,d) este un spatiu metric.


5. Fie d1, d2 metrici pe X. Spunem ca d1 si d2 sunt echivalente, d1 d2 daca
d1 = d2 . Sa se arate ca d1 d2 () m, M > 0 astfel incat

md1(x,y) d2(x,y) M d1(x,y), () x, yX.


6. Sa se arate ca pe Rn urmatoarele metrici sunt echivalente:

d(x,y) =

( xi yi )2 , d1(x,y) = xi yi , d2(x,y) = max xi yi , i =1,n


i =1
i =1

() x, y Rn, x = (x1,x2,,xn), y = (y1,y2, , yn). Mai mult, orice dou metrici pe Rn, sau mai general, pe un spatiu finit dimensional sunt echivalente.

- 42 -

7. Fie(X,d) spatiu metric, (xn), (yn) X doua siruri Cauchy. Atunci sirul

(d(xn,yn)) este un sir Cauchy in R+.


8. Sa se arate ca L este un spatiu Banach relativ la norma ||x|| = sup xn ,
n 1

() x = ( xn)n1.
9. Fie ( X, d) spatiu metric, A X, compacta, a X \ A. Atunci

d(a, A) = sup d(a, x ) > 0 si exista x0A asfel incat d(a, A) = d(a, x0).
xA

1 1 10. Fie sirurile date prin termenul general xn = (1+ )n, yn = (1+ )n+1. Atunci n n
a) irul (xn) este strict crescator iar sirul (yn) este strict descrescator. b) Sirurile (xn) si (yn) sunt convergente la o aceeasi limita. Limita comuna

se noteaza cu e. Sa se arate ca 2 < e < 3.


c) Folosind sirurile (xn), (yn) sa se arate ca sirul cu termenul general
1 1 ++ - ln n, este convergent. Limita sa se noteaz cu c. Sa se arate 2 n

zn = 1+

ca c (0,1).
11. Sa se studieze convergenta sirurilor date prin termenul general: a) xn =
n

k =1

1 ; b) xn = k2

k =1

1 ; k

c) xn = ak qk , unde |q| < 1, |ak| 2, () k 1.


k =1

12. Fie x0 > 0 i xn = (xn-1 +

1 2

a xn1
a .

), () n 1, unde a > 0. Sa se arate ca

(xn) este convergent si are limita

13. Fie (xn), (yn) R astfel incat (yn) este strict crescator si divergent .Sa

se arate ca daca () lim

xn xn +1 xn = L R , atunci exista si lim = L. n yn n y n +1 y n

(Lema lui Stolz-Cesaro)


14. Sa se arate, folosind exercitiul 13, ca daca (xn) este un sir de numere
x n +1 = L R , atunci exista si lim n x n = L. n xn

strict pozitive iar lim

- 43 -

15. Fie (xn) un sir de numere reale. Sa se arate ca (xn) contine cel putin un

subsir monoton. Sa se deduca de aici ca multimea punctelor limita a unui sir este nevida.
16. Sa se calculeze lim inf x n , lim sup x n , pentru
n

a) xn = sin

n n ; b) xn = (-1)n . 4 2n + 1

17. Daca (xn), (yn) R sunt doua siruri marginite, atunci: a) lim sup( x n + yn ) lim sup xn + lim sup yn ;
n n n

b) lim inf( x n + yn ) lim inf xn + lim inf yn ;


n n n

c) lim sup( xn yn ) ( lim sup xn )( lim sup yn )(xn 0, yn 0);


n n n

d) lim inf( x n y n ) ( lim inf xn )( lim inf yn )(xn 0, yn 0).


n n n

18. Sa se arate

ca sirul cu termenul general

xn = (

n! 1 2 1 + 2 + ... + n , (cos )n , ), e convergent si sa se determine limita sa. n n n n


1 sa fie o x +a
2

19. Sa se determine a > 0 astfel incat f : R R, f(x) =

contractie.
20. Fie f :[a, b] [ a,b ] derivabila astfel incat sup |f(x)| = < 1. Atunci f
x[ a,b]

este o contractie.
21. Sa se arate ca ecuatia x3 + 12x - 1= 0 are o singura radacina pe [0, 1].

Sa se determine aceasta radacina cu aproximaie de patru zecimale.


22. Sa se studieze daca urmatoarele submultimi ale lui R2 sunt marginite,

deschise, inchise.
a) A = [0,1) x (1,2]; b) A = {(x,y)R2 : x = y, y[1,3] }; c) A = {( x,y)R2 : x2 - y2 < 1}; d) A = [1,2) ({
1 : n 1} (2, 3] ) . n
o

Pentru fiecare sa se indice A , A , A , IzA, FrA..

- 44 -

CAPITOLUL 3
SERII DE NUMERE REALE

3.1. Serii convergente. Serii divergente


Fie (xn )n1 un ir de numere reale. irului (xn )n1 i atam irul (Sn )n1 , unde S1 = x1, S2 = x1 + x2, ..., Sn = x1 + x2 +...+ xn,... n cazul n care irul (Sn ) este convergent i are limita S se poate scrie
S = lim Sn = lim (x1 + x 2 + ... + xn ) = x1 + x 2 + ... + x n + ... = x n , deci se d un
n n n =1

sens sumei cu o infinitate de termeni.


Definiia 3.1.1. Se numeste serie de termen general
x n perechea de

iruri (( x )n1, (Sn )n1 ) , unde (Sn ) se numete irul sumelor pariale. O serie se noteaz formal astfel

n =1

xn , *xn
nN

sau x1+ x2+...+ xn+...

Definiia 3.1.2. O serie xn se numete convergent, pe scurt (C), dac


n =1

irul sumelor pariale (Sn) este convergent.n acest caz S = lim Sn se numete
n

suma seriei i scriem

n =1

xn = S .

n caz contrar seria se numete divergent, pe scurt (D).Prin natura unei serii nelegem proprietatea sa de a fi convergent sau divergent.

- 45 -

Exemple. 1. Fie seria

n =1

qn , q R , numit i seria geometric. n acest caz


2 n

q qn +1 xn = q i Sn = x1 + x 2 + ... + xn = q + q + ... + q = , pentru q 1 i 1 q


n

Sn= n pentru q =1. Rezult c (Sn) este convergent dac i numai dac q < 1 i n acest caz lim Sn =
n

q . 1 q

Rezult, deci, c seria geometric acest caz

n =1

qn

este convergent q < 1 i n

n =1

qn = 1 q .
n =1

2. Fie seria

n , numita i seria
xn =

armonic.

n acest caz

1 1 1 i Sn = 1 + + ... + . Vom arta c (Sn) nu e ir n 2 n

Cauchy. Presupunem prin absurd c (Sn) este ir Cauchy, deci () > 0, ()n N astfel nct ( ()m, n n avem Sm Sn < . Lund =
() m, n n0 avem
Sm Sn <

1 , ()n0 N astfel nct 2

1 . Pentru m = 2n0 i n = n0 obinem 2

S2n 0 Sn 0 =

1 1 1 1 < . + + ... + n0 + 1 n 0 + 2 2n0 2

Pe de alta parte,

1 1 1 1 1 + + ... + > n0 . = , contradicie. n0 + 1 n0 + 2 2n0 2n0 2

n concluzie (Sn) este divergent, deci seria armonic este divergent.

- 46 -

Proprietai generale ale seriilor


Teorema 3.1.1. Dac unei serii
n =1

xn

i se elimin sau i se adaug un

numr finit de termeni atunci natura seriei nu se schimb; n caz de convergen se modific doar suma.
Demonstraie. Presupunem c eliminm termenii
xi1, xi2 ,...xik ,

unde

i1< i2 <...< ik. Fie (Sn) irul sumelor pariale asociat seriei

n =1

xn

i (Tn) irul sumelor

pariale asociat seriei obinute prin eliminarea termenilor xi1, xi2 ,...xik . Evident, avem Tn = Sn-s, () n > ik unde s = xi1 + x i2 + ...xik , de unde rezult c (Tn) este convergent dac i numai dac (Sn) este convergent i deci seria
nN* \ A

xn , unde A = {i1, i2,,i k} are aceeai natur cu seria iniial xn .


n =1

Cellalt caz se trateaz analog.


Teorema 3.1.2. (Condiia necesar de convergen).

Dac seria

n =1

xn

este convergent, atunci lim xn = 0.


n

Demonstraie. Fie Sn = x1+x2+...+xn i S = lim Sn. Atunci xn = Sn - Sn-1,


n

() n 2 , de unde lim xn = S S = 0.
n

Observaii. 1. Reciproca nu este n general adevrat.

n acest sens considerm seria armonic


lim xn = lim

n =1

1 , care este divergent i n

1 = 0. n n

- 47 -

2. Dac pentru o serie

n =1

xn
n

avem c lim xn 0 sau nu exist, atunci


n

seria este divergent.


Exemplu. Fie seria

2n + 1.
n =1

n acest caz xn =

n 1 0 , deci seria 2n + 1 2

2n + 1
n =1

este divergenta.

Teorema 3.1.3. (Criteriul lui Cauchy).

Seria

n =1

xn

este convergent dac i numai dac pentru ( ) > 0, () n N


xn +1 + xn + 2 + ... + xn + p < .

astfel nct ( ) n n i ( ) p 1 avem

Demonstraie. Fie Sn = x1 + x2 +...+ xn. Vom avea

n =1

xn (C) (Sn )

este

convergent iar (Sn) este convergent dac i numai dac


() > 0 () n N astfel nct () n n i ( ) p 1 avem
Sn + p Sn < () > 0,

este ir Cauchy

() n N

astfel nct

() n n i () p 1 avem

xn +1 + xn + 2 + ... + xn + p < .

Exemplu. Fie seria

n =1

n(n + 1), x R.

cos nx

Vom arta c aceast serie este

convergent folosind criteriul lui Cauchy. Fie > 0, n, p N* . Vom avea


xn +1 + x n + 2 + ... + xn + p = cos(n + 1)x cos(n + p )x + ... + (n + 1)(n + 2) (n + p)(n + p + 1)

1 1 1 + + ... + = (n + 1)(n + 2 ) (n + 2 )(n + 3 ) (n + p )(n + p + 1) 1 1 1 1 1 1 = + + ... + = n +1 n + 2 n + 2 n + 3 n + p n + p +1


1 1 1 < < , n +1 n + p +1 n +1

- 48 -

1 1 dac n + 1 > , adic n > 1.

Fie n = i atunci () n n , ( ) p 1 vom avea xn+1 + xn + 2 + ... + xn+p < , deci seria

n(n + 1)
n =1

cos nx

este convergenta.

Teorema 3.1.4. Dac seriile

n =1

xn

n =1

yn
n =1

sunt convergente i , R ,

atunci seria

n =1

(xn + yn ) este convergent i (xn + yn ) = xn + yn .


n =1 n =1

Demonstraie. Rezult imediat folosind irurile sumelor pariale asociate

seriilor date.

3.2. Serii cu termeni pozitivi


Definiia 3.2.1. O serie
n0 *

n =1

xn

se numete cu termeni pozitivi dac exist De obicei considerm n0 = 1, deci

astfel nct

x n 0, () n n0 .

x n 0, () n 1 .

Observaie. Dac seria

n =1

xn este cu termeni

pozitivi, atunci irul

sumelor pariale (Sn) este monoton cresctor. ntr-adevr, Sn+1 - Sn = xn+1 0, ( ) n 1.


Teorema 3.2.1. (Criteriul monotoniei). O serie

n =1

xn

cu termeni pozitivi

este convergent dac i numai dac irul sumelor pariale (Sn ) este mrginit superior.

- 49 -

Demonstraie. evident.
" " Presupunem c (Sn ) este mrginit superior. Cum (Sn )

este i monoton cresctor, din teorema lui Weierstrass rezult c este convergent deci seria

n =1

xn

este convergent.

S remarcm faptul c, dac seria cu termeni pozitivi divergent atunci lim Sn = + i astfel vom scrie
n

n =1

xn

este

n =1

xn = + .

Exemplu. Fie seria

n
n =1

, > 1, numit i seria Riemann (sau seria

armonic generalizat). Vom avea xn = 2m n < 2m+1. Vom avea :


1 1 1 1 1 1 1 1 1 Sn = 1 + + + + + + + + + ... + + ... 3 4 5 6 7 8 9 15 2 1 1 1 1 1 + + + ... + < 1 + 2 + 4 + ... 2m 2m + 1 n 2 4

1 1 1 i Sn = 1 + + ... + . Fie nN* i mN astfel nct n 2 n

( ) ( ) (2 )
1
m

+ 2m

1 1 1 2 = 1 1 1 2

m +1

< 1

1 1 2 1

2 1 2 1 1

deci (Sn ) este mrginit superior i atunci seria

n =1

1 este convergenta. n

Teorema 3.2.2.( Criteriul de comparaie de spea I ).

Fie seriile

x , y ,
n =1 n n =1 n

unde 0 xn yn , ()n 1. Atunci: este convergent, rezult c i seria

a) Dac seria

n =1

yn

n =1

xn

este

convergent.

- 50 -

b) Dac seria

n =1

xn este divergent, rezult c i seria


n n

n =1

yn

este

divergent.
Demonstraie. Fie Sn = x k , Tn = y k . Cum x n y n , ( ) n 1 rezult
k =1 k =1

Sn Tn , ( ) n 1.

( 3.1 )

a) Dac seria

y
n =1

este convergenta atunci (Tn) este convergent, deci

mrginit superior i din (3.1), rezult c (Sn) este mrginit superior. Folosind teorema 3.2.1 rezult c seria
b) Prin reducere la absurd, folosind a). Exemplu. Fie seria

n =1

xn

este convergent.

n =1

1 , < 1. n

Evident avem x n = rezult c i seria

1 1 , () n 1 i cum seria n n

n =1

1 este divergenta n

n =1

1 este divergenta. n

n concluzie seria Riemann divergent pentru 1.

n =1

1 n

este convergent pentru > 1 i

Teorema 3.2.3. (Criteriul de comparaie de spea a-II-a).

Fie seriile

xn,
n =1

y , cu x
n n =1

> 0, yn > 0, () n 1 i

x n +1 y n +1 , () n 1. xn yn

(3.2)

Atunci
a) Dac seria

n =1

yn

este convergent, rezult c i seria

n =1

xn

este

convergent.
b) Dac seria

n =1

xn

este divergent, rezult c i seria

n =1

yn

este divergent.

- 51 -

Demonstraie. Din (3.2) vom avea


x2 y2 x3 y3 x y , , ..., n n , ( ) n 2 . nmulind membru cu membru aceste x 1 y1 x 2 y 2 x n 1 y n 1

inegaliti obinem

xn yn x , adic x n 1 yn , ( ) n 1. Demonstraia se ncheie y1 x1 y1

folosind teorema 3.2.2.


Teorema 3.2.4. (Criteriul comparaiei la limit).

Fie seriile
lim

xn ,
n =1

y
n =1

, cu xn , yn > 0, () n 1. Presupunem c exist

xn = L . Atunci yn

a) Dac L (0, ) rezult c cele dou serii au aceeai natura. b) Dac L = 0 i seria

n =1

yn

este convergent, rezult c i seria

n =1

xn yn

este convergenta, iar dac seria este divergent.


c) Dac L = + i seria

n =1

xn

este divergent, rezult c i seria

n =1

n =1

xn

este convergent, rezult c i seria

n =1

yn

este convergent, iar dac seria

n =1

yn

este divergent, rezult c i seria

n =1

x n este divergent.

Demonstraie. a) Dac L (0, ) , () n0 N , astfel nct


L 3L yn < xn < y n , ( ) n n0 . 2 2 L x n 3L < < , ( ) n n0 , sau echivalent 2 yn 2

Demonstraia se ncheie folosind teorema 3.2.2.

- 52 -

b) Cum L = 0, () n0 N , astfel nct

xn 1, ( ) n n 0 sau echivalent yn

x n y n , () n n0 i demonstraia se ncheie folosind din nou teorema 3.2.2.

c) Dac L = +, () n0 N astfel nct

xn 1, ( ) n n0 sau echivalent yn

x n yn , () n n0 i demonstraia se ncheie folosind din nou teorema 3.2.2.

Exemplu. Fie seria

n =1

1 3n + 1

. Cum seria

n =1

1 este divergenta i 1 n2 1 3n + 1

lim

1 3n + 1 = 1 (0, ) , rezult c 1 3 n 2

n =1

este divergenta.

Teorema. 3.2.5. (Criteriul condensrii).

Fie ( xn ) un ir descresctor de numere pozitive. Atunci seriile

n =1

xn

n =0

2n x 2n au aceeai natur.
n =1

Demonstraie. Fie ( Sn) irul sumelor pariale asociat seriei

xn

i (Tn)

irul sumelor pariale asociat seriei


2m n < 2m +1 .Atunci

n=0

2 x
n

2n

. Fie n N* i m N astfel nct

Sn = x1+x2++xn x1+x2++x 2

m +1

=
2m 2 m + 1 1

= x1 + (x2+x3)+(x4+x5+x6+x7)++(x ++ x
x 1 + 2x 2 + 2 2 x 2 2 + ... + 2 m x 2m =Tm , adic

) (3.3)

Sn Tm .

Cum 2m n avem
Sn = x1 + x 2 + ... + xn x1 + x 2 + ... + x 2m =

= x1 + x 2 + ( x 3 + x 4 ) + ( x5 + x 6 + x 7 + x 8 ) + ... + ( x 2m1+1 + ... + x 2m )


x1 + x 2 + 2x
22

+ 22 x

23

+ ... + 2m 1x

2m

1 Tm , care mpreun cu (3.3) 2

conduce la

1 Tm Sn Tm , de unde rezult imediat concluzia teoremei. 2

- 53 -

Exemplu. Fie seria

n =2

1 . n ln n

Avem xn = descresctor.

1 , deci (xn) este un ir de numere pozitive monoton n ln n


1 1 = , care este divergent , n n 2 ln 2 n =1 n ln2

Pe de alt parte

2n x 2n = 2n
n =1 n =1

deci seria dat este divergent.


Teorem 3.2.6. (Criteriul raportului sau al lui DAlambert).

Fie seria

n =1

xn, xn > 0, ()n 1. Atunci


x n +1 k, ()n n 0 rezult c seria xn

a) Dac () k [0, 1) i n0 N astfel nct

n =1

xn

este convergent.
xn +1 1,() n n0, rezult c seria xn

b) Dac ()n0 N astfel nct

n =1

xn

este

divergent.
Demonstraie. a) Fr a restrnge generalitatea putem presupune n0 = 1

i atunci
xn kx n 1 k 2 x n 2 ... k n 1x1, ()n 1.

Cum

k n1x1 = x1 kn1 , care este convergenta, din teorema 3.2.2.


n =1 n =1

rezult ca i seria

x
n =1

este convergenta.

b) Vom avea

xn xn-1 xn-2 x n > 0,() n n0, deci lim xn 0 i din teorema 3.1.2 rezult
0

c seria este divergent.

- 54 -

Consecin. (Criteriul raportului la limit).

Fie seria

n =1

xn , cu xn>0, () n 1. Presupunem c () lim


n =1 n =1

x n +1 = L. Atunci xn

a) Dac L < 1 rezult c seria b) Dac L > 1 rezult c seria

xn xn

este convergent. este divergent.


xn +1 k, xn

Demonstraie. a) Fie L<k<1;atunci ()n0 N astfel nct


() n n0 i folosind teorema 3.2.6 rezult c seria

n =1

xn

este convergent.

b) Din definiia limitei () n0 N astfel nct

x n +1 1, () n n0 i folosind xn

teorema 3.2.6 rezult c seria este divergent.


Exemplu. Fie seria

n =1

n2 . 2n

n2 x 1 Atunci x n = n > 0, ( )n 1 i lim n +1 = < 1, deci seria este convergent. n xn 2 2

Teorema 3.2.7.(Criteriul rdcinii sau al lui Cauchy).

Fie seria

n =1

xn, xn > 0, ()n 1. Atunci


n

a) Dac ()k [0,1) i n0 N astfel nct

xn k, () n n0, rezult c seria

n =1

xn

este convergent.
xn 1, () n n0 rezult c seria

b) Dac () n0 N astfel nct

n =1

xn

este divergent. .

- 55 -

Demonstraie. a) Vom avea x n k n , ( )n n0 i cum seria

k
n =1

este

convergent, folosind teorema 3.2.2 rezult c i seria deci seria

n =n o

este convergenta,

x
n =1

este convergenta.
n

b) Vom avea xn 1, () n n0 , deci lim xn 0 i din teorema 3.1.2 rezult c

seria este divergent.


Consecin. (Criteriul rdcinii la limit).

Fie seria

x , x
n =1 n

> 0, () n 1. Presupunem c exist lim n x n = L . Atunci


n

a) Dac L<1, rezult c seria b) Dac L>1, rezult c seria

n =1

xn
n =1

este convergent. este divergent.

xn

Demonstraie. a) Fie L<q<1 i din definiia limitei () n0 N , astfel nct


n

x n q , () n n0 i

folosind teorema 3.2.7 rezult c seria este convergent.


b) Din definiia limitei () n0 N astfel nct
n

x n 1 () n n0 i din teorema ,

3.2.7 rezult c seria

x
n =1

este divergenta.
n
n

n n Exemplu. Fie seria > 0, ( ) n 1 i . Avem x n = 2n 1 n =1 2n 1


n

lim n x n = lim

n 1 = < 1, deci seria n 2n 1 2

x
n =1

este convergenta.

- 56 -

Teorema 3.2.8. (Criteriul lui Raabe-Duhamel).

Fie seria

x , x
n =1 n

> 0, () n 1. Atunci
xn 1 k, ( ) n n0 , rezult c x n +1

a) Dac exist k>1 i n0 N astfel nct n

seria

n =1

xn

este convergent.
xn 1 1, ( ) n n0 , rezult c seria x n +1

b) Dac exist n0 N , astfel nct n

n =1

xn

este divergent.

Demonstraie. a) Fr a restrnge generalitatea, putem presupune n0 = 1 i atunci


nx n nx n +1 kx n +1, ()n 1, de unde x1 x 2 kx 2 2x 2 2x 3 kx 3 .......... .......... ..... nx n nx n +1 kx n +1

Adunnd aceste inegaliti obinem


x1 + x 2 + ... + xn nx n +1 k( x 2 + x 3 + ... + xn +1) , adic
Sn nx n +1 k(Sn + x n +1 x1 ), () n 1 de unde , Sn (k 1) kx1 nx n +1 kx n +1 < kx1, () n 1, i atunci Sn < kx1 , () n 1, k 1

adic irul (Sn) este mrginit superior i conform teoremei 3.2.1 seria convergent.

n =1

xn

este

b) Presupunem din nou no=1 i atunci nx n nx n +1 xn +1, ( ) n 1, de unde


x n +1

1 1 n 1 n 2 n 1 n x n , ( ) n 1 i xn xn 1 ... x1 = x1 , () n 1 . n +1 n 2 n n 1 n
1 n x1 este divergenta, din teorema 3.2.2 rezulta c seria n =1

Cum seria divergenta.

x
n =1

este

- 57 -

Consecin. (Criteriul lui Raabe-Duhamel la limit).

Fie seria Atunci

x , x
n =1 n

x > 0, () n 1. Presupunem c exist lim n n 1 = L . n x n +1

a) Dac L>1, rezult c seria b) Dac L<1, rezult c seria

n =1

xn xn

este convergent. este divergent.

n =1

Demonstraie. a) Fie L > k > 1. Din definiia limitei ()no N astfel nct
x n n 1 k, ( ) n n0 i folosind teorema 3.2.8 x n +1

rezult c seria

x
n =1

este

convergenta.
b) Din definiia limitei

() no N , astfel nct

x n n 1 1, ( ) n n0 i x n +1

folosind teorema 3.2.8 rezult c seria

n =1

xn

este divergent.

Exemplu. Fie seria

1.3.5...(2n 1) . 2.4.6...(2n) n =1

n acest caz x n =

1 3 5 ... (2n 1) > 0, () n 1 i 2 4 6 ... (2n )

x 2n + 2 1 lim n n 1 = lim n n 2n + 1 1 = 2 < 1, deci seria n x n +1


Teorema 3.2.9. (Criteriul logaritmic).

n =1

xn

este divergent.

Fie seria

x , x
n n =1

> 0, () n 1. Atunci
ln

1 xn a) Dac () k > 1 i n0 N astfel nct k, ( ) n n0 rezult c seria ln n

n =1

xn

este convergent.

- 58 -

1 xn b) Dac () n0 N , astfel nct 1, ( ) n n0 , rezult c seria ln n ln

n =1

xn

este divergent.
Demonstratie. a)
ln

Fr a restrnge generalitatea, putem presupune n0 = 1 si atunci

1 1 1 k ln n, ( ) n 1 , de unde nk , ( ) n 1 sau echivalent x n k , () n 1. , xn xn n

Cum seria

n =1

nk
n =1

este convergent (deoarece k>1), folosind teorema 3.2.2 este convergent.


1 ln n, ( ) n 1 , de unde xn

rezult c i seria

xn

b) Presupunem din nou n0 = 1 i atunci ln


xn 1 , ( ) n 1, i cum seria n

n
n =1

este divergenta, folosid din nou teorema 3.2.2

rezult c i seria

x
n =1

este divergenta.

Folosind teorema 3.2.9 i definiia limitei se obine imediat criteriul logaritmic la limit:
1 xn Fie seria xn , xn > 0, () n 1. Presupunem c () lim = L . Atunci: n ln n n =1

ln

a) Dac L > 1, rezult c seria b) Dac L < 1, rezult c seria Exemplu. Fie seria

n =1

xn xn
.

este convergent. este divergent.

n =1

n =1

n2

ln n

- 59 -

1 2 ln n ln (ln n) xn ln n n acest caz, x n = 2 > 0, ( ) n 2 i lim = lim = 2 > 1, n ln n n n ln n ln

de unde rezult c seria

x
n =1

este convergenta.

3.3. Serii cu termeni oarecare


Definiia 3.3.1. O serie

n =1

xn, xn R

se numete cu termeni oarecare

dac are o infinitate de termeni pozitivi i o infinitate de termeni negativi.


Definiia 3.3.2. O serie

n =1

xn

cu termeni oarecare se numete absolut

convergent( pe scurt (AC)), dac seria modulelor O serie

n =1

xn

este convergent.

n =1

xn

cu termeni oarecare se numete semiconvergent dac este

convergent, dar nu este absolut convergent.


Teorema 3.3.1. O serie absolut convergent este convergent.

Demonstraie. Presupunem c seria

n =1

xn

este absolut convergent. Din

criteriul lui Cauchy, pentru () > 0, () n N astfel nct () n n i ( ) p 1 avem xn +1 + xn + 2 + ... + xn + p < i atunci () > 0, () n N astfel nct () n n i ( ) p 1 avem xn +1 + xn + 2 + ...xn + p xn +1 + xn + 2 + ... + x n + p < i folosind din nou criteriul lui Cauchy rezult c seria

x
n =1

este convergenta.

- 60 -

Exemplu. Fie seria


xn = sin nx n +1
3

n =1

sin nx

sin nx , x R. n acest caz x n = n3 + 1 n3 + 1


3 n =1 n 2

1 n +1
3

1 n
3 2

, ( ) n 1. Cum

(C) folosind teorema 3.2.2,

rezult c

n =1

xn (C) , deci xn ( A.C) i din teorema 3.3.1 rezult c


n =1

n =1

x n (C) .

Teorema 3.3.2. (Criteriul lui Dirichlet).

Fie seria

n =1

xn

cu irul sumelor pariale mrginit. Dac (yn) este un ir

monoton descresctor cu limita zero, atunci seria


n n

n =1

xn yn

este convergent.

Demonstraie. Fie Sn = xk , Tn = xk yk i M>0 astfel nct


k =1 k =1

Sn M, ( ) n 1.

Pentru n, p N* vom avea:


Tn +p Tn = x n +1 y n +1 + x n + 2 y n + 2 + ...x n +p y n +p = = y n +1 (S n+1 S n ) + y n + 2 (S n+ 2 S n+1 ) + ... + y n+p (S n+p S n+p 1 ) = = y n+1S n + S n +1 (y n +1 y n + 2 ) + ... + S n+p 1 (y n +p 1 y n+p ) + y n +p S n+p

yn +1 Sn + Sn +1 (yn +1 yn + 2 ) + ... + Sn + p 1 (yn + p 1 yn + p ) + yn + p Sn + p Myn +1 + M(yn +1 yn + 2 ) + ... + M(yn + p 1 yn + p ) + Myn + p = = 2Myn +1 + M(yn +1 yn + 2 ) + ... + M(y n + p 1 yn + p ) + Myn + p = 2Myn +1 (am folosit

faptul c yn>0 i (yn) monoton descresctor). Fie > 0 ; cum y n 0, () n N astfel nct () n n , avem yn <
i 2M

atunci () n n i () p 1, Tn + p Tn 2Myn +1 < , deci (Tn) este ir Cauchy. Rezult, deci, c (Tn) este convergent, adica seria convergent.

n =1

xn yn

este

- 61 -

Teorema 3.3.3. (Criteriul lui Abel).

Dac seria atunci seria

n =1

xn

este convergent i (yn) este un ir monoton i mrginit,

n =1

xn yn

este convergent.

Demonstraie. Fr a restrnge generalitatea, vom presupune c (yn) este

monoton i descresctor cu limita y R . Fie zn = yn y, ( ) n 1; atunci (zn) este un ir monoton descresctor cu limita zero. Cum seria

n =1

xn

este convergent,

are irul sumelor pariale mrginit, i folosind criteriul lui Dirichlet rezulta c seria

n =1

x n zn

este convergent

Avem x n yn = x n (zn + y ) = xn zn + yxn , ( ) n 1, i cum seriile sunt convergente, rezulta c i seria

n =1

x n zn ,

n =1

yxn

n =1

xn yn

este convergent.

3.4. Serii alternate


Definiia 3.4.1. O serie
n =1

xn

se numeste serie alternat dac xnxn+1< 0,

() n 1.
Observatie. O serie alternat poate fi scris n una din urmtoarele dou

forme:

( 1)
n =1

n 1

xn , sau

( 1) x
n =1

, unde xn > 0, () n N* .

Teorema 3.4.1. (Criteriul lui Leibniz).

Dac (xn) este un ir monoton descrescator cu limita zero, atunci seria

n =1

( 1)

n 1

xn este convergent.

- 62 -

Demonstraie. Cum seria

n =1

( 1)

n 1

are irul sumelor pariale mrginit

(Sn=0 pentru n par i Sn=1 pentru n impar) iar irul (xn) este monoton descresctor i convergent la zero, rezult, conform criteriului lui Dirichlet, c seria

n =1

( 1)

n 1

x n este convergent.

Exemplu. Fie seria


xn =

( 1)
n =1

n 1

1 . Seria se scrie sub forma n

( 1)
n =1

n 1

xn , unde

1 > 0, ()n 1. Cum (xn) este monoton descresctor i lim xn = 0 rezult, n n

conform criteriului lui Leibniz, c seria

n =1

( 1)

n 1

1 este convergent. n

Observaie. Cum seria modulelor

1 n 1 1 ( 1) n = n este divergent
n =1 n =1

rezult c seria este semiconvergent.

Probleme propuse
1. Folosind definiia, s se studieze natura seriilor: a) b)
1 n(n + 1) ; b) n =1

1 n(n + 1)(n + 2) , c) n =1

nq ,
n n =1

q < 1;

n=0

n + 2 2 n + 1 + n ; e)

n =1

sin

3 cos n + 2 ; f) 2n + 2 2

n =1

cos

n . 2

2. Folosind criteriile de convergen, s se studieze natura seriilor: a)


n n + 1 n =1

; ;

b) d)

n =1

sin n
n =1

;
1

ln n c) n =1 n

2n4 + n + 1

e) sin ; n n =1

f)

n=2

n+2 n2 , k R ; nk

- 63 -

n + 1 g) n n =1 3
i) k)

; ;

n! 3n h) n n =1 n

;
n!

(n!)2 2n (2n)! n =1

j)

a(a + 1)...(a + n 1) , a > 0


n =1

n =1

a(a + 1)...(a + n 1) b(b + 1)...(b + n 1)xn, a, b, x > 0 ;

an l) n ,a>0 ; n n =1 2 + 3

2n + 1 m) ; n =1 3n 2

n) p) r)

n=2

1
n

3 2n + 1

o) q) s)
n

ln n ; n n =1

n=1

cosnx n3 + x2

, x R ; 1 ; 3n + 2
n ; 2n 1

sin nx ; n n =1

n =1

n 1 ( 1)

n=2

( 1)

1 . n

t)

n =1

( )

n(n +1) 1 2

3. S se determine valorile parametrului real x pentru care seriile

urmtoare sunt convergente:


1 x a) ln x ; b) . n =1 n = 0 1 + x
n

4. S se arate c seria

2n 1 5an +1 , a < 3 este convergent i s se 3n n =1

determine suma sa.


5. Folosind, eventual, seriile s se arate c:
nk a) lim n = 0 , unde k>0, a>1; n a

b) irul cu termenul general x n =

cos k! este convergent. 2k k =1


n

- 64 -

6. S se arate c seria

n =0

n!

este convergent i are suma e.

7. S se arate ca, dac seria

n =1

nxn

converge atunci i seria

n =1

xn

converge.
8. S se arate c, dac seria

n =1

xn

este convergent, atunci

seria

xn este absolut convergent. n =1 n

- 65 -

CAPITOLUL 4
FUNCII NTRE SPAII METRICE
4.1. Limita unei funcii ntr-un punct
Fie (X, d1 ) , (Y, d2 ) dou spaii metrice. Dac a X, b Y, r > 0 vom nota cu B(a, r ) i B(b,r ) bilele deschise n X i

respectiv n Y. Problema limitei ntr-un punct consta n cercetarea comportrii funciei n vecintatea unui punct fixat, mai precis ce se ntmpl cu valorile funciei atunci cnd argumentul su se apropie "din ce n ce mai mult" de punctul fixat. Deoarece comportarea funciei n vecintatea unui punct fixat se poate pune chiar dac funcia nu este definit n acel punct, dar trebuie s existe puncte din vecintatea lui n care funcia s fie definit, punctul dat trebuie s fie de acumulare. Fie f : A X Y i a A .
Definiia 4.1.1. Un punct L Y se numete limita funciei f n punctul a i se

noteaz L = lim f (x ) dac () V (L ), () U (a) astfel nct ( ) x U A, x a


x a

rezult f (x ) V .
Teorema 4.1.1. (de caracterizare a limitei ntr-un punct).

Fie f : A X Y , a A i L Y . Urmtoarele afirmaii sunt echivalente


a) L = lim f (x ) ;
x a

b) ( ) > 0, () > 0

astfel

nct

() x A, x a cu

d1(x, a ) <

rezult

d2(f(x), L) < ;
c) ( )(xn ) A, xn a, xn a rezult c f (xn ) L .

- 66 -

Demonstraie. a) b) Fie > 0

i V = B(L, ) (L ) ; atunci
f (x ) V .

() U (a ) astfel nct
astfel nct

() x U A, x a rezult

Cum

U (a ), () > 0

B(a, ) U i atunci ()x A, x a cu d1(x, a ) < rezult x B (a, ) U i deci f (x ) V = B(L, ) , adic d2 ( f (x ), L ) < .

b) c) Fie

(xn ) A, xn a, xn a

i > 0; din b) () > 0 astfel nct

()x A, x a cu d1(x, a ) <

avem d2 ( f (x ), L ) < .

Cum x n a, () n N astfel nct ( ) n n avem d1(xn, a) <


xn B(a, ), ( ) n n i atunci d2 ( f (x n ), L ) < , ( ) n n ,adic f (x n ) L .

c) a) Presupunem prin absurd c ()V (L) astfel nct ( ) U (a )

()xU U A, xU a

astfel nct f (xU ) V . Lund Un = B a, (a ) , () n 1

1 n

rezult c exist un ir

(xn ) B a, 1 A , xn a ,
n

deci xn a astfel nct

f (x n ) V, ( ) n 1 , care contrazice faptul c f (x n ) L .

Observaie. Din

aceast teorem rezult c, dac exist dou iruri


n n

(xn ), (xn ) A \ {a}, convergente ctre a astfel nct lim f (xn ) lim f (xn ) sau cel
puin una nu exist atunci funcia f nu are limit n punctul a.
Exemplu.

Fie

funcia

f : R2 \ {(0,0)} R ,

f (x, y ) =

xy . x + y2
2

Fie

irul

(zn) R2 \ {(0,0)}, zn = , (0,0 ) , unde , R. n n


, adic limita irului (f (zn )) depinde de i Avem f (zn ) = 2n n 2 = 2 + 2 + n2 n 2 . Astfel, spre exemplu fie

1 1 1 zn = , ( 0,0 ) i lim f (zn ) = ; n 2 n n

- 67 -

2 2 1 z'n = , (0,0 ) i lim f (z'n ) = , deci ( ) lim f (x, y ) . n ( x,y ) (0,0 ) 5 n n

Observaie. Din definiia cu iruri a limitei unei funcii ntr-un punct i

folosind faptul c limita unui ir de puncte dintr-un spaiu metric este unic, rezult c dac f : A X Y are limit n a A atunci aceast limit este unic. Dac X = R, Y = R i f : A R R , f = f (x ) , f se numete funcie real. Dac X = Rk , k 2, Y = R i f : A Rk R, f = f(x1,x2,...,xk), f se numete funcie real de variabil vectorial x = (x1, x 2 ,..., xk ), sau de k variabile reale x1, x2,...,xk . Dac X = R, Y = Rm , m 2 i f : A R Rm, f = f (x ) , atunci f (x ) Rm , () x A , deci f (x ) este de forma f (x ) = (f1(x ), f2 (x ),..., fm (x )) , ( ) x A , unde f1, f2,..., fm : A R . n acest caz, funcia f este de forma f = (f1, f2,..., fm ) i o numim funcie vectorial de variabil real. Dac X = Rk , k 2, Y = Rm , m 2 i f : Rk Rm, f = f (x ) atunci f este de forma f = (f1, f2,..., fm ) i o numim funcie vectorial de variabil vectorial
x = (x1, x 2 ,..., x k ) sau de k variabile reale x1, x2,...,xk.

Folosind definiia limitei cu iruri i teorema de caracterizare a convergenei unui ir n spaiul metric Rm , m 1, se obine imediat
Teorema 4.1.2. O funcie f : A Rk Rm , f = (f1, f2 ,..., fm ) are limit n
a A egal cu L = (L1, L 2 ,..., Lm ) Rm dac i numai dac exist simultan
lim fi (x ) = Li , i = 1, m .
x a

n acest caz lim f (x ) = lim f1(x ), lim f2 (x ) ,..., lim fm (x ) .


x a x a x a x a

- 68 -

n continuare vom considera funcii cu valori reale f : A (X, d) R .


Teorema 4.1.3. Fie f, g : A X R i a A . Dac
lim g( x ) = L 2 R atunci
x a

() lim f (x ) = L1 R , x a

a) exist lim (f + g)(x ) = L1 + L 2 ;


x a

b) exist lim (fg)(x ) = L1L 2 ;


x a

c) exist lim (x ) = 1 , dac L 2 0 i g(x ) 0 , () x A . x a g L2 Demonstraie. Rezult imediat folosind definiia cu iruri a limitei unei funcii

ntr-un punct i proprietile cunoscute de la iruri.


Teorema 4.1.4. (Criteriul majorrii).

Fie f , g : A X R , a A i V (a ) astfel nct


f (x ) L g (x ), ()x V A, x a , unde L R.

Dac () lim g(x ) = 0 atunci () lim f (x ) = L .


x a
x a

Demonstraie. Fie (xn ) A, xn a, xn a .

Cum x n a , () n0 N , astfel

nct x n V, () n n0 i atunci f (xn ) L g(xn ), ( ) n n0 . Cum () lim g(x ) = 0 rezult g(xn ) 0 i atunci f (xn ) L , deci () lim f (x ) = L .
x a x a

Exemplu. Fie limita

( x,y ) (0,0

lim

(x )

+ y 2 sin

1 . x + y2
2

Cum ( x 2 + y 2 ) sin
lim

1 x 2 + y 2 , ()(x, y ) (0,0 ) i 2 x +y
2

( x, y ) (0,0

(x )

+ y 2 = 0 rezult c

(x,y )(0,0

lim

(x )

+ y 2 sin

1 = 0. x + y2
2

n continuare vom considera funcii reale de variabile reale, f : A R R . Dac a este punct de acumulare pentru mulimea A1 = A ( , a ) = {x A : x < a}

- 69 -

atunci vom spune c a este punct de acumulare din stnga pentru A, adic este satisfcut definiia punctului de acumulare, utilizndu-se doar punctele x A cu

x < a.
Analog, dac a este punct de acumulare pentru mulimea
A 2 = A (a, ) = {x A : x > a}, atunci vom spune c a este punct de acumulare

din dreapta pentru A.


Definiia 4.1.2. Fie f : A R R , a A , i A1 = A ( , a ) .

Funcia f are limit la stnga n punctul a, egal cu Ls dac f

A1

are limit n

punctul a, adic () V (L s ) , () U (a ) astfel nct ()x U A1, x a , avem


f (x ) V .

Vom nota aceasta prin L s = lim f (x ) sau f (a 0 ) , x a


x <a

x a

lim f (x ) .

n mod analog se definete limita la dreapta n a care se noteaz


L d = lim f (x ) sau f(a + 0), lim f (x ) .
x a x >a x a +

Limitele la stnga i la dreapta ntr-un punct se numesc limite laterale. Teorema 4.1.5. Fie f : A R R i a A1 . Atunci L s = lim f (x ) dac i numai dac pentru orice ir strict cresctor x a
x <a

(xn ) A1 cu

xn a avem f (xn ) L s .

Demonstraie. Presupunem c exist Ls = lim f ( x ).


x a x< a

Din teorema de caracterizare a limitei ntr-un punct cu iruri, rezult c, n particular, pentru orice ir strict cresctor (xn) A1 cu x n a avem f(xn) Ls. Reciproc, s presupunem c pentru orice ir strict cresctor (xn ) A1 cu xn a avem f (xn ) L s . Fie (xn ) A1 un ir arbitrar (deci xn < a ) cu xn a . Atunci f (xn ) L s , deoarece n caz contrar extragem un subir xkn (xn ) strict cresctor cu limita a i atunci f (xn ) L s , contradicie.

( )

- 70 -

n mod analog se arat


Teorema 4.1.6. Fie f : A R R i a A . 2

Atunci L d = lim f (x ) dac i numai dac pentru orice ir strict descresctor x a


x >a

(x n ) A

cu xn a avem f (xn ) L d .

Legtura dintre limita unei funcii ntr-un punct i limitele laterale n acel punct este dat de:
Teorema 4.1.7. Fie f : A R R i a A1 A . 2

Funcia f are limit n punctul a dac i numai dac exist limitele laterale i acestea sunt egale. n acest caz L = Ls = Ld.
Demonstraie. Dac ( ) L = lim f (x ) atunci rezult imediat c () L s = L i
x a

Ld = L. Reciproc, presupunem c
L s = lim f (x ) = L d = lim f (x ) .
x a x <a x a x >a

Vom

arta

() lim f (x ) = L = Ld = L s . x a
Fie > 0 ; atunci () ' > 0, " > 0 astfel nct

() x A, x < a cu

x a < ' f (x ) L s < i

() x A, x > a

cu x a < " f (x ) L d < .

Lund = min(' , '' ) rezult c pentru ()x A, x a cu x a < avem


f (x ) L s < , unde Ls = Ld, deci () lim f (x ) = L = L s = L d .
x a

Observaie. Fie f : A R R i a A . Dac a = i L = , pentru a

defini lim f (x ) = L folosim aceeai definiie 4.1.1 cu remarca c n acest caz


x a

considerm vecinti ale punctului . Trecnd la caracterizarea cu iruri va rezulta c lim f (x ) = L dac i numai dac ( )(x n ) A, x n a , cu lim x n = a
xa

(n R ) rezult c lim f(xn ) = L (n R ).


n

- 71 -

4.2. Funcii continue


Fie (X, d1 ) , (Y, d2 ) spaii metrice i f : A X Y .
Definiia 4.2.1.

Funcia

este

continu

punctul

aA

dac

() V (f (a )), () U (a ) astfel nct () x U A rezult f (x ) V .


n caz contrar, funcia f este discontinu n a A . Intuitiv, funcia f este continu n a A dac f(x) este "orict de aproape" de
f (a ) de ndat ce x este suficient de aproape de a.

Observaie. Dac a A este un punct izolat atunci orice funcie f : A Y

este continu n punctul a. Analog cu teorema 4.1.1 obinem


Teorema 4.2.1. (de caracterizare a continuitii ntr-un punct).

Fie f : A X Y i a A . Urmtoarele afirmaii sunt echivalente


a) f este continu n a; b) () > 0, () > 0 astfel nct ( ) x A, cu d1(x, a ) < rezult
d 2 (f(x), f(a) ) < ;

c) ( )(x n ) A , x n a rezult c f (x n ) f(a). Demonstraie. Analog cu demonstraia teoremei 4.1.1.

Observaie. Din teoremele 4.1.1 i 4.2.1 rezult c dac a A A atunci


f : A X Y este continu n punctul a dac i numai dac () lim f (x ) = f (a ) .
x a

Definiia 4.2.2. Funcia

f : A X Y este continu pe A dac este

continu n toate punctele din A.


Exemplu. Fie f : Rk R , liniar, adic
f (x + y ) = f (x ) + f (y ) , ( ) , R , x, y Rk .

- 72 -

Se arat c o aplicaie liniar f pe R k este de forma


f (x

)=

i= 1

c ix i,

() x = (x1, x 2,..., xk ) Rk , unde

c = (c1, c 2 ,..., c k ) Rk .

Dac a Rk , a = (a1, a2 ,.., ak ) este fixat i x Rk , x = (x1, x 2 ,..., x k ) R k este arbitrar, atunci:
k f (x ) f (a ) = c i (xi ai ) c i2 i =1 i =1
k 12 12

k (xi ai )2 i =1

= c xa .

(am considerat pe R k norma euclidian). Va rezulta c lim f (x ) = f (a ) , deci f este continu n a i cum a este arbitrar
x a

rezult c f este continu pe R k . n particular, funciile proiecie pri : Rk R , pri (x ) = xi , () x = (x 1, x 2 ,..., x k ) R k , i = 1, k , sunt continue pe R k .
Teorema 4.2.2. O funcie f : A Rk Rm , k 1, m 1, f = (f1, f2 ,..., fm ) este

continu n punctul a A dac i numai dac funciile f1, f2,,fm sunt continue n a.
Demonstraie. Rezult din definiia continuitii cu iruri i teorema de

caracterizare a convergenei unui ir n spaiul metric R m , m 1 .

Observaie. Din aceast teorem i exemplul anterior rezult c o aplicaie

liniar f : Rk Rm este continu pe R k .


Teorema 4.2.3. (de caracterizare a continuitii pe un spaiu).

Fie (X, d1 ) , (Y, d2 ) spaii metrice i f : X Y . Atunci urmtoarele afirmaii sunt echivalente
a) f este continu pe X; b) ( ) D Y , deschis f 1(D) este deschis n X; c) ( ) F Y , nchis f 1(F ) este nchis n X; d) ( ) A X
f A f (A ) .

( )

- 73 -

Demonstraie. a) d) Fie A X i y f (A ) , y = f (x ) , cu x A . Cum x A , ()(xn ) A

astfel nct x n x . Cum f este continu n x, rezult f (xn ) f (x ) = y , deci y f (A ) i astfel am artat c f (A ) f (A ) .
d) c) Fie F Y, nchis, adic F = F n Y, i fie A = f
-1

(F). Conform

ipotezei:
f A f (A ) = f f 1 (F ) F = F , de unde

( ) A f 1 (f ( A ) ) f 1 ( F ) = A ,

( )

i cum A A , rezult A = A , deci A = f 1 (F ) este nchis n X.


c) b) Fie D Y , deschis; atunci F = Y \ D este nchis.

Conform ipotezei, f 1 (F ) = f 1 (Y \ D ) este nchis n X, de unde


X \ f 1 (F ) = X \ f 1 (Y ) \ f 1 (D ) = f 1 (D ) , i cum f

(F) este nchis, rezult c

f 1(D) este deschis.


b) a) Fie a X. Vom arta c f este continu n a.

Fie D = B(f (a ) , ) Y , deschis. Conform ipotezei f-1(D) X este deschis i cum a f-1(D),

() > 0 astfel nct B (a , ) f-1(D), de unde rezult c

f (B(a, )) B(f (a ), ) , adic f este continu n a.

Fie f : A \ {a} X Y , unde aA. Dac exist lim f (x ) = L Y , atunci funcia


x a

f :A Y,

f (x ) dac x A \ {a} prelungete funcia f i este continu n punctul a . f (x ) = L dac x = a

n acest caz, spunem c f este prelungirea prin continuitate a lui f.


Exemplu.

Fie f : R \ {0} R , f (x ) =

sin x . x

- 74 -

Cum lim f (x ) = 1 , funcia f poate fi prelungit prin continuitate n punctul 0.


x 0

Prelungirea ei este funcia f : R R , f (x ) = x


sin x dac x 0 1 dac x = 0

Definiia 4.2.3. a) f este bijectiv;

O aplicaie f : (X, d1 ) (Y, d 2 ) se numete izomorfism

topologic sau homeomorfism dac:


b) funciile f i f 1 sunt continue.

n acest caz, spunem c spaiile X i Y sunt homeomorfe. O aplicaie ce satisface condiia b) se mai numete i bicontinu. S observm c dac f este homeomorfism atunci i f 1 este un homeomorfism. Dac f : X Y este un homeomorfism spunem c spaiile metrice (X, d1 ) ,

(Y, d2 ) sunt homeomorfe.


Exemplu. Funcia f : R R , f (x ) = x 5 este homeomorfism a lui R pe R. Definiia 4.2.4. O aplicaie f : (X, d1 ) (Y, d2 ) se numete izometrie dac a) f este bijectiv; b) d2 (f (x ), f (y )) = d1(x, y ) , () x, y X .

n acest caz, spunem c spaiile metrice (X, d1 ) , (Y, d2 ) sunt izometrice. S observm c dac f este izometrie de la X la Y atunci i f 1 este o izometrie de la Y la X.
Exemplu. Fie a Rn i f : R n R n , f (x ) = x + a .

Este evident faptul c f este o bijecie, iar


f (x ) f (y ) =

(x + a ) (y + a)

= x y , () x, y Rn , deci f este o izometrie.

Rezult c orice translaie este o izometrie. S observm, de asemenea, c orice izometrie este un homeomorfism. ntr-adevr, fie f : (X, d1 ) (Y, d 2 ) izometrie.

- 75 -

Vom arta c f este continu. Fie x 0 X i > 0 , arbitrar. Vom avea:


f 1(B(f (x 0 ), )) = {x X : f (x ) B(f (x 0 ), )} =

= {x X : d2 (f (x ), f (x 0 )) < } = {x X : d1( x, x 0 ) < } = B(x0, ),

deci f este continu n x0 i atunci pe X. Analog se arat c f 1 este continu.

4.3. Proprieti ale funciilor continue


Teorema 4.3.1. Fie (X, d1 ) , (Y, d2 ) spaii metrice i f : X Y , continu. Dac
A X este compact, atunci f (A ) este compact n Y.

Demonstraie. Fie (D i )iI o acoperire deschis n Y pentru f (A ) , adic


f (A ) Di , Di Y deschis ( ) i I .
iI

Atunci

A f 1(f(A)) f 1 Di = f 1(Di )
iI iI

( )

Cum f este continu, din teorema 4.2.3, f 1 (D i ) este deschis n X, () i I i cum A este compact n X, exist J I , finit astfel nct A f 1(Di ) , de unde
iJ

f(A) f ( f 1(Di)) = f(f 1(Di)) Di


iJ iJ iJ

adic (Di )iJ este o subacoperire finit pentru f (A ) . Prin urmare, f (A ) este compact n Y.

Corolarul 4.3.1. Orice funcie continu pe un compact dintr-un spaiu metric

este mrginit.
Demonstraie. Dac f : (X, d1 ) (Y, d 2 ) este continu i A X este

compact, din teorema 4.3.1 rezult c f(A) este compact i atunci este nchis i mrginit n Y.

- 76 -

Definiia 4.3.1. Fie f : (X, d) R , A X i M = sup f (x ) , m = inf f (x ) . Spunem


xA xA

c:
i) Funcia f i atinge marginea inferioar pe A dac () a A astfel nct
m = f (a ) .

ii) Funcia f i atinge marginea superioar pe A dac () b A astfel nct


M = f (b ) .

iii) Funcia i atinge marginile pe A dac i atinge marginea superioar

ct i marginea inferioar.
Teorema 4.3.2. (Weierstrass). Dac A este o submulime compact a

spaiului metric (X, d) i f : A R este continu atunci f este mrginit pe A i i atinge marginile.
Demonstraie. Din corolarul 4.3.1 avem c f (A ) este mrginit n R.

Fie M = sup f (x ) i m = inf f (x ) .


xA xA

Din definiia marginii inferioare exist un ir (xn ) A astfel nct f (x n ) m i cum A este compact, exist un subir (xk punct din A, xk a A.
n n

al lui (x n ) A , convergent la un

Cum f este continu, avem f (xk ) f (a ) i cum f (xk ) m , folosind unicitatea


n n

limitei unui ir convergent ntr-un spaiu metric, rezult f(a) = m. Folosind definiia marginii superioare se arat n mod asemntor c

() b A astfel nct f(b) = M.

Observaie. n particular, dac X = R , A = [a, b], a, b R , a < b se obine

teorema lui Weierstrass cunoscut din liceu:


Teorema 4.3.3. Dac f: [a, b] R este continu atunci f este mrginit i i

atinge marginile pe [a, b].

- 77 -

Definiia 4.3.2. Fie (X, d) spaiu metric. O submulime A X se numete

conex dac nu exist dou mulimi deschise D 1,D 2 X cu proprietile


i) ii) iii)
D1 A , D1 A ; A D1 D2 ; D1 D2 A = .

n caz contrar, spunem c A este neconex. Spunem c spaiul metric (X, d) este conex dac nu exist dou mulimi deschise D 1,D 2 X nevide i disjunte astfel nct X = D1 D2 .
Exemple. 1. A R , A = (-2, 1] [5, 6) nu este conex. 2. A R2 , A = {(x, y ) R2 : 1 < x 2 + y 2 < 3} este o mulime conex. Teorema 4.3.4. O submulime nevid A R este conex dac i numai

dac A este interval. Pentru demonstraie se poate consulta [11].

Teorema 4.3.5. Fie (X, d1 ) , (Y, d2 ) spaii metrice, A X , conex i f : A Y ,

continu. Atunci f(A) este conex n Y.


Demonstraie. Presupunem prin absurd c f (A ) nu este conex. Atunci

exist dou mulimi nevide D1, D2 Y deschise astfel nct


D1 D2 f (A ) = , D1 f (A ) , D1 f (A ) i f (A ) D 1 D 2 . ~ ~

Cum f este continu, mulimile D 1 = f 1 (D 1 ), D 2 = f 1 (D 2 ) sunt deschise n X. n plus,


~ ~ D1 , D2 , ~ ~ D1 D2 A = f 1(D1 ) f 1(D2 ) A = f 1(D1 D2 ) A = ,

~ ~ ~ ~ D1 A , D2 A , iar A D1 D2 ,ceea ce arat c A este neconex,

absurd.
Corolarul 4.3.2. (Teorema valorii intermediare).

Fie (X, d) un spaiu metric, A X , conex, f : A R , continu i a, b A astfel nct f (a ) < f (b ) . Atunci ( ) (f (a ) , f (b )) , () c A , astfel nct f (c ) = .

- 78 -

Demonstraie. Din teorema 4.3.5 rezult c f (A ) este conex n R i din

teorema 4.3.4 f (A ) este un interval n R. Cum f (a ), f (b ) f (A ) rezult (f (a ), f (b )) f (A ) i demonstraia este ncheiat.


Teorema 4.3.6. Fie f , g : (X, d) R , continue i R . Atunci a) f + g , fg, f, g sunt continue pe X; b)
f (unde g(x ) 0, ( ) x X ) este continu pe X; g

c) f este continu pe X. Demonstraie. Rezult imediat folosind caracterizarea cu iruri.

Teorema 4.3.7. Fie f : A (X, d) R , continu n a A i f (a ) 0 . Atunci

exist o vecintate V a lui a astfel nct f are semn constant pe V A .


Demonstraie. Fr a restrnge generalitatea s presupunem c f (a ) > 0 .

Fie R astfel nct f (a ) > > 0 . Atunci (f (a ) , f (a ) + ) (f (a )) , n R i cum f este continu n a, () V (a) astfel nct ( ) x V A avem
f (x ) (f (a ) , f (a ) + ) , de unde f (x ) > f (a ) > 0 , ()x V A .

Continuitate lateral
Definiia 4.3.3. Fie f : D R , D R i a Int D . i) Funcia f este continu la stnga n a dac () f (a 0 ) = f (a ) . ii) Funcia f este continu la dreapta n a dac () f (a + 0 ) = f (a ) .

Folosind teorema de caracterizare a limitei unei funcii ntr-un punct cu ajutorul limitelor laterale (teorema 4.1.7) se obine:
Teorema 4.3.8. Fie f : D R , D R i a Int D .

Atunci f este continu n a dac i numai dac i numai dac este continu la stnga i la dreapta n a.

- 79 -

4.4. Funcii uniform continue


Definiia 4.4.1. O funcie f: (X, d1 ) (Y, d2 ) se numete uniform continu pe X

dac

pentru

() > 0, () > 0

astfel

nct

() x, y X

cu

d1(x, y ) <

rezult d 2 (f (x ), f (y )) < .
Observaii. 1. n timp ce noiunea de continuitate are un caracter local, cea

de uniform continuitate are un caracter global, ea definindu-se pe o mulime.


2. Din definiie rezult c, dac exist dou iruri (xn ), (yn ) X astfel nct
d1(x n , yn ) 0 i d2 (f (x n ), f (y n )) 0 atunci f nu este uniform continu

3. Din definiie rezult c, dac f este uniform continu pe X atunci f este

continu pe X.
Exemplu. Fie funcia f : (0,1]R, f (x ) =
1 . x

Lund xn = , yn =

1 n

1 , n 1 avem x n y n 0 i f (x n ) f (y n ) = n , deci f 2n

nu este uniform continu pe (0,1]. S observm totui c f este continu pe (0,1]. Din acest exemplu rezult c o funcie continu nu este n general uniform continu.
Teorema 4.4.1. (Cantor). Fie (X, d1 ), (Y, d 2 ) spaii metrice A X , compact i
f : A Y continu. Atunci f este uniform continu pe A.

Demonstraie. Presupunem prin absurd c f nu este uniform continu, deci


() 0 > 0 asfel nct () > 0 , () x , y A cu d 1 (x , y ) < i d2 (f ( x ), f ( y )) 0 .

Lund = d1(xn,yn)<
kn n

1 , cu n 1, rezult c exist dou iruri (xn), (yn) din A cu n

1 i d2(f(xn),f(yn)) 0 . Cum A este compact () x,y A i dou subiruri n

(x ) (x )

i (yk ) (yn ) astfel nct


n n n n

x k n x , yk n y .Cum d1 xkn , ykn <


n

1 0, kn

vom avea d(x, y ) d(x, xk ) + d(xk , yk ) + d(yk , y ) 0 , pentru n , deci d(x, y ) = 0 i atunci x = y.

Cum f e continu avem f (x k ) f (x ), f (y k ) f (y ) i atunci


n n

- 80 -

0 d2 f xk n , f y k n d2 f xk n , f (x ) + d2 f yk n , f (y ) 0 , pentru n , contradicie.

( ( ) ( ))

(( )

(( )

Probleme propuse.
1. S se studieze existena limitelor: a) d)
x 2y ; ( x, y ) (2,3 ) x 2 + 3 y lim

b)

sin xy ; ( x, y ) (0,2 ) x lim

c) lim f)

( x, y ) (0,0 )

x cos

1 ; x + y2
2

x3 + y3 ; ( x, y ) (0,0 ) x 2 + y 2 lim

e) lim (1 + sin x )e x ;
x

x 2y ; ( x, y ) (0,0 ) x + y lim

x g) lim ; ( x, y ) (0,0 ) y 2

x2y h) lim ; i) ( x, y ) (0,0 ) x 2 + y 2

x lim 1 + . ( x, y ) (2,+ ) y

2. S se studieze continuitatea funciilor: a) f : R R , f (x, y ) =


xQ x, ; 2 x , x R \ Q

sin x3 + y 3 , (x, y ) (0,0 ) b) f : R R , f (x, y ) = x 2 + y 2 ; 0, (x, y ) = (0,0)


2

c) f : R2 R2 , f (x ) = e x sin y,

x + a, x < 0 , unde a R . x e , x0

3. Fie f : D R R , monoton. S se arate c f are limite laterale n toate

punctele de acumulare.
4.

Fie

f , g : (X, d1 ) (Y, d 2 ) ,

continue.

se

arate

mulimea

A = {x X : f (x ) = g(x ) } este nchis n X.

5. Fie (X, d) spaiu metric, A X, A i f : X R ,


f (x ) = d (x, A ) = inf d(x, a ) . S se arate c f este continu pe X.
aA

6.

Fie

f , g : (X, d) R ,

continue.

se

arate

mulimea

A = {x X : f (x ) < g(x ) } este deschis n X.

- 81 -

7. S se determine funciile continue f : R R cu proprietatea:

f(x + y) = f(x) + f(y), () x, y R.


8. Fie f : I R,I R , interval, f monoton i
Df = { x I : f discontinu n x} . S se arate c D f este cel mult numrabil.

9. Fie f : R R continu i mrginit. S se arate c f are cel puin un

punct fix, adic () x 0 R astfel nct f (x 0 ) = x 0 .


10. Fie spaiul metric (L , d) , unde:
L = { (xn ) R : (xn ) mrginit
nN *

}, i

d (x, y ) = sup x n y n , () x, y L , x = (x n ) , y = (yn ) .

S se arate c L nu este compact.


11. S se arate c funcia f : R R , f (x ) = sin x este uniform continu pe R. 12. Fie (X, d) un spaiu metric i x 0 X . S se arate c funcia f : X R ,
f (x ) = d(x, x 0 ) este uniform continu pe X.

13. S se studieze uniform continuitatea funciilor: a) f : (0, ) R , f (x ) =


1 1+ x2

b) f : (1, 2 ] R , f (x ) =

1 ; x 1

c) f : R R , f (x ) = sin x 2 ; d) f : [0, ) R , f (x ) = x ; e) f : (0, ) x (0, ) R , f (x, y ) =


x +1 . y

14. Fie a 0 i funcia f : (a, ) R , f (x ) = ln x .

S se arate c f este uniform continu pe (a, ) dac i numai dac a > 0.

- 82 -

15. Fie f : R R continu i periodic.

Atunci f este uniform continu.


16. Fie f , g : (X, d) R uniform continue. Atunci : a) f + g este uniform continu. b) dac f i g sunt mrginite fg este uniform continu.

- 83 -

CAPITOLUL 5
DERIVABILITATEA I DIFERENIALITATEA FUNCIILOR REALE DE VARIABIL REAL

5.1. Proprieti de baz ale derivatei.


Definiia 5.1.1. Fie f : D R R , x 0 D D .

Funcia f are derivat n x 0 dac exist n R urmtoarea limit:


df f (x ) f (x 0 ) (x0 ) ). = f (x 0 ) (sau x x0 x x0 dx
lim

Dac, n plus derivata f (x 0 ) este finit, spunem c funcia f este derivabil n x 0 .


Propoziia 5.1.1. Dac f : D R R este derivabil n x 0 D D atunci f

este continu n x 0 .
Demonstraie. Din ipotez avem c () lim
f (x ) f (x 0 ) = f (x 0 ) R . x x0 x x0

Pentru ( )x D, x x 0 avem
f (x ) = f (x 0 ) +
x x0

f (x ) f (x 0 ) (x x0 ) , de unde rezult prin trecere la limit: x x0

lim f (x ) = f (x 0 ) , deci f este continu n x 0 .

Observaie. Reciproca acestei afirmaii nu este n general adevrat. n

acest sens, fie f : R R , f (x ) = x . Evident, f este continu pe R dar nu este derivabil n x 0 =0.

- 84 -

Definiia 5.1.2. Funcia f : D R R este derivabil pe A D dac este

derivabil n orice punct din A . n acest caz se poate defini funcia f : A R , x f (x ) , numit derivata funciei f .
Definiia 5.1.3.

Fie D R i x 0 D un punct de acumulare pentru

D1 = D ( , x 0 ) (respectiv pentru D2 = D (x 0 , ) ).

Funcia f are derivat la stnga (respectiv la dreapta) n x 0 , dac

()

f (x ) f (x 0 ) f (x ) f (x 0 ) = fs (x 0 ) R , (respectiv () lim = fd (x 0 ) R ). x x0 x x0 x x0 x x0
lim
x < x0

x > x0

Numrul fs (x 0 ) R (respectiv fd (x 0 ) R ) se numete derivata la stnga

(respectiv la dreapta) a lui f n x 0 .


Dac n plus fs (x 0 ) R (respectiv fd (x 0 ) R ), spunem c f este derivabil la

stnga (respectiv la dreapta) n x 0 .


Observaie. Folosind teorema de caracterizare a limitei unei funcii ntr-un

punct cu ajutorul limitelor laterale rezult c o funcie f : D R R este derivabil n x 0 D (care este punct de acumulare att la stnga ct i la dreapta lui D) dac i numai dac f este derivabil la stnga i la dreapta n x 0 , iar
fs (x 0 ) = fd (x 0 ) = f ' (x 0 ) .

Reamintim din liceu proprietile de baz ale funciilor derivabile:


Teorema 5.1.1. Fie I, J R , intervale, f : I J , g : J R . Dac f este

derivabil n x 0 I i g este derivabil n y 0 = f ( x 0 ) J atunci g o f : I R este derivabil n x 0 i (g o f )' ( x 0 ) = g' ( f ( x 0 ))f ' ( x 0 ) . Dac f este derivabil pe I i g este derivabil pe J atunci g o f este derivabil pe I i (g o f )' = (g'o f )f ' .

- 85 -

Teorema 5.1.2. Fie I, J R , intervale i f : I J continu i bijectiv. Dac f

este derivabil n x 0 I i f (x 0 ) 0 , atunci funcia invers f 1 este derivabil n


1 y 0 = f (x 0 ) i f 1 (y 0 ) = . f (x 0 )

( )

Definiia 5.1.4. Fie f : D R R . Un punct x 0 D se numete punct de

minim (respectiv de maxim) local pentru f dac


f (x ) f (x 0 ) (respectiv ), () x V D .

() V (x0) astfel nct

Dac aceste inegaliti au loc () x D , atunci x 0 se numete punct de minim (respectiv de maxim) global. Un punct x 0 D se numete punct de extrem local dac este punct de minim local sau de maxim local.
Teorema 5.1.3. (Fermat).

Fie f : I R R , I interval. Dac x 0 I este punct de extrem local pentru f i f este derivabil n x 0 atunci f (x 0 ) = 0 .
Definiia 5.1.5. Fie f : I R R , I interval. Un punct x 0 I n care f este
o

derivabil i f (x 0 ) = 0 se numete punct critic (sau staionar) pentru f .


Observaii. 1. Dac x 0 I , atunci concluzia teoremei lui Fermat nu este ntotdeauna
o

adevrat. n acest sens, fie funcia f : [0,1] R, f (x ) = x . Atunci x 0 = 0 este punct de minim pentru f i f (0 ) 0 .
2. Reciproca teoremei lui Fermat nu este n general adevrat. n acest

sens fie funcia f : R R , f (x ) = x 3 . Atunci f (0 ) = 0 dar x 0 = 0 nu este un punct de extrem local.


Teorema 5.1.4. (Teorema lui Rolle).

Fie funcia f : [a, b] R , unde a, b R, a < b, f

este continu pe

[a,b] ,

derivabil pe (a,b), f (a ) = f (b ) . Atunci () c (a, b ) astfel nct f (c ) = 0 .

- 86 -

Teorema 5.1.5. (Teorema lui Cauchy).

Fie f : [a, b] R dou funcii continue pe [a,b] , derivabile pe (a,b) i g(x ) 0 ,


()x (a,b). Atunci ()c (a,b ) astfel nct f (b ) f (a ) = f (c ) . g(b ) g(a ) g(c )

Teorema 5.1.6. (Teorema lui Lagrange).

Fie f : [a,b] R continu pe [a,b] , derivabil pe (a, b ) . Atunci () c (a, b ) astfel nct f (b ) f (a ) = (b a)f ' (c ) . Din teorema lui Lagrange se obin cu uurin urmtoarele dou consecine:
Propoziia 5.1.1. Fie f : I R R , I interval, f derivabil pe I .

Atunci:
a) Dac f 0 pe I rezult c f este cresctoare pe I ; b) Dac f 0 pe I rezult c f este descresctoare pe I ; c) Dac f = 0 pe I rezult c f este constant pe I . Propoziia 5.1.2. Fie f : I R R , I interval, f continu pe I i x 0 I astfel

nct f este derivabil pe I \ {x 0 } i () lim f (x ) i este finit.


x x0

Atunci f este derivabil n x 0 i f ' ( x 0 ) = lim f (x ) .


xx0

Teorema 5.1.7. (Regula lui L'Hospital).

Fie f , g : (a, b ) R unde a < b + . Presupunem c


i) f , g sunt derivabile pe (a, b ) i g 0 pe (a, b ) ; ii) () lim
f (x ) = L R ; x a g(x )
x >a

iii) lim f (x ) = lim g(x ) = 0 (sau ).


x a x >a x a x >a

Atunci () lim x a
x >a

f (x ) =L. g(x )

- 87 -

5.2. Difereniala unei funcii


Fie I R interval deschis, f : I R i x 0 I .
Definiia 5.2.1. Funcia f este difereniabil n x 0 dac ()A R astfel

nct
f (x ) f (x 0 ) A (x x 0 ) =0 x x0 x x0 lim

(5.1)

f (x ) f (x 0 ) A (x x 0 ) dac x x 0 Observaie. Lund (x ) = . x x0 0 dac x = x 0

rezult c (5.1) se scrie echivalent:


f (x ) = f (x 0 ) + A (x x 0 ) + (x )(x x 0 ), ( )x I ,

(5.2)

unde A R , : I R , lim (x ) = (x 0 ) = 0 .
x x0

Teorema

5.2.1.

Fie

IR

interval

deschis.

Funcia

f :I R

este

difereniabil n x 0 I dac i numai dac este derivabil n x 0 .


Demonstraie. Presupunem mai nti c f este difereniabil n x 0 .

Conform definiiei () A R , : I R , continu n x 0 , (x 0 ) = 0 astfel nct


f (x ) = f (x 0 ) + A (x x 0 ) + (x )(x x 0 ), ( )x I , de unde
f (x ) f ( x 0 ) = A + ( x ), ()x I, x x 0 . x x0

Cum lim ( x ) = 0 () lim


x x0

x x0

f (x ) f (x 0 ) = A , deci f este derivabil n x0. x x0

Reciproc, fie f derivabil n x 0 i fie : I R ,


f (x ) f (x 0 ) f ' ( x 0 ) dac x x 0 (x ) = x x 0 . dac x = x 0 0

- 88 -

Evident avem lim (x ) = (x 0 ) = 0 i din definiia lui avem


x x0

f (x ) = f (x 0 ) + f' (x 0 )(x x 0 ) + (x )(x x 0 ), ( ) x I ,

(5.3)

deci f este difereniabil n x 0 , iar A = f (x 0 ) .

Observaie. Teorema 5.2.1 exprim faptul c noiunile de difereniabilitate i

derivabilitate sunt echivalente.


Definiie. Fie f : I R R , I interval deschis, f derivabil n x 0 I .

Funcia T : R R definit prin T(h ) = f (x 0 ) h , () h R se numete difereniala funciei f n x 0 i se noteaz cu df (x 0 ) , adic df (x 0 ) (h ) = f (x 0 )h , () h R .


Observaie. Dac f este difereniabil n x 0 , din (5.3) rezult c, pentru x

ntr-o vecintate V a lui x 0 , avem aproximarea


f (x ) f (x 0 ) f (x 0 )(x x 0 ) ,

(5.4)

adic creterea funciei ntr-o vecintate a punctului x 0 , f se poate aproxima printr-o cretere liniar f (x 0 )x . Notnd x x 0 = h , relaia (5.4) se mai scrie:
f (x ) f (x 0 ) f (x 0 ) h = df (x 0 ) h .

n cazul particular, n care f (x ) = x avem f (x 0 ) = 1 i atunci dx(x 0 ) (h) = h ,

()h R .
Notnd cu dx difereniala funciei identice, (deci dx : R R , dx (h ) = h ,

()h R ), obinem

df (x 0 ) = f (x 0 )dx .

Dac f este derivabil pe I atunci vom avea df(x)= f ' (x)dx, ( ) x I .

5.3. Derivate i difereniale de ordin superior


Fie I R un interval deschis, f :I R derivabil pe I i f ' : I R derivata sa.

- 89 -

Definiia 5.3.1. Funcia f este de dou ori derivabil n x 0 dac funcia f

este derivabil n x 0 . n acest caz, derivata lui f n x 0 se mai numete derivata a doua a lui f n x 0 i se noteaz cu f (x 0 ) (sau
d2 f (x 0 ) ). dx 2

Dac f este derivabil pe I atunci derivata lui f se numete derivata a doua (sau de ordinul doi) a lui f i se noteaz cu f . Prin recuren se definesc derivatele de ordin superior:
Definiia 5.3.2. Fie f : I R R , I interval deschis.

Funcia f este de n (n 2) ori derivabil n x 0 dac f este de (n 1) ori derivabil pe o vecintate V a lui x 0 i dac derivata de ordin (n 1) , notat prin
f (n1) este derivabil n x 0 . n acest caz derivata lui f
(n-1)

n x0 se mai numete

derivata de ordinul n a lui f n x0 i se noteaz cu f

(n)

dn f (x0) ( sau n ( x 0 ) ). dx

Funcia f este de n ori derivabil pe I dac este de n ori derivabil n toate punctele din I .
Definiia 5.3.3. Fie f : I R R , I interval deschis.

Funcia f este de clas Cn , (n 1) pe I i scriem f Cn (I) dac f este de n ori derivabil pe I i derivata de ordin n este continu pe I . Vom nota C0 (I) = { f : I R , f continu};
C (I) = { f : I R , f indefinit derivabil pe I , adic este derivabil

de n ori pe I, ( )n N* } .
Teorema 5.3.1. Fie I R interval deschis, n N* , f , g C n ( I ) , , R .

Atunci f + g, fg C(n ) ( I) i
i) (f + g)(n ) = f (n ) + f (n ) ;
( ) ii) (fg) n = Ck f (n k )g(k ) . n
k =0 n

- 90 -

Formula lui Taylor

Fie f : I R R , I interval deschis, x0 I, n N*, iar funcia f de n ori derivabil pe I .

Ne propunem s aproximm valorile funciei f n vecintatea lui x 0 printr-un polinom de grad n . Cutm un polinom Tn R[X] de grad n care s aproximeze valorile funciei
f ntr-o vecintate a lui x 0 i s verifice n plus condiiile
Tn (x 0 ) = f (x 0 ) , Tn (x 0 ) = f (x 0 ) ,..., Tn(n ) (x 0 ) = f (n ) (x 0 ) .

(5.5)

Considerm Tn descompus dup puterile lui (x x 0 )


Tn (x ) = a0 + a1(x x 0 ) + ... + an (x x 0 )
n

Folosind relaiile (5.5) vom avea a0 = f(x0) , a1 = (x0) , , an = Astfel obinem


1 (n) f (x0). n!

(x x 0 ) f (n ) (x ) , unde x I . x x0 Tn (x ) = f (x 0 ) + f (x 0 ) + ... + 0 1 ! n!
n

Polinomul Tn se numete polinom Taylor asociat funciei f n x 0 . Fie Rn : I R , Rn (x ) = f (x ) Tn (x ) . Funcia Rn se numete restul Taylor de ordin n asociat funciei f n x 0 . Obinem f (x ) = Tn (x ) + Rn ( x ) , () x I , adic
f (x ) = f (x 0 ) +

(x x 0 ) f (n ) (x ) + R (x ) , () x I , x x0 f (x 0 ) + ... + 0 n 1 ! n!
n

(5.6)

numit formula lui Taylor cu rest de ordin n asociat funciei f n x 0 .


Observaie. Cum
lim Rn (x ) = 0 rezult c pentru x ntr-o vecintate V a

x x0

lui x 0 avem aproximarea: f (x ) f (x 0 ) + Se poate arta c lim


Rn ( x ) x x0
n

(x x 0 ) f (n ) (x ) . x x0 f (x 0 ) + ... + 0 1 ! n!
n

x x0

= 0.

- 91 -

Teorema 5.3.2. (Formula lui Taylor cu rest Lagrange).

Fie I R interval deschis, x 0 I i f : I R de (n + 1) ori derivabil pe I . Atunci ()x I, x x 0 exist ntre x 0 i x (deci = (1 )x 0 + x , cu (0,1) ), astfel nct

(x x0 ) f (n ) (x ) + (x x0 ) f (n+1) ( ) . x x0 f (x ) = f (x 0 ) + f (x 0 ) + ... + 0 1 ! n! (n + 1)!


n n +1

Demonstraie. Cutm n (5.6) pe Rn de forma Rn (x ) = (x x 0 ) pk , unde


p N * , k = k (x, x 0 ) .

Considerm funcia : I R

(x t ) f (n ) (t ) + (x - t) pk. xt (t ) = f (t ) + f (t ) + ... + 1 ! n!
n

Evident este derivabil pe I , (x 0 ) = f (x ), (x ) = f (x ) i din teorema lui Rolle

() ntre

x0 i x astfel nct ( ) = 0 . Dar


(t ) = f (t ) +

(x t ) f (n ) (t ) + f (t ) x t + f (t ) ... (n 1)! 1 ! 1 !
n 1

(x t )n f (n+1) (t ) p(x t )p 1k =
n! n!

(x t )n f (n+1)( t ) p(x t )p 1k . =
Din ( ) = 0 rezult

(x )n f (n +1)() = p(x ) p 1k .
n!

Lund p = n + 1 obinem k =
R n (x ) =

f (n +1) ( ) i atunci (n + 1)!

(x x 0 )n +1 f (n+1) ( ) , () x I . (n + 1)!

i demonstraia este ncheiat.

Observaie. Dac x 0 = 0 I , din teorema 5.3.2 rezult c () x I, x 0 ,

()

ntre 0 i x (deci = x , cu (0,1) ), astfel nct

- 92 -

f (x ) = f (0 ) +

x xn (n ) xn +1 (n +1) ( ) , f (0 ) + ... + f (0 ) + f (n + 1)! n! 1 !

numit formula Mac-Laurin cu rest de ordin n sub forma Lagrange.

Aplicaii. 1. Fie f : R R , f (x ) = e x . Atunci f este indefinit derivabil pe R i


f (n ) (x ) = e x , () n N , ( ) x R .

Rezult f (n ) (0 ) = 1 , ( ) n N . S scriem formula Mac-Laurin cu rest de ordin n sub forma Lagrange. Pentru ( ) x R , x 0 , () (0,1) astfel nct
ex = 1+ x x2 xn x n +1 x e + + ... + + 1 2! ! n! (n + 1)!

2. Fie f : R R , f (x ) = cos x . Atunci f este indefinit derivabil pe R i


n f ( n ) (x ) = cos x + , ( ) n N , () x R . 2

Din formula lui Mac-Laurin cu rest de ordin n sub forma Lagrange pentru

() x R ,

x 0 , () (0,1) astfel nct

cos x = 1

x2 x4 x6 xn n x n +1 + + ... + + cos cos + (n + 1) . 2! 4! 6! n! 2 (n + 1)! 2

Pentru n = 2k obinem:
cos x = 1 x2 x4 x6 x 2k x 2k + 1 + + ... + ( 1)k + cos + (2k + 1) 2! 4! 6! (2k )! (2k + 1)! 2

i analog pentru n impar.


Extreme locale pentru funcii derivabile Teorema 5.3.3. (Condiia suficient de extrem local).

Fie I R interval deschis, f : I R de clas C n , (n 1) pe I , x0 R un punct critic pentru f astfel nct f (x 0 ) = f (x 0 ) = ... = f (n 1) (x 0 ) = 0 , f (n ) (x 0 ) 0 .

- 93 -

Atunci
a) Dac n este par, rezult c x 0 este punct de extrem local i anume:
f (n ) (x 0 ) > 0 x 0 punct de minim local f (n ) (x 0 ) < 0 x 0 punct de maxim local

b) Dac n este impar, rezult c x 0 nu este punct de extrem local. Demonstraie. a) Presupunem n par i f (n ) (x 0 ) > 0 . Cum f (n ) este continu, ()V (x 0 ), V I

astfel nct f (n ) (x ) > 0 , ( ) x V . Din formula lui Taylor cu rest de ordinul (n 1) sub forma Lagrange, pentru
x V, x x 0 , exist ntre x i x 0 astfel nct: f (x ) = f (x 0 ) +

(x x 0 ) f (n 1) (x ) + (x x0 ) f (n) () , x x0 f (x 0 ) + ... + 0 1 ! (n 1)! n!


n 1 n

de unde
f (x ) f (x 0 ) =

(x x 0 )n
n!

f (n ) ( ) , () x V .

Cum n este par avem

(x x0 )n
n!

0 , () x V i cum este ntre x i x0

rezult V i atunci f (n ) () > 0 , de unde f(x) f(x0) 0, () x V , adic x0 este punct de minim local pentru f. Cazul f (n ) (x 0 ) < 0 se trateaz analog.
b) Dac n este impar atunci diferena f (x ) f (x 0 ) nu are semn constant pe

nici o vecintate V a lui x 0 , deci x 0 nu este punct de extrem local.


Exemplu. Fie funcia f : R R , f (x ) = x 6 2x 3 + 5 . Vom avea:
f (x ) = 6 x 5 6 x 2 ;

f (x ) = 0 6 x 2 x 3 1 = 0 x {0,1} , deci x = 0 i x = 1 sunt puncte critice.

Caz 1. Fie x 0 = 0 .Vom avea:


f (x ) = 30 x 4 12x f (0 ) = 0 ;

- 94 -

f (x ) = 120 x 3 12 f (0 ) = 12 0 , deci n = 3 , impar i x 0 = 0 nu este punct

de extrem local.
Caz 2. Fie x 0 = 1 .V0m avea:
f (1) = 30 12 = 18 > 0 , n = 2 , par.

Rezult c x 0 = 1 este punct de minim local.

Definiia 5.3.4. Fie I R , interval deschis i x 0 I i f : I R .

Funcia f este de dou ori difereniabil n x 0 (respectiv pe I) dac f este derivabil pe I i derivata sa f este difereniabil n x 0 (respectiv pe I). Difereniala de ordinul doi a funciei f n x 0 se noteaz cu d 2 f (x 0 ) i este definit prin
d 2 f (x 0 ) = f (x 0 )(dx ) = f (x 0 )dx 2 .
2

Prin recuren obinem:


Definiia 5.3.5. Fie I R , interval deschis, x 0 R i f : I R . Funcia f este

de n (n 2) ori difereniabil n x 0 dac este de (n 1) ori derivabil pe I i derivata f (n1) este difereniabil n x 0 . Difereniala de ordinul n se noteaz cu d n f (x 0 ) i este definit astfel:
d n f (x 0 ) = f (n ) (x 0 )(dx ) = f (n ) (x 0 )dx n .
n

Probleme propuse 1. S se studieze derivabilitatea funciei f : R R ,


1 n x sin , x 0 f (x ) = , unde n N x 0 , x=0

2. S se arate c punctul c din teorema lui Lagrange aplicat funciei


f : [a, b] R , 0 < a < b , f (x ) = ln x verific inegalitile

ab < c <

1 (a + b ) . 2

- 95 -

3. S se arate c, dac a, b > 0 atunci ab + ba > 1. 4. Fie f : R R , f (x ) = e x cos x . S se calculeze f (n ) (x ) , unde n N * i s se

aproximeze funcia f printr-un polinom de grad trei n vecintatea originii.


5. S se arate c polinomul P(x ) = 1 +
x xn + ... + nu poate avea rdcini 1 ! n!

multiple.

6. S se arate c ecuaia xn nx + 1 = 0 , n 3 are dou rdcini pozitive


n , n cu proprietile n 0 , n 1.

7. S se scrie formula Mac-Laurin cu rest de ordin n sub forma Lagrange

pentru funciile:
a) f : R R , f (x ) = sin x; b) f : ( 1, ) R , f (x ) = ln(1 + x ) ; c) f : ( 1, ) R , f (x ) = 1 + x . 8. S se studieze extremele locale ale funciilor: a) f : R R , f (x ) = x 2arctg x; b) f : (0, ) R , f (x ) = ecos
2

sin x .

- 96 -

CAPITOLUL 6
DERIVABILITATEA I DIFERENIABILITATEA FUNCIILOR DE MAI MULTE VARIABILE

6.1. Derivata dup o direcie. Derivate pariale de ordinul nti.


Fie A Rn, n 2, deschis, a A i f : A R, f = f(x1, x2, , xn). Pornind de la definiia derivatei din cazul funciilor reale de variabil reala suntem tentai s definim derivata funciei f n punctul a pornind de la raportul
f ( x ) f (a ) , pentru xa

x a, ns acest raport nu are sens ntruct x-a este un vector i nu este definit mprirea unui scalar la un vector. Dac am defini derivata funciei f n a prin lim
f ( x ) f (a ) , raportul dat are xa x a

sens ns nu vom obine o definiie satisfctoare deoarece considernd funcia cu valorile f(x) = f(x1, x2,, xn) = x1, pentru x = (x1, x2,, xn) A, aceasta nu ar avea derivat n origine n sensul menionat. Vom defini mai nti derivata dup o direcie. Fie r > 0 astfel nct B(a, r) A i s Rn un versor, deci s = (s1, s2,, sn), IIsII =
2 2 s1 + s2 + ...sn = 1. 2

Fie funcia g : (-r, r) R, g(t) = f(a + ts). S observm c, pentru t (-r, r) avem a + ts B (a, r) A, deoarece IIa + ts aII = IItsII = ItI IIsII = ItI < r i deci funcia g este bine definit. Definiia 6.1.1. Funcia f este derivabila n punctul a dup versorul s dac funcia g este derivabil n t = 0 iar g(0) se numete derivata lui f n punctul a i se noteaz prin
df (a ) . ds

- 97 -

Observaie. Conform definiiei avem:


g( t ) g(0) f (a + ts) f (a) df . (a) = g (0) = lim = lim t 0 t 0 t t ds

Fie B = {e1, e2,..., en } baza canonic din Rn, e1 = (1, 0, 0,, 0), e2 = (0, 1, 0,, 0), , en = (0, 0, 0,, 0,1). Definiia 6.1.2. Funcia f este derivabil parial n punctul a n raport cu variabila xi, i 1, n dac f este derivabil n punctul a dup versorul s = ei. Numrul
df (a) se numete derivata parial a funciei f n punctul a n dei

raport cu variabila xi i se noteaz prin Observm, deci,

f ' (a) sau fxi (a) . xi

df f f (a + tei ) f (a) (a ) = = (a) = lim t 0 xi t dei


= lim

f (a1, a2 ,..., ai 1, ai + t, ai +1,..., an ) f (a1, a 2 ,..., an ) , 1 i n . t 0 t

Notnd cu xi = ai + t, pentru 1 i n , obinem:


f f (a1, a 2 ,..., ai 1, xi, ai +1,..., an ) f (a1, a2 ,..., an ) (a) = lim . xi ai xi x i ai

Caz particular. n = 2, f : A R2 R, f = f(x,y), a = (x0, y0) A.


f ( x, y 0 ) f ( x 0 , y 0 ) f ( x 0 , y 0 ) = lim ; x x0 x x0 x f ( x0, y) f ( x0, y0 ) f . ( x 0 , y 0 ) = lim y y0 y y y0

Definiia 6.1.3. Funcia f este derivabil parial n punctul a dac este

derivabil parial n a n raport cu toate variabilele x1, x2, x3,, xn. n acest caz se poate defini
f f f gradf (a) = x (a), x (a),..., x (a) , numit i gradientul funciei f n punctul a. 2 n 1

- 98 -

Definiia 6.1.4. Funcia f este derivabil parial pe A dac este derivabil

parial n toate punctele din A. n acest caz se pot defini funciile


f f : A R , i 1, n , x ( x ), ()x A, numite i derivatele pariale de ordinul x i x i

nti ale funciei f. Funcia f este de clas C1 pe A i scriem f C1(A), dac f admite derivate pariale de ordinul nti pe A i acestea sunt continue pe A. n capitolul 5 am vzut c, dac f este derivabil ntr-un punct, atunci f este continu n acel punct. n cazul funciilor de mai multe variabile, rezultatul nu mai este adevrat. Dac o funcie este derivabil ntr-un punct dup un versor s, nu rezult neaprat c f este continu n acest punct. n acest sens considerm funcia:
x5 , daca ( x, y ) (0,0) f : R 2 R, f ( x, y ) = ( y x 2 )2 + x 8 0, daca ( x, y ) = (0,0)

Atunci funcia f este derivabil n origine dup orice versor s R 2 , dar f nu este continu n origine.
Observaie. Dac gi(xi) = f(a1, a2, , ai-1, xi, ai+1,, an), atunci
f g (x ) gi (ai ) (a) = lim i i = g'i (ai ), i 1, n . x i a i x i x i ai

De aici rezult i metoda practic de calcul al derivatelor pariale i anume o derivat parial n raport cu una din variabile se obine derivnd funcia f n raport cu aceea variabil conform regulilor de derivare de la funcia real, de variabil real, celelalte variabile considerndu-se constante.
Exemplu. Fie funcia f : D R 2 R , f(x,y) = x3y+ln(x2+y2).

Atunci
f 2x ( x, y ) = 3 x 2 y + 2 , ()( x, y ) D, x x + y2 f 2y ( x, y ) = x 3 + 2 , ()( x, y ) D. y x + y2

- 99 -

Fie f : A Rn Rm , f = ( f1, f2 ,..., fm ) , unde n,m 2 , A este deschis i a A .


Definiia 6.1.5. Funcia f este derivabil parial n punctul a n raport cu

variabila xi, i 1,n dac toate funciile f1, f2, , fm, sunt derivabile parial n punctul a in raport cu variabila xi. In acest caz
f f f f (a) = 1 (a), 2 (a),..., m (a) , i 1 n. , x x i x i x i i

Definiia 6.1.6. Funcia f este derivabil parial n punctul a dac este

derivabil parial n punctul a n raport cu toate variabilele x1, x2,, xn. n acest caz se poate defini matricea notat
f1 f f1 (a) (a) ... 1 (a) x 2 xn x1 f f2 f 2 (a) (a) ... 2 (a) x 2 xn Jf (a)(sauf ' (a)) = x1 Mmxn (R) , .................... ................ fm fm fm x (a) x (a) ... x (a) 2 n 1

numit matricea jacobian a lui f n punctul a. Dac m = n atunci Jf (a) Mn (R) , iar d = detJf(a), se numete determinantul funcional al funciilor f1, f2, , fn, n raport cu variabilele x1, x2,, xn n punctul a i se noteaz: detJf(a) =
D( f1.f2 ,..., fn ) D( f ) (a)(sau (a)) . D( x1, x 2 ,..., xn ) D( x )

6.2. Difereniabilitatea funciilor reale de mai multe variabile

Fie A Rn , deschis, a A i f : A R , f = f(x1, x2,, xn).


Definiia 6.2.1. Funcia f este difereniabil n punctul a dac exist o

aplicaie liniar T:Rn R astfel nct

- 100 f ( x ) f (a) T( x a) =0 x a x a lim

(6.1)

Funcia f este difereniabil pe A dac este difereniabil n orice punct din A.


f ( x ) f (a) T( x a) , daca x a x a Dac notm cu ( x ) = , atunci (6.1) se scrie 0, daca x = a

echivalent astfel: f(x) = f(a)+T(x-a)+ ( x ) x a ,()x A , unde : A R este continu n a i (a) = 0.


Propoziia 6.2.1. Dac f este difereniabil n a atunci aplicaia liniar T

(6.2)

este unic.
Demonstraie.

Presupunem c exist T1, T2 : R n R , liniare, 1, 2

continue n a, 1(a) = 2 (a) = 0 , astfel nct f(x) = f(a)+T1(x-a)+ 1( x ) x a , ()x A ; f(x) = f(a)+T2(x-a)+ 2 ( x ) x a , ()x A . Notnd T =T1-T2, = 2 1 i scznd cele dou relaii obinem T(x-a) = ( x ) x a , ()x A . Fie h Rn , fixat i t > 0 suficient de mic astfel nct a+th A. Lund x = a+th obinem T(th) = (a + th) th , de unde tT(h) =t (a + th) h , adic T(h) = (a + th) h . Pentru t 0 obinem T(h) = 0 i cum h Rn este arbitrar luat rezult T = 0, deci T1 = T2.

Definiia 6.2.2. Aplicaia liniar T se numete difereniala funciei f n

punctul a i noteaz T = df(a).

- 101 -

Observaie. Dac f : Rn R este liniar atunci f este difereniabil n orice

punct a Rn i df(a) = f. ntr-adevr, lim


f ( x ) f (a ) f ( x a ) f ( x ) f ( a ) f ( x ) + f (a ) = lim = 0. x a x a xa x a

n particular funciile proiecie pr i: Rn R , pri(x1, x2,, xn) = xi, 1 i n sunt difereniabile n ()a Rn i dpri(a) = pri, i = 1,n .
Teorema 6.2.1. Fie f : A Rn R , difereniabil n punctul a A. Atunci: a) f este continu n a; b) f este derivabil n a dup orice versor s Rn i
df (a) = df (a)(s) . ds

n particular f este derivabil parial n a i

f (a) = df (a)(ei ),()i = 1, n . xi

Demonstraie. a) Cum f este difereniabil n punctul a, conform

definiiei (vezi 6.2), exist T : Rn R , liniar, T = df(a) i : A R , continu n a,


(a) = 0 astfel nct

f(x) = f(a)+T(x-a)+ ( x ) x a , () x A
x a

(6.3)

Cum T este liniar, este continu i atunci lim f ( x ) = f (a) , adic f este continu n a.
b) Dac s Rn este un versor, din (6.3) avem
f (a) + T(a + ts a) + (a + ts) a + ts a f (a) df f (a + ts) f (a) (a) = lim = lim = t 0 t 0 ds t t T( ts) + (a + ts) ts tT(s) + t (a + ts) t = lim = lim = lim (T(s) + (a + ts)) = T(s) = df (a)(s) t 0 t 0 t 0 t t t

Observaie. Cum lim ( x ) = 0 din (6.3) rezult c, pentru x ntr-o vecintate


x a

V a lui a avem aproximarea f(x)-f(a) T(x-a) = df(a)(x-a), deci creterea funciei f ntr-o vecintate a punctului a se poate aproxima printr-o cretere liniar.

- 102 -

Observaie. Fie f : A Rn R , difereniabil n a A . Funciile proiecie

sunt difereniabile n a i dpri(a) = pri, ()i = 1,n . Notm cu dxi difereniala funciei pri n a, dxi = dpri(a), i= 1, n . Pentru ()h Rn , h = (h1, h2,, hn) vom avea:
n f n n df (a)(h) = df (a) hiei = hidf (a)(ei ) = hi (a ) = xi i =1 i =1 i =1 n n n f f f (a)pri (h) = (a)dpri (a)(h) = (a)dx i (h), = i =1 x i i =1 x i i =1 x i

de unde df (a) = a lui f n a.

f (a)dx i , care reprezint expresia diferenialei de ordinul nti x i i =1

Deci df(a): Rn R este o aplicaie liniar i df(a)(h) =


f (a)hi, () h = (h1, h2 ,..., hn ) Rn . i =1 x i
n

Din observaia anterioar rezult c pe o vecintate V a lui a avem aproximarea


f ( x ) f (a ) + f (a)( x i ai ) . x i i =1
n

Caz particular. n =2, f : A R 2 R, f = f(x,y), a = (x0,y0) A ,

df(x0, y0) =

f f ( x 0 , y 0 )dx + ( x 0 , y 0 )dy . x y

Teorema 6.2.2. (Criteriu de difereniabilitate).

Fie A Rn , deschis, a A i f : A R . Dac f este derivabil parial ntr-o vecintate V a punctului a iar derivatele pariale sunt continue n a atunci f este difereniabil n a.
Demonstraie. Fie a = (a1, a2, an) A , r > 0 astfel nct B(a,r) A i f

este derivabil parial pe B(a,r). Pentru x B (a,r), vom avea : f(x) - f(a) = f(x1, x2,, xn) - f(a1, a2, an) = [f(x1, x2,, xn) - f(a1, x2,, xn)] + + [f(a1, x2,, xn) - f(a1, a2, xn)] ++ [f(a1, a2, an-1, xn) - f(a1, a2, an)].

- 103 -

Cum f este derivabil parial n raport cu x1 pe B(a,r), din teorema lui Lagrange aplicat funciei g1(t) = f(t, x2, x3,, xn) pe intervalul nchis determinat de punctele x1 i a1 rezult c exist 1 ntre x1 i a1 astfel nct f(x1, x2,, xn) - f(a1, x2,, xn) = (x1-a1)
f (1, x 2 ,..., xn ) . x1

Procednd analog cu funcia g2(t) = f(a1, t, x2,, xn) pe intervalul nchis determinat de punctele x2 i a2 rezult c exist 2 ntre a2 i x2 astfel nct: f(a1, x2,, xn) - f(a1, a2,, xn) = (x2-a2)
f (a1, 2 , x 3 ,..., xn ) . x 2

Continund procedeul i pentru celelalte paranteze rezult c exist 1 ntre x1 i a1, 2 ntre a2 i x2, , n ntre an i xn astfel nct f(x) - f(a) = (x1-a1) + (xn-an)
f f (1, x 2 ,..., xn ) + ( x2-a2) (a1, 2 , x 3 ,..., xn ) + x1 x 2 f (a1, a2 ,..., an 1, n ) . xn
n

Fie T: Rn R, T( x ) =
i =1

f (a)x i, care evident este liniar i oricare x B(a, r ) , xi

x a , avem
f ( x ) f (a) T( x a) = xa ( x 2 a2 )[ + ( x1 a1 )[ f f (1, x 2 ,..., x n ) (a1, a2 ,..., an )] x1 x1 xa

f f (a1, 2 , x 3 ,..., x n ) (a1, a2 ,..., an )] x 2 x 2 + ... x a f f (a1, a2 ,..., n ) (a1, a2 ,..., an )] xn xn . x a

( xn an )[ ... +
x i ai xa

Cum

, 1, pentru i = 1 n i derivatele pariale ale lui f sunt continue n

a = (a1, a2,, an) rezult c exist limita membrului doi al egalitii pentru x a i este egal cu zero. Prin urmare difereniabil n punctul a.
lim
x a

f(x) f(a) T(x a) = 0, xa

deci f este

- 104 -

Observaie. Din aceast teorem rezult c, dac f C1(A) , atunci f este

difereniabil pe A.
Exemplu. Fie funcia f:D R 2 R , f(x,y) = arctg
x i (x0, y0) = (2, -1) D . y

Avem:
f ( x, y ) = x f ( x, y ) = y 1 1 y = 2 , ()( x, y ) D ; 2 x y x + y2 1+ 2 y x x 1 ( 2 ) = 2 , ()( x, y ) D , 2 x y x + y2 1+ 2 y

deci f C1(D) i atunci f este difereniabil pe D, deci n (x0, y0) = (2, -1) i df(2, -1) =
f 2 f 1 (2,1)dx + (2,1)dy = dx dy . x y 5 5 2 1 h1 h2 . 5 5

Dac h R 2 , h = (h1, h2), atunci df(2, -1)( h1, h2) =

Presupunem n continuare c f este funcie vectorial, f:A Rn Rm , m 2 , f = (f1, f2, , fm), A deschis i a A .
Definiia 6.2.3. Funcia f este difereniabil n punctul a dac exist o

aplicaie liniar T: Rn Rm , astfel nct


lim f(x) f(a) T(x a) = 0, x a x a

(6.4)

sau echivalent (vezi cazul m = 1), dac exist T: Rn Rm , liniar i : A R m, continu n punctul a, (a) = 0 astfel nct f(x) = f(a)+T(x-a)+ ( x ) x a ,()x A . Ca i n cazul m = 1 se arat c aplicaia liniar T este unic i prin definiie T se numete difereniala funciei f n punctul a i se noteaz T = df(a).
Teorema 6.2.3. Fie A Rn , deschis, a A, f : A Rm , f = ( f1, f2 ,..., fm ). Atunci

funcia f este difereniabil n punctul a dac i numai dac funciile f1, f2 ,..., fm sunt difereniabile n a i n acest caz avem

- 105 -

df(a) = (df1(a),df2(a),, dfm(a)).


Demonstraie. Fie T: Rn Rm liniar, T = (T1, T2,, Tm), unde Ti: Rn R

este liniar, ()i = 1,m . Din proprietile normei euclidiene avem:


fi ( x ) fi (a) Ti ( x a) xa f ( x ) f (a) T( x a) xa

fi ( x ) fi (a) Ti ( x a)
i =1

xa

, ()x a,

de unde rezult teorema.


6.3. Difereniabilitatea funciilor compuse

Teorema

6.3.1.

Fie

u:A B ,

unde

A Rn ,B Rm sunt

deschise,

u = (u1, u2,, um), : B Rp (m, n, p 1 ), = (1, 2 ,..., p ). Dac u este difereniabil n a A i este difereniabil n b = u(a) B , atunci f = o u : A Rp este difereniabil n a i d f(a) = d (b) o du(a) .
Demonstraie. Fie T = du(a) : Rn Rm , L = d (b) : Rm Rp . Conform

definiiei difereniabilitii exist funciile : A Rm , : B Rp , continue n a, respectiv b, astfel nct (a) = 0, (b) = 0 i
u( x ) = u(a) + T( x a) + ( x ) x a , ()x A ; ( y ) = (b) + L( y b) + ( y ) y b , ()y B .

(6.5)

Lund n (6.5) y = u(x), cu x A obinem:


(u( x )) = (u(a)) + L(u( x ) u(a)) + (u( x )) u( x ) u(a) , ()x A , de unde rezult c
f ( x ) = f (a) + L(T( x a) + ( x ) x a ) + (u( x )) T( x a) + ( x ) x a = = f (a) + (L o T )( x a) + x a [L(( x )) + (u( x )) T( x a) + ( x ) ], ()x A, x a. x a

(6.6)

Fie : A R,
T( x a) + ( x ) , L(( x )) + (u( x )) ( x ) = xa o , daca x = a daca x a

- 106 -

Din (6.6) obinem: f ( x ) = f (a) + (L o T )( x a) + x a ( x ), ()x A . Vom arta c lim ( x ) = 0 .


x a

Cum L este operator liniar, este continuu i atunci


x a

lim L(( x )) = L((a)) = L(0) = 0 .


x a

Cum u este continu n a i n b, rezult c lim (u( x )) = (u(a)) = (b) = 0 . Cum T este operator liniar exist M > 0, astfel nct T( x ) M x ,()x Rn , i atunci
T( x a) T( x a) + ( x ) + ( x ) M + ( x ) , ()x A, x a , de unde va x a xa
lim (u( x )) T( x a) + ( x ) = 0 . xa

rezulta c

x a

n concluzie lim ( x ) = 0 = (a), deci f este difereniabil n a i


x a

df(a) = L o T = d (b) o du(a) .


Observaie. Dac Ju(a), J (b), Jf(a) sunt matricile jacobiene ale funciilor

u, , f, n a, b i respectiv a, din relaia df(a) = d (b) o du(a) obinem


Jf (a) = J (b) Ju (a) , de unde rezult i formulele pentru calculul derivatelor pariale

ale funciei f:
m fi u (a) = i (b) k (a),()i = 1, p, j = 1, n . x j x j k =1 uk

n particular, dac u este difereniabil pe A i este difereniabil pe B, atunci f = o u este difereniabil pe A i


m fi u = i k , ()i = 1, p, j = 1, n . x j k =1 uk x j

Cazuri particulare.
1. f ( x, y ) = (u( x, y ), v( x, y )), n = 2, m = 2, p = 1 ; f u v ; = + x u x v x f u v . = + y u y v y

- 107 -

2. f ( x, y ) = (u( x, y )), n = 2, m = 1 p = 1 ; , f u ; = ' (u( x, y )) x x f u . = ' (u( x, y )) y y


3. f ( x ) = (u( x ), v( x )), n = 1, m = 2, p = 1 ; f ' (x) = u' ( x ) + v' ( x ) . u v

Teorema 6.3.2. (Teorema de medie).

Fie A Rn , deschis, f : A R difereniabil pe A i [a,b] un segment din A ([a,b] = {(1- )a+ b: [0,1] }). Atunci exist (a, b) = {(1 )a + b : (0,1)} , astfel nct f(b)-f(a) = df( )(b-a).
Demonstraie. Fie funcia : [0,1] R , ( t ) = f (a + t(b a)) .

Cum funcia u : [0,1] A,u( t ) = a + t(b a) este difereniabil pe [0,1] i f este difereniabil pe [a,b], rezult c este difereniabil pe [0,1] i
' ( t ) =
i =1 n

f (a + t(b a))(bi ai ) = df (a + t(b a))(b a) . x i

Din teorema lui Lagrange exist (0,1) astfel nct (1) (0) = ' ( ) , sau echivalent f(b) - f(a) = df(a+ (b a) )(b-a) i demonstraia este ncheiat lund

= a+ (b a) (a,b) .

Definiia 6.3.1. O funcie f:A Rn R se numete omogen dac exist

p R astfel nct ( )x A i t > 0 cu tx A avem f(tx) = tpf(x). Numrul real p se numete grad de omogenitate.
Teorema 6.3.3. (Relaia lui Euler). Fie A Rn deschis, f:A R ,

difereniabil i omogen cu grad de omogenitate p. Atunci

xi x
i =1

= pf .
i

Demonstraie. Fie funcia cu valorile g(t) = f(tx) = f(tx1, tx2,, txn), unde

x = (x1, x2,, xn) A , este fixat iar t V , unde V (1), astfel nct tx A, ( )t V . Evident avem g(t) = f(u(t)), unde u: V A , u =(u1, u2,, un), ui(t) = txi, i 1, n .

- 108 -

Cum f este difereniabil pe A iar u este difereniabil pe V rezult c g = f o u este difereniabil pe V, deci derivabil pe V i
g' ( t ) =
i =1 n n f u f (u( t )) i ( t ) = x i (u( t )). ui t ui i =1

Pentru t = 1avem ui = xi, i = 1, n , i atunci g(1) = x i


i =1 n

f (x) . x i

(6.7)

Pe de alt parte cum f este omogen, avem:


f ( tx ) = t p f ( x ) , adic g(t) = t g(1) i deci g(t) = pt
p p-1

g(1), () t V.

Pentru t =1 obinem g(1) = pf(x), care combinat cu relaia (6.7) ncheie demonstraia.
6.4. Derivate pariale i difereniale de ordin superior Definiia 6.4.1. Fie A Rn , deschis, f:A R , derivabil parial pe A i
f f f ,..., , , derivatele sale pariale de ordinul nti. x1 x 2 xn

Funcia f este de dou ori derivabil parial n punctul a A , n raport cu variabilele xi i xj, unde i,j 1, n dac funcia a n raport cu variabila xj. Derivata parial de ordinul nti a funciei
f n punctul a n raport cu xi f este derivabil parial n punctul xi

variabila xj se numete derivata parial de ordinul doi a funciei f n punctul a i se noteaz prin:
2f f " (a ) = (a) sau fxix j (a) dac j i i xix j x j x i
f 2f " x (a ) = 2 (a) sau fxi2 (a) dac j = i. x i i x i

- 109 -

Derivata punctul a.

2f (a) cu j i se numete i derivat mixt de ordinul doi n xix j

Funcia f este de dou ori derivabil parial n punctul a dac toate funciile
f f f , ,..., sunt derivabile parial n punctul a. x1 x 2 xn

Funcia f este de dou ori derivabil parial pe A dac este de dou ori derivabil parial n toate punctele din A. n acest caz se pot defini funciile
x j f f : A R, x (x ), x x j xi i
i, j 1, n numite i derivatele pariale de ordinul

doi ale lui f. Derivatele


f cu j i se numesc derivate pariale mixte de ordinul x j x i 2f f " = x x x sau fx i x j . x j i i j
f 2 f " = sau fx2 . x i x i x i2 i

doi i se noteaz prin

Pentru j = i acestea se noteaz prin

Dac f:A Rn R este de dou ori variabil parial n punctul a vom nota cu
2f H(a) = (a) n (R) i o vom numi matricea hessian a funciei f n a. xix j

Exemplu. Fie funcia f:D R 2 R , f(x,y) = arctg . Ne propunem s

y x

determinm H(1,-2). Funcia f este derivabil parial pe D i


f = x 1 y2 1+ 2 x 1 f = y2 y 1+ 2 x y y 2 = 2 2 x x +y 1 x . = 2 x x + y2

(derivatele pariale sunt calculate n punctul curent (x,y)).

- 110 -

Funciile

f f sunt derivabile parial pe D i , x y

2 xy 2 f f = = 2 x 2 x x ( x + y 2 ) 2 y2 x2 2 f f ( x 2 + y 2 ) + y 2 y = = = 2 (x2 + y 2 )2 (x + y 2 )2 yx y x y2 x2 2 f f x 2 + y 2 x 2 x = 2 = = (x2 + y 2 )2 (x + y 2 )2 yx x y 2 f f 2 xy = = 2 y 2 y y ( x + y 2 ) 2
(derivatele pariale sunt calculate n punctul curent)
2f (1,2) x 2 Vom avea H(1,2) = 2 f (1,2) yx 2f (1,2) 4 25 xy = 2 3 f (1,2) 25 y 2 3 25 . 4 25

Derivatele pariale de ordin superior se vor defini n mod asemntor cu derivatele pariale de ordinul doi. Astfel, derivatele pariale de ordinul trei se vor defini ca derivatele pariale de ordinul nti ale derivatelor pariale de ordinul doi. Continund recurent, vom putea defini derivatele pariale de ordin k 2 ale funciei f ca derivatele pariale de ordinul nti ale derivatelor pariale de ordin k-1. De exemplu, dac f:D R3 R, f = f ( x, y, z),
6f 2 = 2 xy 3z 2 z 3 f 3 . y x

De asemenea, spunem c f este derivabil parial de ordin k n raport cu variabila xi, i 1,n dac toate derivatele pariale de ordin k-1 sunt derivabile n raport cu variabila xi. Derivatele pariale de ordin superior calculate n raport cu cel puin dou variabile diferite se numesc derivate pariale mixte.

- 111 -

Definiia 6.4.2. Fie A Rn , deschis i f : A R . Funcia f este clas Ck pe

A (k 2 ) i scriem f Ck ( A ) dac f este de k ori derivabil parial pe A i derivatele pariale de ordinul k sunt continue pe A. Funcia f este de clas C pe A, f C ( A ) dac f Ck ( A ),()k 0 . n exemplul dat n acest subcapitol ( f(x,y) = arctg
y ) am vzut c x

derivatele pariale mixte de ordinul doi sunt egale. Acest lucru nu este adevrat ntotdeauna. n acest sens considerm funcia f : R 2 R ,
x 3 y xy3 , daca (x, y ) (0,0 ) f ( x, y ) = x 2 + y 2 0, daca (x, y ) = (0,0 ) .

Printr-un calcul simplu obinem


4 4 2 2 f (x, y ) = y x y + 4x2 y , ()( x, y ) (0,0) ; x (x2 + y2 ) 4 4 2 2 f (x, y ) = x x y 4x2 y , ()( x, y ) (0,0) . y (x2 + y2 )

de unde

f f (0, y ) = y i ( x,0) = x, ()x 0, y 0 . x y lim


x 0

Cum

f ( x,0) f (0,0) 0 = lim = 0 , x 0 x x f (0, y ) f (0,0) 0 lim = lim = 0 , y 0 y 0 y y f f (0,0) = 0, (0,0) = 0 . x y

rezult c f este derivabil parial n (0,0) i

Vom calcula acum derivatele mixte de ordinul doi n (0,0):


f f (0, y ) (0,0) y0 f x (0,0) = lim x = lim = 1 , y 0 y 0 y y xy f f ( x,0) (0,0) 2 f x y y (0,0) = lim = lim = 1 , x 0 x 0 x yx x
2

deci

2f 2f (0,0) (0,0) . xy yx

- 112 -

Urmtorul rezultat furnizeaz o condiie suficient care s asigure egalitatea derivatelor pariale mixte de ordinul doi ntr-un punct.
Teorema 6.4.1. (teorema lui Schwarz).
2f Fie A R , deschis i f:A R . Dac f are derivate pariale mixte i xix j
n

2f 2f 2f (i j) ntr-o vecintate V a lui a A, i funciile , sunt continue x jxi xix j x jxi

n a, atunci
2f 2f (a ) = (a ) . xix j x jxi

(6.8)

Demonstraie. Vom demonstra mai nti teorema pentru n = 2.

Fie f : A R , A R2 , f = f ( x, y ), a A, a = ( x 0 , y 0 ) .Fie V = B(a,r) (a), pe care exist funciile


2f 2f , . Pentru (x,y) V, x x0 i y y0 considerm expresia xy yx

E(x,y) = f(x,y)-f(x0,y)-f(x,y0)+f(x0,yo) (x,y) I x J B(a,r). Fie funcia

(6.9)

Fie I,J R dou intervale deschise astfel nct x0I, y0J i g : I R , g( t ) = f ( t, y ) f ( t, y 0 ) , si atunci E (x,y) = g(x) - g(x0). Cum f este derivabil parial n raport cu x pe V, rezult c g este derivabila pe I i din teorema lui Lagrange exist ntre x0 i x, astfel nct E (x,y) = g(x) - g(x0) = (x-x0)g( ). Dar, din (6.10), g' () = (6.11) (6.10)

f f (, y ) (, y 0 ) , i atunci folosind (6.11) obinem: x x

E(x,y) = (x-x0)[ Fie funcia h : J R , h( ) =

f f (, y ) (, y 0 ) ]. x x

(6.12)

f (, ) . Din ipotez h este derivabil pe J i din x

teorema lui Lagrange exist ntre y0 i y astfel nct h(y) - h(y0) = (y-y0)h( ) = (y-y0)
2f (, ) , i atunci din (6.12) obinem: xy

- 113 -

E(x,y) = (x-x0)(h(y)-h(y0)) = (x-x0)(y-y0)

2f (, ) . xy

(6.13)
~

Schimbnd pe x cu y i raionnd analog gsim c exist un punct ntre x0 i x i un punct ~ ntre y0 i y astfel nct E(x,y) = (x-x0)(y-y0) Din (6.13) i (6.14) obinem
2f ~ ~ ( , ) . yx

(6.14)

2f 2f ~ ~ (, ) = ( , ) . xy yx ~

Fie un ir de puncte (xn,yn) V astfel nct (xn,yn) (x0,y0), cu xn x0,


~ yn y0. Atunci exist punctele n , n ntre x0 i xn i punctele n ,n ntre y0 i yn

astfel nct
2f 2f ~ ~ (n , n ) = ( n , n ) , ()n N . xy yx

Cum (xn,yn) (x0,y0) rezult c n x 0 , n x 0 , n y 0 , ~n y 0 , deci

(n, n ) (x 0 , y 0 ) , (~n, n ) (x0 , y0 ) i folosind continuitatea funciilor ~


2f 2f , obinem xy yx
2 2f (x0 , y0 ) = f (x0 , y 0 ) . yx xy

a=(a1,a2, an) A i funcia cu valorile g(x, y ) = f (a1,...ai 1, x, ai +1,..., a j 1, y, a j +1,..., an ) definit pe un deschis ce conine punctul (ai,aj). Folosind prima parte a demonstraiei (cazul n=2) vom avea
2f 2g 2g 2f (a ) = (a ) = (a ) = (a) , i demonstraia este xix j xy yx x jxi

Dac n>2, fr a restrnge generalitatea putem presupune i<j. Fie

ncheiat.

Corolarul 6.4.1. Fie A Rn deschis i f:A R . Dac f C1 ( A ) i derivatele

pariale mixte de ordinul doi atunci

2f 2f , ( j i) exist si sunt continue pe A xix j x jx i

- 114 2f 2f = . xix j x jx i

Corolarul 6.4.2. Dac f Ck ( A ), k 2 , atunci derivatele pariale mixte de

ordin q k nu depind de ordinea de derivare. O alt condiie suficient pentru ca derivatele pariale mixte de ordinul doi s fie egale este dat de criteriul lui Young.
Teorema 6.4.2. Fie A Rn , deschis i f : A R . Dac f este derivabil

parial pe o vecintate V a punctului a A, iar derivatele pariale

f , i = 1, n sunt xi

difereniabile n punctul a atunci exist derivatele pariale mixte de ordinul doi i sunt egale n a. Pentru demonstraie se poate consulta [11].
Definiia 6.4.3. Fie A Rn deschis i f : A R . Funcia f este de k (k 2 )

ori difereniabil ntr-un punct a A dac f este de k-1 ori derivabil parial ntr-o vecintate V a lui a, iar toate derivatele pariale de ordin k-1 ale lui f sunt difereniabile n a. Funcia f este de k ori difereniabil pe A dac este de k ori difereniabil n orice punct a A.
Observaie. Folosind teorema 6.2.2. rezult c, dac f : A Rn R este

derivabil parial de ordin k pe A i dac derivatele pariale de ordin k sunt continue n a, atunci f este de k ori difereniabil n a. n particular, dac f Ck ( A ) atunci f este de k ori difereniabil pe A.
Definiia 6.4.4. Fie A Rn , deschis, f:A R , o funcie de k ori

difereniabil ntr-un punct a A . Numim diferenial de ordinul k a funciei f n punctul a funcia dk f (a) : Rn R definit prin:
dk f (a)(h) = (h1 f f f (a ) + h 2 (a) + ... + hn (a))(k ) , x1 x 2 xn

(6.15)

unde ridicarea la puterea simbolic k se face n sensul lurii derivatelor.

- 115 -

Astfel (

k f f (a))(k ) reprezint (a ) , xi x k i

f k f ( k 1) f (a ) , (a)) reprezint (a)) ( ( x i x j xk 1x j i

f f k f (a))( k 2 ) ( (a))( 2 ) reprezint (a) , etc. xi x j x k 2x 2 i j

Observaii. 1. In cazul k = 2, d2f(a) este o form ptratic:


d2 f (a)(h) = 2f x x (a)hih j,()h = (h1,h2,...,hn ) Rn, cu i j 1i, j n

matricea

asociat

H(a) = (

2f (a))1i, jn , deci tocmai matricea hessian a lui f n a. xix j

Evident H(a) este o matrice simetric.


2. Presupunem c f este de k ori difereniabil n a. Notnd cu dxi

difereniala funciei pri n punctul a, dxi = dpri(a), i 1, n , formula 6.15 se mai scrie i astfel : dkf(a)(h) = [
n f f (k) k (k) (a)dx i (h) ] , ()h Rn , de unde d f(a) = [ (a)dx i ] . i =1 x i i =1 x i
k

Dac f este difereniabil de k ori pe A, atunci d f = [


3. Pentru n = 2,
f : A R 2 R, f = f ( x, y ), a A, a = ( x 0 , y 0 ), dk f (a)(h) = ( f f (a)h1 + (a)h2 )(k ), ()h = (h1, h2 ) R 2 , x y

f (k) dx i ] . i =1 x i

sau dk f (a) = (

f f (a)dx + (a)dy )(k ). x y

Pentru k = 2,
2f 2f d f ( x 0 , y 0 ) = 2 ( x 0 , y 0 )dx + 2 ( x 0 , y 0 )dxdy + 2 ( x 0 , y 0 )dy 2 . xy x y
2 2

2f

Exemplu. Fie funcia

f : A R2 R , f ( x, y ) = ln x 2 + y 2 .

S calculm

diferenialele de ordinul unu i doi ale lui f n punctul curent.

- 116 -

Avem
f = x 1 x2 + y2 2 x2 + y2 2x = x x2 + y2

y f = 2 y x + y 2
df ( x, y ) = x y dx + 2 dy , ()( x, y ) A. 2 x +y x + y2
2

, de unde

Dac (x0, y0) = (1, -1) atunci df(1, -1) =

1 1 dx- dy. Pentru a determina d2f 2 2

vom calcula mai nti derivatele pariale de ordinul doi. Vom avea
2f y2 x2 2f x2 y2 2f 2xy = 2 , = 2 , = 2 . 2 2 2 2 2 2 x (y + x ) y (y + x ) xy ( y + x 2 )2

Atunci d2 f ( x, y ) = d2f(1, -1) = dxdy.

y2 x2 (x + y )
2 2

dx 2 2

4 xy (x + y )
2 2

dxdy + 2

x2 y2 (x + y )
2 2 2

dy 2 , iar

Teorema 6.4.3.(Formula lui Taylor). Fie A Rn , deschis, f : A R , o

funcie de k+1 ori difereniabil pe A, a A i r > 0, astfel nct B(a,r) A. Atunci, pentru orice punct x B(a, r ), x a exist un punct pe segmentul deschis de extremiti a i x astfel nct
1 1 df (a)( x a) + d2 f (a)( x a) + ... 1 ! 2! 1 1 ... + dk f (a)( x a) + dk +1f ()( x a). k! (k + 1)! f ( x ) = f (a ) +

(6.16)

Demonstraie. Fie s = (s1, s2,, sn) Rn , un versor. Dac t (r,r ) , atunci

x = a + ts B( r, r ) A i x a , dac t 0 . S considerm funcia g:(-r,r) R , g(t) = f(a+ts) = f(a1+ ts1, a2+ ts2,, an+ tsn). Cum f este de k+1 ori difereniabil pe A rezult c g este de k+1 ori difereniabil pe (-r, r). Vom calcula derivatele lui g pn la ordinul k+1.

- 117 -

Vom avea
g' ( t ) = s1
" 2 1

f f f (a + ts), ()t ( r, r ); (a + ts) + ... + sn (a + ts) + s2 x n x 2 x1

2f 2f 2f (a + ts) + (a + ts) + ... + s1sn (a + ts) + s1s2 g (t) = s 2 x1xn x1x 2 x1


2 ... + sn

f f 2f (a + ts)]( 2 ) = [g' ( t )]( 2 ). (a + ts) + ... + sn (a + ts) = [s1 2 x n x1 xn

Procednd inductiv obinem


g( m ) ( t ) = [g' ( t )]( m ) , pentru m = 1, 2,, k+1.

(6.17)

Din formula lui Mac-Laurin cu rest de ordin k sub forma Lagrange aplicat funciei g gsim c pentru ()t ( r,r ), t 0, exist ntre 0 i t astfel nct
t ' t2 " t k (k ) t k +1 (k +1) g( t ) = g(0) + g (0) + g (0) + ... + g (0) + g ( ) . 1 ! 2! k! (k + 1)!

(6.18)

Cum x = a+ ts x-a = ts, din 6.17 obinem


tg' (0) = ts1 f f f (a) + ... + tsn (a ) + (a) = ( x1 a1 ) x1 xn x1 f (a) = df (a)( x a); xn

... + ( xn an )
2 " 2 2 1

2 2f f 2 2 f t g (0 ) = t s (a) + ... + t sn 2 (a) = [( x1 a1 ) (a ) + 2 x1 xn x1

... + ( xn an )

f (a)]( 2 ) = d2 f (a)( x a); xn

.................... .......... .................... .......... ............. t k gk (0) = dk f (a)( x a);

i
t k +1g(k +1) ( ) = t k +1[s1 = [ ts1 f f (a + s) + ... + sn (a + s)](k +1) = x1 x n

f f (a + s)]( k +1) = (a + s) + ... + tsn xn x1 f f () + ... + ( x n an ) ()]( k +1) = dk +1f ()( x a), x1 xn

= [( x1 a1 )

unde = a + s (a, x ) . nlocuind derivatele gsite n 6.18 i innd cont c g(t) = f(x) iar g(0) =f(a), obinem formula (6.16).

- 118 -

Observaie. Dac Rk ( t ) =

t k +1 (k +1) 1 ~ g ( ) i Rk ( x ) = dk +1f ()( x a) sunt (k + 1)! (k + 1)!


~ Rk ( x ) xa
k

resturile de ordin k sub forma Lagrange ale lui g i respectiv f, atunci, din
t 0

lim

Rk ( t ) t
k

= 0 , rezult c exist lim

x a

~ = 0 i n particular lim Rk ( x ) = 0 , deci


x a

pentru x ntr-o vecintate V a lui a avem aproximarea


1 1 f ( x ) f (a) + df (a)( x a) + ... + dk f (a)( x a) , 1 ! k!

deci funcia se aproximeaz printr-un polinom de gradul k. n particular, pentru n = 2, f = f(x,y), a = (x0, y0) rezult c pentru (x,y) ntr-o vecintate V a lui (x0, y0) avem
1 f ( x, y ) f ( x 0 , y 0 ) + df ( x 0 , y 0 )( x x 0 , y y 0 ) = 1 ! f f = f ( x 0 , y 0 ) + ( x 0 , y 0 )( x x 0 ) + ( x 0 , y 0 )( y y 0 ), x y

aproximarea de ordinul nti,


1 f ( x, y ) f ( x 0 , y 0 ) + df ( x 0 , y 0 )( x x 0 , y y 0 ) + 1 !

f f 1 2 ( x 0 , y 0 )( x x 0 ) + ( x 0 , y 0 )( y y 0 ) + d f ( x 0 , y 0 )( x x 0 , y y 0 ) = f ( x 0 , y 0 ) + x y 2! 1 2f 2f 2f ( x 0 , y 0 )( x x 0 )2 + 2 ( x 0 , y 0 )( y y 0 )2 + 2 ( x 0 , y 0 )( x x 0 )( y y 0 ), 2 2 x xy y

aproximarea de ordinul doi.


6.5. Funcii i sisteme de funcii implicite

Considerm ecuaia F(x,y)=0, unde F:D R 2 R . Ne propunem s rezolvm aceast ecuaie n raport cu x sau y.
Exemple. 1. Fie ecuaia 4x-3y=5, (x,y) R 2 , care este de forma F(x,y)=0, unde

F(x,y) = 4x-3y-5. Aceast ecuaie poate fi rezolvat n raport cu y i are o unic soluie y =

4x 5 , deci exist o unic funcie f : R R , astfel nct y = f(x), unde 3

- 119 -

f(x) = cu x.

4x 5 , este soluie a ecuaiei. Analog ecuaia poate fi rezolvat n raport 3


2.

Fie ecuaia x-y2 = 0, (x,y) R 2 . Ecuaia admite o singur soluie n cu x i anume x = y2, deci exist o unic funcie g : R R , astfel nct

raport

x = g(y), unde g(y) = y2, este soluie a ecuaiei. n raport cu y ecuaia admite o infinitate de soluii pe [0, ), de exemplu: f ( x ) =
x , x A1 x , x A 2

unde

A1,A2 [0, ),A1 A2 = [0, ), A1 A2 = . Dintre acestea doar dou sunt continue i anume f1(x)= x i f2(x)= x , oricare x [0, ) .

3. Ecuaia x2+y2+3 = 0, (x,y) R 2 nu are nici o soluie n raport cu x sau y.

Fie ecuaia unde F : A x B R x R R .


n

F(x1, x2,, xn, y) = 0 ,

(6.19)

Ecuaia (6.19) se scrie echivalent F(x,y) = 0, unde x = (x1, x2,, xn).


Definiia 6.5.1. O funcie f : A B se numete soluie a ecuaiei (6.19)

pe A dac F( x1, x 2 ,...., xn , f ( x1, x 2 ,...., xn ) ) = 0, ( ) ( x1, x 2 ,...., xn ) A , sau echivalent F(x, f(x)) = 0, ( ) x A . Dac exist o singur funcie f : A B care s verifice ecuaia (6.19) i eventual i alte condiii suplimentare spunem c funcia f este definit de ecuaia F(x,y) = 0. Funciile definite cu ajutorul ecuaiilor se numesc funcii definite implicit sau funcii implicite.

- 120 -

Teorema 6.5.1. (Teorema funciilor implicite).

Fie A Rn ,n 1,B R , deschise, (a, b)A x B i F:A x B R astfel nct


1) F(a, b)=0; 2) ()U (a),U A, V (b), V B astfel nct F C1(U V ) ; 3)
F (a, b) 0 . y

Atunci:
a) ecuaia F (x,y) = 0 definete funcia y pe o vecintate a punctului (a,b),

adic exist o vecintate U0 (a ) , o vecintate V0 (b ) i o unic funcie


f : U0 V0 cu valorile y = f(x) astfel nct f(a) = b i F (x, f(x)) = 0, () xU0;

Fxi ( x, f ( x )) f b) f C (U0 ) i (x) = , ()x U0 ; xi F ( x, f ( x ))


1

c) dac FCk (U x V), k 2 , atunci fCk(U0). Demonstraie. a) Fr a restrnge generalitatea s presupunem c


F (a, b) > 0 . Cum y

funcia nct

F este continu pe U x V, () U1 (a), U1 U, V1(b), V1 V astfel y

F ( x, y ) > 0 , () (x,y)U1 x V1. y

Fie , V1 astfel nct < b < i V0 = (,) (b). Funcia y F(a, y ) se anuleaz n b, are derivat pozitiv pe V0 deci este strict cresctoare pe V0 i F(a,) < 0, F(a,) > 0. Cum funcia x F(x,) este continu pe U1, exist o vecintate
U (a),U U1 astfel nct F(x,) < 0, () x U .

De asemenea, cum funcia x F(x,) este continu pe U1, exist o vecintate U (a ) , U U1 astfel nct F(x,) > 0, () x U . Fie U0 = U U (a) i vom avea

- 121 -

F(x,) < 0, F(x,) > 0, () xU0. Fie acum x U0 , arbitrar. Cum funcia y F( x, y ) este continu pe [,], strict cresctoare pe [,] i
F( x, ) < 0 , F( x, ) > 0 , rezult c exist un unic

punct y (, ) astfel nct F( x, y) = 0 . Deoarece x U0 a fost ales arbitrar rezult c pentru orice punct xU0, fixat, exist un singur punct yV0 astfel nct F(x,y) = 0. Definim f : U0 V0, f(x) = y i atunci f este bine definit i F(x, f(x)) = 0, () xU0. Pentru x = a avem F(a,b) = 0 i cum b este singurul punct din V0 cu aceast proprietate deducem f(a) = b i demonstraia punctului a) este ncheiat.
b) S observm mai nti c f este continu n a. Pentru vecintatea V0

aleas n mod arbitrar, din (a) exist o vecintate U0(a) astfel nct pentru orice xU0, f(x)V0, deci f este continu n a. Analog raionm pentru x U0 i atunci rezult c f este continu pe U0. Fie a = (a1, a2, , an), (a1, a2, ,ai-1, xi, ai+1,, an)U0, unde i 1,n i y = f (a1, a2, ,ai-1, xi, ai+1,, an)V0. Din Teorema lui Lagrange exist ntre ai i xi, ntre b i y astfel nct 0 = F(a1, a2, ,ai-1, xi, ai+1,, an, y) - F(a1, a2,, an, b) = = F(a1, a2, ,ai-1, xi, ai+1,, an, y) - F(a1, a2,, ai-1, ai, ai+1,, an, y) + + F(a1, a2,, ai-1, ai, ai+1,, an, y) - F(a1, a2,, an, b) = =
F (a1, a2,..., ai1, , ai+1,..., an, y )( xi ai ) + F (a1, a2,..., an, )( y b) , xi y

de unde rezult c
F (a1, a2,..., ai1, , ai+1,..., an, y ) + xi
+ F (a1, a2,..., an, ) f (a1, a2,..., ai1, xi, ai+1,..., an ) f (a1, a2,..., an ) = 0 , x i ai y

pentru x i ai . Pentru x i ai avem i ai, b = f (a) i folosind continuitatea funciilor

- 122 F F si pe U0 x V0, prin trecere la limit cu xiai obinem x i y F F f F (a, f(a)) + (a, f(a)) (a) = 0 i cum (a, f(a)) 0 rezult y xi y xi
' Fxi (a, f(a)) f (a) = ' . xi Fy (a, f(a))

Relund raionamentul cu x n loc de a, unde xU0 este arbitrar i cu y = f(x)V0 gsim c exist
Fx (x, f(x)) f , () xU0, i = 1, n . (x) = i xi F (x, f(x))
y

Cum F C1(U V ) va rezulta c

f , i = 1,n sunt continue pe V0, deci xi

f C1(U0 ) . Mai mult, folosind teorema 6.2.2 va rezulta c f este difereniabil pe

U 0.
c) Prin inducie dup k.

Exemplu. S se arate c ecuaia x3 - y3 + x + y = 10 definete funcia y pe

o vecintate a punctului (2,1). S se calculeze y(2), y(2) . Ecuaia este de forma F(x,y) = 0, unde F:R2R, F(x,y) = x3 - y3 + x + y - 10. Evident F(2,1)=0, F C1 (R2),
F (x, y) = -3y 2 + 1 , () (x,y)R2, de unde y

F (2,1) = -2 0 , deci ipotezele teoremei 6.5.1 sunt ndeplinite. Conform y

teoremei ecuaia F(x,y) = 0 definete funcia y pe o vecintate a punctului (2,1), adic () U0(2), U0 R ,V0 (1), V0 R i o unic funcie y : U0V0, cu valorile y = y(x) astfel nct y(2) =1 i F(x,y(x)) = 0, ()xU0, adic x3 - y3(x) + x + y(x) =10, () xU0. Din teorema 6.5.1(b) funcia y este derivabil pe U0 i derivnd ultima relatie n raport cu x obinem 3x2 - 3y2(x) y( x ) + 1 + y( x ) = 0, () xU0. Pentru x = 2 avem y(2) = 1 i atunci (6.20)

- 123 -

12- 3 y(2) + 1 + y(2) = 0, de unde y(2) = Derivnd din nou n (6.20) n raport cu x obinem

13 . 2

6 x 6 y( x )y2 ( x ) 3 y 2 ( x )y( x ) + y( x ) = 0, ()xU0.

Pentru x = 2 obinem
12 6 483 169 3 y(2) + y(2) = 0, de unde y(2) = . 4 4

Sisteme de funcii implicite

Considerm sistemul de ecuaii


F1( x1, x 2 ,..., x n , y1, y 2 ,..., y m ) = 0 F2 ( x1, x 2 ,..., x n , y1, y 2 ,..., y m ) = 0 .......... .......... .......... .......... .......... . Fm ( x1, x 2 ,..., x n , y1, y 2 ,..., y m ) = 0

(6.21)

unde F1, F2,,Fm: A x B Rn x Rm R, m,n N * . Dac x = ( x1, x 2 ,..., xn ), y = ( y1, y 2 ,..., ym ) i F = (F1, F2,,Fm) atunci sistemul (6.21) se scrie echivalent sub forma unei ecuaii vectoriale F(x,y) = 0 (6.22)

Definiia 6.5.2. O funcie f : A B, f = (f1, f2,.,fm) se numete soluie a

sistemului (6.21) (sau echivalent a ecuaiei vectoriale (6.22)) pe A dac


F1 (x1, x 2 ,..., x n , f1(x1, x 2 ,..., xn ),..., fm (x1, x 2 ,..., x n )) = 0 F2 (x1, x 2 ,..., x n , f1(x1, x 2 ,..., xn ),..., fm (x1, x 2 ,..., xn )) = 0 , () xA .......... .......... .......... .......... .......... . Fm (x1, x 2 ,..., x n , f1(x1, x 2 ,..., x n ),..., fm (x1, x 2 ,..., x n )) = 0

(sau echivalent F(x,f(x))= 0, ()xA). n cazul n care sistemul (6.21) are pe mulimea A o singur soluie f = (f1, f2,.,fm) spunem c funiile f1, f2,.,fm constituie un sistem de funcii definite n mod implicit sau un sistem de funcii implicite.
Teorema 6.5.2. Fie sistemul de ecuaii (6.21) n urmtoarele ipoteze:
m A Rn ,B Rm , deschise, F:A x B R , F =(F1, F2,,Fm), (a,b)A x B astfel nct

1. F(a,b) = 0 ( F1(a,b) = F2 (a, b) = ..... = Fm (a,b) = 0 ) ;

- 124 -

2. ()U(a), U A i V (b), V B astfel nct

F C1(U V ) F1,F2 ,...,Fm C1(U V ) ;


3.
D(F1,F2 ,...,Fm ) (a, b) 0 . D(y1, y 2 ,..., ym )

Atunci:
a.

sistemul (6.21) definete funciile y1, y2,...,ym

pe o vecintate a

punctului (a,b), adic exist o vecintate U0(a), o vecintate V0(b), i o unic funcie f:U0V0, f = (f1, f2,.,fm) cu valorile y = f(x)

( y1 = f1( x1, x 2,..., xn ),..., ym = fm (x1, x 2,..., xn )) astfel nct b = f(a)


( b1 = f1(a),..., bm = fm (a )) i F(x,f(x)) = 0, ()xU0.
b. f C1(U0 ) i
D(F1,F2 ,..., Fm ) ( x, f ( x )) f1 D(xi, y 2 ,..., ym ) , () x U0 , i = 1, n (x) = D(F1,F2 ,..., Fm ) xi ( x, f ( x )) D(y1, y 2 ,..., ym ) .......... .......... .......... .......... .......... .......... .... D(F1,F2 ,..., Fm ) ( x, f ( x )) fm D(y1, y 2 ,..., ym 1, x i ) (x) = , () x U0 , i = 1, n D(F1,F2 ,..., Fm ) xi ( x, f ( x )) D(y1, y 2 ,..., y m )

c. dac FCk(U x V), k 2 , atunci fCk(U0) Demonstraie. Prin inducie dup m.

Pentru m=1 teorema se reduce la teorema 6.5.1 iar pentru celelalte detalii se pot consulta lucrrile [8] sau [11] .
Exemplu. Fie sistemul de ecuaii
xyu yv 2 + 2v 3 = 0 2 4u + 2v 2 x 3 y = 0

(6.23)

S se arate c sistemul definete funciile u i v pe o vecintate a punctului (x0, y0, u0, v0) = (1,2,0,1). S se calculeze du(1,2), dv(1,2). Sistemul (6.23) este de forma
F1( x, y, u, v ) = 0 , unde F1, F2 : R4 R, F2 ( x, y, u, v ) = 0

F1(x,y,u,v) = xyu yv2 + 2v3,

F2(x,y,u,v) = 4u2 + 2v2 x3y.

- 125 -

Evident avem F1(1,2,0,1) = 0, F2(1,2,0,1) = 0, F1, F2 C1(R 4 ) ,


F1 D(F1, F2 ) = u F2 D(u, v ) u F1 2 v = xy 2yv + 6v , de unde F2 8u 4v v

2 2 D(F1, F2 ) (12,0,1) = , = 8 0, 0 4 D(u, v )

deci ipotezele teoremei (6.5.2) sunt ndeplinite. Conform teoremei sistemul (6.23) poate fi rezolvat n raport cu u i v pe o vecintate a punctului (1,2,0,1), deci pe o vecintate U0((1,2)) avem u = u(x,y), v = v(x,y), u(1,2) = 0, v(1,2) = 1 i
xyu( x, y ) yv 2 ( x, y ) + 2v 3 ( x, y ) = 0 , () (x,y)U0 2 4u ( x, y ) + 2v 2 ( x, y ) x 3 y = 0

(6.24)

Vom avea du(1,2) = dv(1,2) =


u u (1 2)dx + (1,2)dy ; , x y v v (1 2)dx + (1,2)dy . , x y

Derivnd n sistemul (6.24) n raport cu x obinem


u v v 2 yu( x, y ) + xy x ( x, y ) 2yv( x, y ) x ( x, y ) + 6v ( x, y ) x ( x, y ) = 0 , 8u( x, y ) u ( x, y ) + 4v( x, y ) v ( x, y ) 3 x 2 y = 0 x x

() (x,y)U0. Pentru (x,y) = (1,2) obinem


v v u , 2 x (1,2) 4 x (1 2) + 6 x (1,2) = 0 , v 4 (1,2) 6 = 0 x

de unde

v 3 u 3 i (1 2) = , (1 2) = . , x 2 x 2

Derivnd sistemul (6.24) n raport cu y obinem


u v v 2 2 xu( x, y ) + xy y ( x, y ) v ( x, y ) 2yv( x, y ) y ( x, y ) + 6v ( x, y ) y ( x, y ) = 0 , () (x,y)U0 u v 3 8u( x, y ) ( x, y ) + 4v( x, y ) ( x, y ) x = 0 y y

- 126 v v u 2 y (1,2) 1 4 y (1,2) + 6 y (1,2) = 0 , 4 v (1,2) 1 = 0 y

Pentru (x,y) = (1,2) obinem

de unde

v 1 u 1 i atunci (1,2) = , (1,2) = y 4 y 4 3 1 3 1 du(1,2) = dx + dy, dv(1,2) = dx + dy 2 4 2 4

Transformri regulate

Fie A Rn , deschis i f : A Rn, f = (f1, f2, ,fn).


Definiia 6.5.3. Funcia vectorial f este o transformare regulat n punctul

aA dac funciile f1, f2, ,fn au derivate pariale continue ntr-o vecintate V a lui a i
D( f1, f2 ,...., fn ) (a ) 0 . D( x1, x 2 ,...., x n )

Funcia f este o transformare regulat pe A dac este o transformare regulat n orice punct din A.
Observaie. Cum jacobianul transformrii este o funcie continu rezult

c, dac f este o transformare regulat ntr-un punct aA atunci f este o transformare regulat ntr-o ntreag vecintate a lui a. Mai mult, dac f este o transformare regulat pe A i A este conex atunci det Jf are un semn constant pe A. Folosind teorema 6.5.2 se poate demonstra urmtorul rezultat (vezi [8]):
Teorema 6.5.3. (teorema de inversiune local).

Fie f: A Rn Rn , A deschis, o transformare regulat ntr-o vecintate U a punctului aA. Atunci exist o vecintate U0(a), U0 U i o vecintate V0(f(a)) astfel nct restricia lui f la U0 este o aplicaie bijectiv de la U0 pe V0. Mai mult, dac g= ( f |U0 )1 atunci g este o transformare regulat n punctul b = f(a) i
D(g) (b) = D(f1 . ) (a ) D(y ) D(x )

- 127 -

6.6. Extreme locale pentru funcii reale de mai multe variabile Definiia 6.6.1. Fie A Rn i f : A R. Un punct aA se numete punct de

minim (respectiv maxim) local pentru funcia f dac exist o vecintate V(a) astfel nct f ( x ) f (a) (respectiv ), () xV I A. Dac aceste inegaliti au loc () xA spunem c a este punct de minim (respectiv maxim) global sau absolut. Un punct aA se numete punct de extrem local pentru f dac este punct de minim local sau de maxim local pentru f.
Definiia 6.6.2. Fie A Rn, deschis i f : A R.

Un punct aA se

numete punct critic sau staionar pentru funcia f dac f este difereniabil n a i df(a)=0.
Teorema 6.6.1. (Teorema lui Fermat). Fie A Rn , deschis i f:AR.

Dac aA este un punct de extrem local pentru f iar f este difereniabil n punctul a atunci df(a) = 0, adic a este punct critic.
Demonstraie. Fr a restrnge generalitatea s presupunem c a este

un punct de minim local pentru f. Atunci () r>0 astfel nct B(a,r) A i f(x) f(a), ()xB(a,r). Fie sRn un versor i funcia g: (-r,r)R, g(t)=f(a+ts) i s observm c g este bine definit deoarece pentru t(-r,r) rezult c a+tsB(a,r) A. Lund x = a + ts B(a, r) vom avea g(t) g(0), () t(-r, r) deci t = 0 este punct de minim local pentru g. Cum f este difereniabil n a rezult c f este derivabil n a dup direcia s deci g este derivabil n t = 0 i conform teoremei lui Fermat de la funciile reale de variabil real rezult c g(0) = 0. Dar aceasta implic
df (a ) = 0 . ds

Lund s = ei, i = 1,n , unde e1, e 2 ,..., en sunt versorii bazei canonice obinem
df (a) = f (a)dx i = 0 . i =1 x i
n

- 128 -

Observaie. Din teorema lui Fermat rezult c mulimea punctelor de

extrem local se afl printre mulimea punctelor critice, adic printre mulimea
f x = 0 1 f =0 x 2 ............. f x = 0 n f f (0,0) = 0, (0,0) = 0, deci df(0,0)=0. x y

soluiilor sistemului

Reciproca acestei teoreme nu este n general adevrat. n acest sens fie funcia f:R2R, f(x,y) =x2-3y2. Evident avem

Mai mult, f(x,0)-f(0,0)=x2>0, () (x,y)R2, x 0, f(0,y)-f(0,0)= -3y2<0, () (x,y)R2, y 0, deci diferena f(x,y)-f(0,0) nu are semn constant pe nici o vecintate a originii i atunci punctul (0,0) nu este punct de extrem local pentru f. Vom stabili n continuare o condiie suficient pentru ca un punct critic s fie punct de extrem local. n acest scop vom stabili mai nti :
Lema 6.6.1. Fie : Rn R o form ptratic pozitiv definit, adic
(x) = aij xi x j , () x = ( x1, x 2 ,..., xn ) Rn , aijR, i,j 1,n i ( x ) > 0, () xRn, x 0 .
i, j =1 n

Atunci exist o constant m >0 astfel nct ( x ) m x , ()xRn.


Demonstraie. Fie S = {xRn: ||x||=1}.

Cum S este nchis i mrginit n Rn, este compact i din teorema lui Weierstrass, cum este continu pe S este mrginit inferior pe S i i atinge marginea inferioar. Fie aS astfel nct (a) = m = inf {( x ) : x S} Cum aS avem a 0 i atunci m = (a) > 0 iar pentru orice y S avem
( y ) m . Dac xRn, x 0 , atunci y =
2

x x n S i deci m, () xR , x 0 x x

de unde ( x ) m x , () xRn, x 0 i cum aceast inegalitate este verificat i pentru x = 0, demonstraia este ncheiat.

- 129 -

Teorema 6.6.2. Fie A Rn , deschis, f:AR,f C2 ( A ) i a A punct critic

pentru f. Dac forma ptratic d2f(a) este


a) pozitiv definit atunci a punct de minim local; b) negativ definit (adic d2f(a) este pozitiv definit) atunci a este punct

de maxim local.
Demonstraie. Presupunem mai nti c d2f(a) este pozitiv definit. Cum
2f fC2(A) rezult c matricea asociat (a ) xix j 1i, jn

este simetric.

Aplicnd lema 6.6.1 formei ptratice d2f(a) rezult c exist m>0 astfel nct
d2 f (a)( x ) m x , ()x Rn i atunci d2 f (a)( x a) m x a , ()x Rn .
2 2

Pe de alt parte, cum fC2(A), din formula lui Taylor cu rest de ordin k = 2 sub forma Lagrange, pentru orice xB(a,r) A, exist un punct ntre a i x astfel nct f(x) = f(a)+ df (a)( x a) +
1 1 ! 1 2 d f ()( x a) . 2!

Cum a este punct critic avem df(a) = 0 i atunci f(x) - f(a) = d2 f ()( x a) =
1 2
1 2 1 d f (a)( x a) + d2 f ()( x a) d2 f (a)( x a) 2 2

m 1 2 d f ()(x a) d2f (a)(x a) , () xB(a,r). || x a ||2 + 2 2

d2 f ()( x a) d2 f (a)( x a) , daca x a, x B(a, r ) 2 Fie ( x ) = xa , daca x = a 0

Cum fC2(A) avem lim ( x ) = 0 i atunci


x a

xa ( x ) m 2 2 f ( x ) f (a ) xa + xa = (m + ( x )), 2 2 2

Cum m > 0 i lim ( x ) = 0 , exist r1 > 0, r1 < r astfel nct


x a

m + ( x ) > 0, ()x B(a, r1) B(a, r ) .

- 130 -

n concluzie f(x) - f(a) 0, () xB(a,r1), adic a este un punct de minim local. Dac d2f(a) este negativ definit atunci folosim prima parte a demonstraiei. -d2f(a) este pozitiv definit i

Teorema 6.6.3. Fie A R 2 , deschis, fC2(A), f = f(x,y), aA, a =(x0,y0) un

punct critic pentru f. Fie r0 = Dac


1) r0 t 0 s2 > 0 i r0 > 0 atunci (x0,y0) este punct de minim local;
0

2f 2f 2f ( x 0 , y 0 ), s0 = ( x 0 , y 0 ), t 0 = 2 ( x 0 , y 0 ), (notaiile lui de Monge). x 2 xy y

2) r0 t 0 s2 > 0 i r0 < 0 atunci (x0,y0) este punct de maxim local;


0

3) r0 t 0 s2 < 0 atunci (x0,y0) nu este punct de extrem local.


0

Demonstraie. Pentru (x,y)A avem


2f 2f 2 d f(x 0 , y 0 )(x x 0 , y y 0 ) = 2 (x 0 , y 0 )(x x 0 ) + 2 (x 0 , y 0 )(x x 0 )(y y 0 ) + x xy
2

x x 2 2f x x0 2 2 0 + 2s0 + 2 (x 0 , y 0 )(y y 0 ) = (y y 0 ) r0 + t 0 , pentru y y 0 . y y y0 y y0

Dac notm cu u =

x x0 atunci se observ c d2f(x0, y0) are un semn y y0

constant dac i numai dac trinomul r0u2+2s0u+t0 are rdcini complexe, adic
2 dac r0t0 - s0 > 0 .

n acest caz semnul su este dat de semnul lui r0. n consecin, d2f(a) este pozitiv definit dac r0>0 i negativ definit dac r0 < 0. Conform teoremei 6.6.2, pentru r0 > 0 punctul (x0,y0) este punct de minim local iar pentru r0 < 0, punctul (x0,y0) este punct de maxim local. Dac r0t0 - s02 < 0 atunci d2f(x0,y0) nu mai pstreaz semn constant i atunci diferena f(x,y) - f(x0,y0) nu are semn constant pe nici o vecintate a punctului (x0,y0), deci (x0,y0) nu este punct de extrem.

- 131 -

Observaie. Dac r0t0 - s02 = 0 nu putem preciza dac punctul (x0, y0) este

sau nu punct de extrem. n acest caz semnul diferenei f(x,y) - f(x0, y0) depinde de semnul diferenelor de ordin superior.
Exemplu. S se determine punctele de extrem local ale funciei

f:A R 2 R, f ( x, y ) = x 2 + xy + y 2 4 ln x 10 ln y . Determinm mai nti punctele critice:


f x = 0 f =0 y 4 2x + y x = 0 , care are soluia (x0,y0) = (1,2) deci punctul (1,2) este 10 x + 2y =0 y

punct critic. Pentru a vedea dac este punct de extrem local s calculm derivatele pariale de ordinul doi. Evident avem:
2f 4 2f = 2 + 2 r0 = 2 (1,2) = 6; x 2 x x 2 2 f f = 1 s0 = (1,2) = 1; xy xy 2f 10 2f 10 9 = 2 + 2 t 0 = 2 (1,2) = 2 + = . 2 y y y 4 2

Prin urmare r0t0 - s02 = 6 1 = 26 > 0 i cum r0 > 0 rezult c (1,2) este punct de minim local.
Teorema 6.6.4. Fie A Rn, deschis, f : A R, f C2 (A ) i a A punct

9 2

critic pentru f. Fie aij =


2f (a ) = a ji, 1 i, j n . x ix j
a11 a12 ... a1n

Fie 1 = a11, 2 =

a11 a12 a21 a22

, ..., n =

a21 a22 ... a2n ... .... ... .... an1 an2 ... ann

- 132 -

Atunci:
1) Dac 1 > 0 , 2 > 0, ..., n > 0 , rezult c punctul a este punct de

minim local;
2) Dac 1 < 0, 2 > 0, ..., (- 1)n n > 0 , rezult c punctul a este punct de

maxim local.
Demonstraie. Se folosete urmtorul rezultat algebric (condiiile lui

Sylvester):

Fie o form ptratic a crei matrice asociat (aij )1i, jn este simetric.
a11 a12 ... a1n a11 a12 a21 a22 a21 a22 ... a2n ... .... ... .... an1 an2 ... ann

Fie

1 = a11, 2 =

, ..., n =

minorii

principali

ai

matricei. Atunci:
1) Dac 1 > 0, 2 > 0, ..., n > 0, rezult c este pozitiv definit; 2) Dac 1 < 0, 2 > 0, ..., (- 1)n n > 0, rezult c este negativ definit. Exemplu. S se determine punctele de extrem local ale funciei cu

valorile f (x, y, z ) =

1 x y z + + + , x, y, z > 0 . x y z 16

Determinm mai nti punctele critice:


f x = 0 f =0 y f =0 z 1 1 - x 2 + y = 0 x 1 2 + = 0 , care are soluia x = 2, y = 4, z = 8, deci z y y 1 =0 2 + 16 z

(x0, y0, z0) = (2, 4, 8) este punct critic. Pentru a vedea dac este punct de extrem local s calculm derivatele pariale de ordinul doi.

- 133 -

Evident avem
2f x 2 = 2 x , 3 2f y 2 = 2x y , 3 2f z 2 = 2y 2 f 1 2f 2f 1 = 2, = 0, = 2 3 xy xz yz z y z

i matricea hessian devine


2f 2 (2, 4, 8 ) x 2 f (2, 4, 8) H(2, 4, 8 ) = yx 2 f (2, 4, 8 ) zx 2f (2, 4, 8) xy 2f (2, 4, 8) y 2 2f (2, 4, 8) yz 2f (2, 4, 8) 1 1 0 xz 16 4 1 1 1 2f (2, 4, 8) = . 64 yz 16 16 1 1 2f (2, 4, 8) 0 64 64 z 2

Observm c 1 =

3 1 1 > 0, 2 = > 0, 3 = > 0, deci 4 256 2 642

(x0, y0, z0) = (2, 4, 8) este punct de minim local.


Extreme condiionate. Metoda multiplicatorilor lui Lagrange

Fie f : A Rn R, g1, g2 , ..., gk : A R, k < n i B A mulimea soluiilor sistemului


g1(x1, x 2, ..., xn ) = 0 g2 (x1, x 2 , ..., xn ) = 0 .......... .......... .......... gk (x1, x 2 , ..., xn ) = 0

(6.25)

Ne propunem s determinm extremele funciei f care s verifice n plus condiiile (6.25), numite legturi.
Definiia 6.6.2. Un punct a B se numete punct de extrem local al

funciei f cu restriciile (6.25) sau punct de extrem local condiionat dac exist o vecintate V (a ) astfel nct diferena f (x ) f (a ) are semn constant pe V B . Cu alte cuvinte punctul a este punct de extrem condiionat pentru f dac este punct de extrem pentru restricia lui f la B, deci pentru fB.

- 134 -

Definiia 6.6.3. Un punct a B se numete punct critic condiionat pentru f

dac este punct critic pentru fB.. Pentru determinarea extremelor condiionate vom folosi metoda multiplicatorilor a lui Lagrange.
Teorema 6.6.5. (Existena multiplicatorilor lui Lagrange).

Fie A Rn , deschis, f, g1, g2, ..., gk C1(A ) i a A punct de extrem local pentru f cu restriciile (6.25). Dac D(g1, g2,..., gk ) (a) 0 atunci exist k numere reale 1, 2,..., k astfel D(x1, x 2 ,..., xk )

nct dac se consider funcia L = f + 1g1 + 2g2 + ... + k gk , s avem


L x (a ) = 0, i = 1, n , adic punctul a s fie punct critic al funciei L. i g (a ) = 0, i = 1, k i

Demonstraie. Cum

D(g1, g2,..., gk ) (a) 0 conform teoremei 6.5.2 sistemul D(x1, x 2 ,..., xk )

(6.25) poate fi rezolvat n raport cu variabilele x1, x2, ..., xk pe o vecintate a punctului a = (a1, a2, ..., ak, ak+1, ..., an), deci exist o vecintate U0 a punctului (ak+1, ..., an) n Rn k , o vecintate V0 a punctului (a1, a2, ..., ak) n Rk astfel nct
V0 xU0 B i sistemul (6.25) are soluie unic (x1, x2, ..., xk) n V0:
x1 = 1(x k +1,..., x n ) x 2 = 2 (x k +1,..., x n ) .......... .......... .......... x k = k (x k +1,..., x n )

(6.26)

Avem a1 = 1(ak +1,..., an ), a2 = 2 (ak +1,..., an ),..., ak = k (ak +1,..., an ) . Funciile 1, 2 ,..., k sunt de clas C1 pe U0.

- 135 -

nlocuind x1, x2, ..., xk din (6.26) n (6.25) obinem:


g1(1(xk +1,..., xn ),..., k (xk +1,..., xn ), xk +1,..., x n ) = 0 g2 (1(xk +1,..., xn ),..., k (xk +1,..., x n ), xk +1,..., xn ) = 0 .......... .......... .......... .......... .......... .......... .......... ........ gk (1(xk +1,..., xn ),..., k (xk +1,..., xn ), xk +1,..., xn ) = 0

(6.27)

()(xk +1, xk + 2,..., xn ) U0 .


Din (6.27), folosind teorema de derivare a funciilor compuse obinem:

xm x
j =1 j

j
i

gm = 0, m = 1, k, i = k + 1, n xi

(6.28)

Considerm funcia cu valorile F(xk +1,..., xn ) = f (1(xk +1,..., xn ),..., k (xk +1,..., xn ), xk +1,..., xn ) . Cum punctul a este punct de extrem local pentru f, cu restriciile (6.25) iar

(1(xk +1,..., xn ),..., k (xk +1,..., xn ), xk +1,..., xn ) B


de extrem local pentru F. Folosind teorema lui Fermat rezult c:

rezult c (ak +1,..., an ) este punct

F (ak +1,..., an ) = 0 , () i = k + 1, n , adic xi


k f (a) + f (a) j (ak +1,..., an ) = 0 , i = k + 1, n xi xi j =1 x j

(6.29)

ntruct

D(g1, g2,..., gk ) (a) 0 D(x1, x 2 ,..., xk )

rezult c sistemul liniar


f (a) , j = 1, k x j (6.30)

m =1

xm (a)
j

m =

are soluie unic. Fie (1, 2 ,..., k ) aceast soluie i L = f + Vom arta c L satisface concluziile teoremei. Cum a B rezult c gi (a ) = 0 , () i = 1, k .

m =1

mgm .

- 136 -

Pentru j 1,k vom avea obinem (innd cont c

k L (a ) = f (a ) + m gm (a ) xj xj x i m =1

i folosind (6.30) a sistemului)

1, 2 ,..., m

este

soluie

L (a ) = 0, () j = 1,k . xj

Pentru i k + 1, n , folosind (6.28) vom avea


k k k L (a) = f (a) + m gm (a) = f (a) m gm (a) j (ak +1,..., an ) = xi xi xi xi xi m =1 m =1 j =1 x j

K k f (a) j (ak +1,..., an ) m gm (a) . xi xj J =1 Xi m =1

Cum (1, 2 ,..., k ) este soluue a sistemului (6.30) va rezulta c


k L (a) = f (a) + j (ak +1,..., an ) f (a) i folosind (6.29) obinem xj xi xi j =1 x i

L (a ) = 0 , ( ) i = k + 1, n i demonstraia este ncheiat. x i

Observaie. Teorema 6.6.5 se poate enuna i astfel: Orice punct de

extrem condiionat este punct critic condiionat. Practic, pentru rezolvarea unei probleme de extrem condiionat se procedeaz astfel:
1. Se asociaz funcia lui Lagrange
L = f + 1g1 + 2g2 + ... + k gk , cu 1, ,2 ..., k R,

nedeterminai (numii multiplicatorii lui Lagrange).


2. Se determin punctele critice ale lui L, adic soluiile sistemului:
L = 0 ,1 i n xi g = 0 , 1 i k i

3. Dac (a1, a2,...an, 1,..., k ) este o soluie a acestui sistem atunci punctul

a = (a1, a2, ..., an) este punct critic condiionat al funciei f.

- 137 -

4. Presupunem c f, g1, g2, ..., gk C2 (A ) .

n acest caz f (x ) f (a ) = L(x ) L(a ) , () x B i pentru a studia dac a este punct de extrem local condiionat vom studia semnul diferenei L(x) - L(a), calculnd d2L(a), n care difereniem legturile (6.25) n punctul a. Vom gsi n acest punct dx1, dx2, ..., dxk n funcie de dxk+1, ..., dxn.
Aplicaie. S se construiasc un rezervor n form de paralelipiped drept,

de volum maxim avnd la dispoziie 48 m2 de tabl (presupunem rezervorul neacoperit). Fie x > 0 lungimea, y > 0 limea i z > 0 nlimea paralelipipedului. Atunci volumul paralelipipedului este V = xyz i xy+2xz+2yz = 48. Problema se reduce la determinarea maximului funciei f cu valorile
f (x, y, z ) = xyz, x > 0, y > 0, z > 0 , cu legtura xy + 2xz + 2yz 48 = 0 g(x, y, z ) = 0 ,

unde g(x, y, z ) = xy + 2xz + 2yz 48 . Fie L(x, y, z, ) = f (x, y, z ) + g(x, y, z ) = xyz + (xy + 2xz + 2yz 48 ) . Determinm punctele critice ale lui L:
L x = 0 L = 0 y L =0 z g = 0 yz + y + 2z = 0 xz + x + 2z = 0 , care admite soluia x = y = 4, z = 2, = 1, xy + 2x + 2y = 0 xy + 2xz + 2yz 48 = 0

deci punctul (4,4,2) este punct critic condiionat. Pentru = 1 , funcia lui Lagrange devine L(x,y,z) = xyz-xy-2xz-2yz+48. Vom calcula d2L(4,4,2). Evident avem
2L x 2 = 2L y 2 = 2L z 2 = 0, 2L 2L 2L = z 1, = y 2, = x 2 i atunci xy xy yz

- 138 d2L(4,4,2 ) =
2 2 2L (4,4,2)dx 2 + L (4,4,2)dy 2 + L (4,4,2)dz2 + x 2 y 2 z 2

2L 2L 2L (4,4,2)dxdy + 2 (4,4,2)dxdz + 2 (4,4,2)dydz = +2 xy xz yz = 2dxdy + 4dydz + 4dzdx.

Din g(x,y,z) = 0 rezult dg(x,y,z) = 0 i n particular dg(4,4,2)=0, adic 8dx+8dy+16dz = 0, de unde dx = -dy -2dz. n concluzie,
d2L(4,4,2) = 2dy (dy + 2dz ) + 4dydz 4dz(dy + 2dz ) = 2dy 2 4dydz 4dz 2 < 0 ,

deci d2L(4,4,2) este negativ definit i atunci punctul (4,4,2) este punct de maxim local.

Probleme propuse
x2y , daca ( x, y ) (0,0) . f(x, y) = x 6 + y 2 0 , daca ( x, y ) = (0,0)

1. Fie funcia f : R2 R,

S se arate c funcia f nu este continu n punctul (0,0) dar este derivabil n (0,0) dup orice versor s = (s1, s2 ) R2.
2. Fie funcia f : R2 R , f(x,y) = cos(2x+3y).

S se calculeze

df 2 ,0 , unde s R este un versor. ds 4


2

xy , daca (x, y) (0,0) 2 3. Fie funcia f : R R , f(x,y)= x + y 2 . daca (x, y) = (0,0) 0,

S se arate c
a) f este continu i derivabil parial pe R 2 ; b) f nu este difereniabil n punctul (0, 0).

- 139 1 2 2 , daca (x, y) (0,0) ( x + y ) cos 2 4. Fie funcia f : D R R , f(x,y)= . x + y2 0 , daca (x, y) = (0,0)
2

S se arate c
a) f este derivabil parial pe R2 iar derivatele pariale nu sunt continue n

punctul (0,0);
b) f este difereniabil n punctul (0, 0). 5. Fie funcia f : D R2 R , f(x,y) =arctg
x . y

a) S se calculeze derivatele pariale de ordinul nti i doi; b) S se calculeze df(-1,2), df(-1,2)(3,-2), d 2 f(-1,2), d 2 f(-1,2)(3,-2).
6. Fie funcia f : D R3 R 2 ,
f ( x, y, z ) = e
x2 +3y + z

. 2 2 x +y 3x

S se determine Jf(2,-1,0).
7. Fie funcia f:R 3 \{(a,b,c)} R,
f ( x, y, z ) = 1

(x a )
+ 2f

+ (y b ) + (z c )
2

, unde a,b,c R.

S se arate c

2f x 2

y 2

2f z 2

= 0.
xy . x+y

8. Fie funcia f:D R 2 R, f ( x, y ) =

S se calculeze

m +nf , unde m,nN . m n x y

9. Fie funcia f : R 2 R,
2 x2 y ln1 + 2 , daca y 0 f ( x, y ) = y 0 , daca y = 0 2f 2f (0,0) = S se arate c f C (R ), (0,0) dar nu este verificat xy yx
1

criteriul lui Schwarz.

- 140 -

10. Fie funcia u : D R 2 R,

u(x, t) = f(x + at) + g(x - at), a R, unde f i g sunt dou funcii de dou ori difereniabile. S se arate c
2u t2 a2 2u x2 = 0.
xy y z ln x + x , , unde : R 2 R z x x

11. Fie funcia f :D R 3 R, f ( x, y, z) =

este difereniabil. S se arate c


x f xy f f +y +z = +f. x y z z

12. S se arate c urmtoarele funcii sunt omogene i s se verifice

relaia lui Euler:


x z a) f:D R 3 R, f ( x, y, z) = ; 3y
y

b)f:D R 2 R, difereniabil.

y f ( x, y ) = x n ,unde x

n N i

este

funcie

x 13. Fie funcia f:D R 2 R, f ( x, y ) = x 2 y 2 , ,unde y


C2 ( ) .

: R 2 R,

S se calculeze df ( x, y ) i d2 f ( x, y ) ,unde (x, y ) D .


14. Fie funcia f:D R 2 R, f ( x, y ) = arctg
x+y . S se aproximeze funcia f 1 xy

printr-un polinom de grad doi n vecintatea punctului (1,-1).


15. S se studieze extremele locale ale funciilor: a) f :R 2 R, f ( x, y ) = x 2 + y 2 e 2 x + 3 y ; b) f :R2 \ {(0,0)} R, f ( x, y ) = xy ln x 2 + y 2 ; c) f : (0, ) (0, ) R, f ( x, y ) = sin x sin y sin(x + y ) ;

- 141 -

d) f :D R2 R, f(x,y) = x-2y + ln x 2 + y 2 + 3arctg ; e) f :R 3 R, f ( x, y, z ) = x 3 + y 2 + z 2 + 12xy + 2z . 16. S se arate c funcia f :R 2 R, f ( x, y ) = 1 + e y cos x ye y are o

y x

infinitate de puncte de maxim local i nici un punct de minim local.


17. S se arate ca ecuaia
xez = xy + z definete funcia z pe o

vecintate a punctului (2,1, 0). S se calculeze dz(2,1) i


2z (2,1) . xy

18. S se calculeze y,y, dac funcia y este definit de ecuaia


x ln x 2 + y 2 = arctg . y

19. Ecuaia F(x, x+z, y+z) = 0, unde F:D R 3 R,F C2 (D ) ,definete

funcia z.
2z . S se calculeze dz i xy

1 2 2 2 x + y = z 20. S se arate c sistemul definete n mod implicit 2 x + y + z = 2

funciile x i y pe o vecintate a punctului (1,-1,2). S se calculeze x(2); y(2); x(2); y(2).


21. Dac funciile u i v sunt definite de sistemul
uv = ax + by + cz 2 v = x 2 + y 2 + z 2

,a, b, c R , atunci x

u u u +y +z = 0. x y z

g( x + u, y v ) = 1 22. Sistemul xu + yv = 1

,unde g:D R2 R , g C1(D) definete

funciile u i v. S se calculeze du, dv.

- 142 -

23. S se studieze punctele de extrem local ale funciei z definit de

ecuaia x2+y2+z2+2x+y-4z =

15 . 4

24. S se determine punctele de extrem ale funciei f:R3 R,

f(x, y, z) = xy2z3 cu legtura x + 2y + 3z = a (x > 0, y > 0, z > 0, a > 0).


25. S se determine punctele de extrem ale funciei f : R3 R,
xy + xz + yz = 8 f(x, y, z) = xyz, cu legturile . x + y + z = 5

- 143 -

CAPITOLUL 7

INTEGRALA RIEMANN
7.1. Primitiva unei funcii reale de variabil real
Definiia 7.1.1. Fie I R un interval i funcia f : I R. Spunem c f admite primitiv pe I dac exist o funcie F : I R, derivabil pe I astfel nct F'(x) = f(x), () xI. Funcia F se numete primitiv a funciei f. Proprieti ale primitivelor Propoziia 7.1.1. Dac F : I R este o primitiv a funciei f pe I atunci F + c, unde c R, este o primitiv a lui f pe I. Demonstraie. Fie c R i G : I R, G = F + c. Vom avea G' = (F + c)' = F' = f.

Propoziia 7.1.2. Dac F,G : I R sunt dou primitive ale lui f pe I atunci acestea difer printr-o constant. Demonstraie. Cum F' = G' = f rezult (F - G)' = F' - G' = 0, deci F G este o constant.

Observaie. Dac I nu este interval, concluzia propoziiei 7.1.2 nu este n general adevrat. n acest sens considerm funcia f : (0,1) (2,3) R, f(x) = 1, () x (0,1) (2,3) i
x + 1, daca x (0,1) F, G : (0,1) (2,3) R, F(x) = x , daca x (2,3 )

- 144 -

x , daca x (0,1) G(x) = x + 1, daca x (2,3 )

Atunci F, G sunt derivabile pe (0,1) (2,3), F' = G' = f dar F i G nu difer printr-o constant. Observaie. Din propoziia 7.1.2 rezult c, dac F : I R este o primitiv a funciei f pe I atunci mulimea primitivelor funciei f pe I este format din funciile de forma F + c, unde c R. Mulimea {F + c: c R} a tuturor primitivelor lui f pe I se numete integrala nedefinit a lui f pe I i se noteaz cu f ( x )dx (sau fdx, Vom scrie f ( x )dx = F(x) + C. Propoziia 7.1.3. Dac f i g au primitive pe un interval I atunci funcia f+g are primitive pe I i ( f + g)( x )dx = f ( x )dx + g( x )dx . Demonstraie. Fie F : I R o primitiv a lui f i G : I R o primitiv a lui g. Atunci vom avea (F + G)' = F' + G' = f + g , deci F + G este o primitiv a lui f + g i demonstraia este ncheiat.

f ).

Propoziia. 7.1.4. Dac f are primitive pe un interval I i R atunci f are primitive pe I iar pentru 0 avem

f ( x )dx

= f ( x )dx .

Demonstraie. Fie F : I R o primitiv a lui f i R. Vom avea (F)' = F' = f, deci F este o primitiv a lui f. Dac = 0 atunci nu este adevrat.

f ( x)dx

0dx

= C iar f ( x )dx = 0, deci egalitatea

Propoziia 7.1.5. Dac f are primitive pe un interval I R atunci f are proprietatea lui Darboux. Demonstraie. Fie F : I R o primitiv a lui f. Atunci F este derivabil pe I i din teorema lui Darboux derivata sa F' = f are proprietatea lui Darboux.

- 145 -

Propoziia 7.1.6. (de existen a primitivelor). Fie f : I R o funcie continu. Atunci f are primitive pe I.

Metode de determinare a primitivelor


Propoziia 7.1.7. (metoda integrrii prin pri) Fie f, g : I R, derivabile cu derivate continue pe I. Atunci

f ( x)g '(x)dx = f(x)g(x) - f '(x)g(x)dx.


Demonstraie. Cum f ' i g' sunt continue rezult fg' i f 'g sunt continue, deci au primitive . Cum (fg)' = f 'g + f g rezult c funcia fg este o primitiv a funciei f 'g + fg', deci

(f '(x)g(x) + f(x)g'(x))dx = f(x)g(x) + C, de unde f ( x )g '(x)dx = f(x)g(x) - f '(x)g(x)dx + C = f(x)g(x) - f '(x)g(x)dx.


Propoziia 7.1.8. (prima metod de schimbare de variabil). Fie I, J R intervale i funciile f : I R, : J I cu proprietile i) ii) este derivabil pe J; f are primitive pe I i F : I R este o primitiv a sa.

Atunci funcia (f)' are primitive pe J i F este o primitiv a sa, adic

f((t)) '(t)dt = F((t)) + C.


Demonstraie. Cum F este derivabil pe J i (F)' = (F')' = (f)' demonstraia este ncheiat. Propoziia 7.1.9. (a doua metod de schimbare de variabil) Fie I, J R intervale i funciile f : I R, : J I, cu proprietile: i) ii) este bijectiv, derivabil cu ' (t) 0 () t J; funcia (f) ' are primitive pe J i G : J R este o primitiv a sa. Atunci f are primitive pe I i G-1 este o primitiv a sa, adic

f ( x)dx

= G-1 + C.

- 146 -

Demonstraie. Din i) rezult c -1 este derivabil pe I i cum G este derivabil pe J rezult c G -1 este derivabil pe I i (G -1)' (x) = G' (-1(x))(-1)' (x) = (f)(-1(x))'(-1(x))(-1)'(x) = = f(x)'(-1(x))
1 = f(x), () xI. ' ( 1 ( x ))

n concluzie funcia G-1 este o primitiv a lui f.

7.1.1.Primitivele funciilor raionale


Vom da o metod de calcul a primitivelor de forma f ( x )dx , xI, unde I R este un interval iar f este o funcie raional, adic f = P, Q R[X], Q(x) 0, () xI. Dac grad P grad Q atunci grad R < grad Q i astfel
P( x ) R( x ) = C(x) + , unde C, R R[X], Q( x ) Q( x )

P , unde Q

f ( x)dx

C(x ) dx + Q( x )dx
C(x ) dx
se calculeaz imediat i atunci
R( x )

R( x )

Cum C este un polinom integrala

integrala f ( x )dx se reduce la calculul integralei

Q( x ) dx.

Astfel, este suficient s studiem cazul n care grad P < grad Q. Definiia. 7.1.2. Se numesc fracii simple funciile cu valorile de forma:
Bx + C A , , unde xI, x , m 2 ( x ) (ax + bx + c )n

A, B, C, , a, b, c R,

m, n N* ,

b2 4ac < 0.
P , Q

Conform unui rezultat cunoscut din algebr orice funcie raional f = cu grad P < grad Q se descompune n mod unic ca o sum de fracii simple.

- 147 -

Exemplu.
x3 C Fx + G Dx + E A B = + + + 2 + 2 , 2 2 2 2 x 1 2x + 1 x +1 (2x + 1)( x 1) ( x + 1) ( x 1) ( x + 1) 2

unde A, B, C, D, E, F, G R se determin prin identificarea coeficienilor dup ce aducem la acelai numitor. Astfel este suficient s calculm primitivele fraciilor simple. Evident avem
( x ) m + 1 + C , pentru m 1 A A ( x )m dx = m + 1 A ln x + C , pentru m = 1

(ax

bB Bx + C B 2ax + b dx = (ax 2 + bx + c )n dx + (C - 2a ) n 2a + bx + c )

(ax

1 dx + bx + c ) n

Fie In =

(ax

2ax + b dx, Jn = + bx + c ) n

(ax

1 dx + bx + c ) n

Evident avem:
(ax 2 + bx + c )' dx = In = 2 n (ax + bx + c )
(ax 2 + bx + c )-n +1 + C , dac n 1 n +1 2 ln ax + bx + c + C , dac n = 1

Pentru Jn avem: Jn =
1 an

1
n 2

c b x + x + a a

dx =

1 an

1
2 n

b 4ac b 2 + x + 2a 4a 2

dx .

Folosind schimbarea de variabil x = t 1 calculul integralei Tn = 2 dx , unde = ( t + 2 )n

b integrala Jn se reduce la 2a

4ac b 2 > 0. 2a

Pentru n = 1 rezult T1 = Tn =

1 t arctg + C , iar pentru n 2,

1 2 + t 2 t 2 1 1 1 t2 dt = 2 2 dt - 2 2 dt = 2 ( t 2 + 2 )n ( t + 2 )n1 ( t + 2 )n

- 148 1 1 = 2 Tn 1 2 2 ( t 2 + 2 ) n + 1 t n + 1 dt =
'

1 1 = 2 Tn1 2 2

( t 2 + 2 ) n+1 ( t 2 + 2 ) n+1 t n + 1 n + 1 dt =

1 1 t 1 2 2 Tn1 + 2 Tn1, deci 2 2 n 1 2 (n 1) ( t + ) 2 (n 1)

Tn =

1 1 t 2n 3 2 + Tn1 , () n 2. 2 2 n 1 2 n 1 ( t + ) n 1

Exemplu. S se calculeze integrala: I=

( x + 1)( x 1)

x+2
2

( x 2 + 1)

dx, x I R \ {1}.

Din descompunerea n fracii simple


x+2 A B C Dx + E = + + + 2 , gsim 2 2 2 ( x + 1)( x 1) ( x + 1) x + 1 x 1 ( x 1) x +1

A=

1 7 3 3 1 , B = - , C = , D = , E = , i atunci 8 8 4 4 4 I= 1 dx 7 dx 3 dx 1 3x + 1 x + 1 8 x 1 + 4 ( x 1)2 + 4 x2 + 1dx = 8 1 1 7 3 1 3 = ln x + 1 ln x 1 + ln( x 2 + 1) + arctgx + C. 8 8 4 x 1 8 4

7.1.2. Primitive de funcii iraionale


Vom prezenta cteva tipuri de integrale de funcii iraionale care prin schimbri convenabile de variabil se reduc la integrale de funcii raionale.
m ax + b m ax + b m ax + b n n n ) ,( ) ,..., ( ) A. R x, ( cx + d cx + d cx + d
1 2 1 2 k k

dx, xI, unde

R este o funcie raional, cx + d 0, () xI, mi,niZ, ni>0, (mi, ni) = 1, i = 1, k . n acest caz efectum schimbarea de variabil dat de unde s = c.m.m.m.c. (n1,n2, ..., nk).
ax + b s =t , cx + d

- 149 -

Obinem x =

s(ad bc ) t s 1 dt s b = ( t ), '(t) = a ct s (a ct s ) 2

i atunci integrala se transform n


m s dt s b m s s(ad bc )t s 1 n n , t ,..., t dt = R1 ( t )dt, I = R a ct s (a ct s )2
1 k 1 k

unde R1 este o funcie raional. Exemplu. S se calculeze integrala Aceast integral este de forma:
1 x +1 2 x + 1 2 x, I = R = t obinem dx i folosind schimbarea de variabil dat de x 1 x 1

1 x +1 dx , x >1. x 1

x = ( t ) , unde ( t ) =

4t t2 + 1 i ' ( t ) = 2 . 2 t 1 ( t 1) 2

Integrala dat se transform n


J=

4t 1 t2 1 t2 1 + 2 t 2 dt = 4 2 dt = 2 2 dt = 2 2 2 t + 1 (t 1) (t + 1)(t 1) t + 1 t 1

= 2arctgt 2

1 t 1 ln + C, 2 t +1
x +1 - ln x 1

de unde I = -2arctg

x +1 x 1 x +1 + x 1

+ C.

B.

R (x,

ax 2 + bx + c dx,

xI, unde R este o funcie raional i

ax2+bx+c 0, () xI. Aceast integral se reduce la o integral dintr-o funcie raional folosind substituiile lui Euler i anume: i) ii) dac a > 0:
ax 2 + bx + c = t x a ;

dac = b 2 4ac > 0 :

t( x x 1 ) ax 2 + bx + c = sau t( x x ) , 2

unde x1, x2 sunt rdcinile ecuaiei ax2 + bx + c = 0;

- 150 -

iii)

dac c > 0:

ax 2 + bx + c = tx c .

Sunt cazuri particulare n care se pot folosi i alte schimbri de variabile care conduc la calcule mai simple i anume: Caz 1. Pentru integrala R x, x 2 + a 2 dx , cu R funcie raional se poate folosi schimbarea x = a tgt sau x = a sht. Caz 2. Pentru integrala R x, x 2 a 2 dx , se poate folosi schimbarea x = a cht. Caz 3. Pentru integrala R x, a 2 x 2 dx , se poate folosi schimbarea x = a sint sau x = a cost. C.

( (

) )

x (ax
m

+ b pdx, xI, m,n,pQ, numit i integral binom.

Matematicianul rus Cebev a artat c aceast integral binom se poate reduce la o integral din funcii raionale numai ntr-unul din urmtoarele cazuri: i) Dac pZ se efectueaz schimbarea x = ts, unde s este cel mai mic multiplu comun al numitorilor lui m i n; ii) Dac pZ dar
m +1 Z se efectueaz schimbarea dat de n

axn + b = ts, unde s este numitorul lui p; iii) Dac pZ,


m +1 m +1 Z dar + p Z, se efectueaz schimbarea n n

de variabil dat de

ax n + b s = t , unde s este numitorul lui p. xn

Exemplu. S se calculeze integrala Integrala este de forma:

xdx 1+ 3 x 2

, x>0

2 2 2 1 I = x x 3 + 1 dx , deci este o integral binom cu m = 1, n = , p = - . 3 2

Evident pZ i

m +1 = 3 Z, deci suntem n cazul ii) i efectum n


2 3

schimbarea de variabil dat de x + 1 = t x = t 1 x = ( t 1)


2

2 3

3 2

- 151 -

dx =

1 1 3 2 t 1 2 2tdt = 3t t 2 1 2 dt . 2

Integrala dat se transform n J=

(t

1 2 t -13t t 2 1 2 dt = 3 t 2 1 dt = 3 t 4 2t 2 + 1 dt =

t5 = 3 - 2t 3 + 3t + C , de unde 5
3 I= 5 x
2 3

+ 1

5 2

- 2 x

2 3

+ 1

3 2

2 3 x + 1 + C . +3

7.1.3. Primitive de funcii raionale n sinus i cosinus


Acestea sunt primitive de forma

R(sinx, cosx )dx, unde R este o funcie raional i xIR.


Efectum schimbarea de variabil dat de tg xI ( , ) . Cum dx = reduce la I = R
x = t x = 2 arctgt, pentru 2

2 2t 1 t 2 dt, sinx = , cosx = , integrala dat se 1+ t 2 1+ t 2 1+ t 2


2 1 + t 2 dt = R1 ( t )dt, unde R1 este o funcie raional.

2t 1 - t 2 , 2 2 1+ t 1+ t

n anumite cazuri particulare se pot folosi i alte schimbri de variabil care conduc la calcule mai simple i anume: i) ii) iii) dac R este impar n sinus, adic dac R este impar n cosinus, adic dac R este par n sinus i cosinus, adic R(-sinx, cosx) = -R(sinx, cosx), se efectueaz schimbarea de variabil cosx = t. R(sinx, -cosx) = -R(sinx, cosx), se efectueaz schimbarea de variabil sinx = t. R(-sinx, -cosx) = R(sinx, cosx), se efectueaz schimbarea de variabil tgx = t. Exemple. 1. S se calculeze integrala I=

3 + cos x dx,

x (0, ) .

- 152 -

Folosind schimbarea tg J=

x = t integrala se transform n 2

t 1 1 1 2 + C , de unde arctg dt = dt = 2 2 2 1 t 1+ t t +2 2 2 3+ 1+ t2

I=

1 2

tg arctg

x 2 +C. 2

2. S se calculeze integrala I =

3 + cos x dx,

x (0, 2) . x = t, deoarece pentru 2

n acest caz nu mai putem folosi schimbarea tg x = nu are sens tg . Fie f : (0, 2) R, f(x) = primitive pe (0, 2 ).
2

1 . Cum f este continu pe (0, 2) are 3 + cos x

O primitiv a funciei f pe (0, ) este (conform exemplului 1) funcia G:(0, ) R, G(x) = funcia F:(0, 2) R,
pentru x (0, ) G(x), F(x) = c 1, , unde c1, c2R pentru x = G( x ) + c , pentru x ( , 2) 2

1 2

tg arctg

x 2 i atunci o primitiv a funciei f pe (0, 2) este 2

Din condiia de continuitate i derivabilitate a lui F n punctul x = obinem


c1 = 2 2 ,c2 = 2

i atunci I = F(x) + C.

3. S se calculeze integrala In = cos n xdx, x R, n N* . Vom stabili o relaie de recuren pentru In. Pentru n = 0 I0 = dx = x + C ; Pentru n = 1 I1 = cosx dx = sinx + C ;

- 153 -

Pentru n 2 In = cos n-1x (sin x )' dx = = cos n1 x sin x - (n - 1)cos n-2 x( sin x )(sin x )dx = = cos n1 x sin x + (n - 1) cos n-2 x sin 2 xdx = = cos n1 x sin x + (n - 1) cos n-2 x(1 cos 2 x )dx = = cos n1 x sin x + (n - 1)(In-2 - In ), de unde In =
cos n1 x sin x n - 1 + In2 , () n2 n n

Analog se calculeaz integrala In = sin n xdx.

7.2. Integrala definit


Fie funcia f:[a, b] R. Definiia 7.2.1. Se numete diviziune(sau divizare) a compactului [a, b] un sistem de puncte notat = (a = x 0 < x 1 < ... < x n = b) . Punctele xk, k = 0,n se numesc punctele diviziunii. Un sistem de puncte
= ( 1, 2 , ..., n ), unde

k [x k 1, x k ], k = 1, n , se

numete sistem de puncte intermediare asociat diviziunii . Fie D([a, b]) mulimea tuturor diviziunilor compactului [a, b]. Pentru D([a, b]), = (a = x 0 < x 1 < ... < x n = b) vom nota cu = max(x k - x k -1 ), k = 1, n ,

norma diviziunii . Numrul real notat ( f, k ) (sau (f, )) i definit astfel


( f , k ) = f( k )(x k - x k -1 ) se numete suma integral (sau Riemann) asociat
k =1 n

funciei f, diviziunii i punctelor intermediare 1, 2 ,..., n .


Observaie. Dac f 0 atunci
(f, k )

reprezint suma ariilor

dreptunghiurilor de baz xk - xk-1 i de nlime f( k ) (1 k n).

- 154 -

Rezult

(f, k )

aproximeaz

aria

subgraficului

lui

f,

f = {(x, y ) R 2 : a x b, 0 y f (x ) }, delimitat de axa Ox, graficul funciei f i

dreptele de ecuaii x = a i x = b. (fig.1).

Fig. 1 Dac funcia f este mrginit vom nota prin mk = Mk =


sup f ( x ) , k = 1,n i prin s (f) = inf f ( x) ,

x[ x k 1 , x k ]

x[ x k 1 , x k ]

m (x
k =1 k

x k 1 ) , S(f) =

M (x
k =1 k

x k 1 ) ,

numite i sumele Darboux inferioar i, respectiv, superioar asociate funciei f i diviziunii . S observm ca () D([a, b]), = (a = x 0 < x 1 < ... < x n = b) i () k [x k 1, x k ], k = 1, n avem s ( f ) ( f , k ) S ( f ) .
Observaie. Dac f 0 atunci s (f ) (respectiv S (f ) )reprezint suma ariilor

dreptunghiurilor de baz xk - xk-1 i nlime mk (respectiv Mk). Deci s (f ) (respectiv S (f ) ) aproximeaz prin lips (respectiv prin adaos) aria subgraficului funciei f.
Definiia 7.2.2. Funcia f este integrabil (n sens Riemann) pe [a, b] dac

exist IR astfel nct () > 0, () > 0 astfel nct () D([a, b]), = (a = x 0 < x 1 < ... < x n = b) cu < i () k [x k 1, x k ], k = 1, n avem (f , k ) I < .

- 155 -

S observm c numrul real I din definiie, n ipoteza c exist, este unic. ntr-adevr, dac ar exista dou numere reale I1, I2 cu proprietatea din definiie atunci pentru orice > 0, exist ' , '' > 0 astfel nct pentru orice diviziune D ([a, b]), = (a = x 0 < x 1 < ... < x n = b) cu < ' , < '' i orice
k [x k 1, x k ], k = 1, n avem
(f , k ) I1 < , 2 (f , k ) I2 < . 2

Lund = min(' , '' ) rezult c pentru orice diviziune D ([a, b]),


= (a = x 0 < x 1 < ... < x n = b) cu < i orice sistem de puncte intermediare

avem (f, ) I1 < ,

(f , ) I2 <

, de unde 2 + = . . 2 2

I1 I2 I1 (f , ) + (f, ) - I2 <

Cum > 0 fost arbitrar rezult I1 = I2.

Prin definiie numrul real I se numete integrala definit a funciei f pe intervalul [a, b] i se noteaz I =

a f (x )dx
b

(sau

b a

fdx ,

b a

f ).

S reamintim din liceu principalele proprieti ale funciilor integrabile:


Propoziia 7.2.1. O funcie f:[a, b] R integrabil pe [a, b] este mrginit

pe [a, b].
Propoziia 7.2.2. Funcia f : [a, b] R este integrabil pe [a, b] dac i

numai dac () I R astfel nct () (n) D ([a,b]),


n n = (a = x n < x 1 < ... < x n = b) cu 0 p
n

n 0 i () n x n 1, x n , k = 1, p n , irul k k k

sumelor Riemann ( (f , n ))nN este convergent ctre I. k


n

Propoziia 7.2.3. Dac f,g:[a, b] R sunt integrabile pe [a, b] i ,R

atunci f + g este integrabil pe [a, b] i

(f + g)(x )dx = f (x )dx + g(x )dx .


b b b a a a

- 156 -

Propoziia 7.2.4. Dac f este integrabil pe [a, c] i pe [c, b], unde

c(a, b) atunci f este integrabil pe [a, b] i

b a

f ( x )dx = f (x )dx + f (x )dx .


c b a c

Propoziia 7.2.5. Dac f este integrabil pe [a, b] i f 0 atunci

f (x )dx 0 .
b a

Consecin. Dac f g iar f i g sunt integrabile pe [a, b] atunci

b a

f (x ) dx g(x ) dx .
b a

Teorema 7.2.1. (Criteriul Darboux de integrabilitate).

Fie f:[a, b] R, mrginit. Atunci funcia f este integrabil dac i numai dac pentru orice > 0, exist > 0 astfel nct () D ([a, b]) cu < avem S(f) s(f) < .
Teorema 7.2.2. Orice funcie continu f:[a, b] R este integrabil pe

[a, b] iar dac F:[a, b] R este o primitiv a sa atunci

f (x )dx = F(x)
b a

b a

= F(b) - F(a)

(formula lui Leibniz-Newton).

Teorema 7.2.3. Fie f:[a, b] R, continu. Atunci () c(a, b) astfel nct

b a

f ( x )dx = (b - a)f(c)

(formula de medie).

Teorema 7.2.4. Dac f,g:[a, b] R sunt dou funcii derivabile cu derivate

continue atunci:

b a

f ( x )g' ( x )dx = f(x)g(x) b f ' ( x )g( x )dx (formula de integrare prin pri). a
a

Teorema 7.2.5. Fie f:[a, b] R, continu i :[c, d] [a, b] bijectiv,

derivabil cu derivat continu pe [c, d]. Atunci

b a

f ( x )dx =

1 (b ) 1 (a )

f ((t ))' (t )dt

(formula de schimbare de variabil).

- 157 -

Aplicaie. S se arate c

cosn xdx = 2 sinn xdx i s se calculeze


0

valoarea comun lor. Folosind schimbarea de variabil x =


- t pentru a doua integral obinem 2

Fie I0 = In =

2 0

0 sinn xdx = sinn t ( 1)dt = 2 cosn tdt . 0 2 2

2 0

cosn xdx . Evident avem


2 0

2 0

dx = x

= , I1 = 2

2 0

cos xdx = sinx

2 0

=1

Pentru n 2, In =

2 0

cosn 1 x(sin x ) ' dx =


2

= cos n 1 x sin x 02 (n - 1)cos n 2 x( sin x )(sin x )dx =


0

= (n-1) 2 cosn 2 x(1 cos2 x )dx = (n - 1)(In 2 In ) , de unde deducem


0

In =

n -1 In2 , () n2. n

Pentru n = 2m, mN* obinem


I2 m =

(2m 1)!! . 2m - 1 2m 1 2m 3 2m 1 2m 3 1 I2 m 2 = I2m 4 = ... = ... I0 = (2m)!! 2 2m 2m 2m 2 2m 2m 2 2


Pentru n = 2m+1, mN obinem

I2 m =

(2m)!! . 2m 2m 2m 2 2m 2m 2 2 I2m 1 = I2m 3 = ... = ... I1 = (2m + 1)!! 2m + 1 2m + 1 2m 1 2m + 1 2m 1 3


n concluzie,
(2m 1)!! * (2m)!! 2 , dac n = 2m, m N In = (2m)!! , dac n = 2m + 1, m N (2m + 1)!!

Vom deduce de aici formula lui Wallis:


2.4.6...(2n) 1 lim = . n 1.3.5...(2n 1) 2n + 1 2
2

- 158 -

Evident avem I2n+1 I2n I2n1, ()n 1, de unde 1

I2n I2n +1

I2n 1 2n + 1 . = I2n +1 2n

(2n 1)!! (2n + 1)!! = 1.3.5...(2n 1) = Dar, (2n)!! 2 (2n)!! 2.4.6...(2n) I2n+1
I2n

1 , unde (2n + 1) = 2 2 an

2.4.6...( 2n) 1 an = 1.3.5...( 2n 1) 2n + 1 .

Obinem deci 1

2n 1 2n + 1 , de unde i prin trecere an 2 an 2n 2 2n + 1 2

la limit obinem formula lui Wallis.

7.3. Aplicaii ale integralei definite


Fie f:[a, b] R+ o funcie continu i
f = {(x, y ) R 2 : a x b,0 y f (x ) }, subgraficul funciei f.

Vom arta c f are arie (ntr-un sens pe care l vom preciza ulterior) i aria f = f (x ) dx .
b a

Definiia 7.3.1. O mulime E R2 se numete elementar dac E =

UD
i=1

unde Di sunt dreptunghiuri cu laturile paralele cu axele de coordonate i


Di D j = , () i j. Prin definiie aria E =
o o

ariaD
i=1

Observaii. i)

Reprezentarea unei mulimi elementare E sub forma

E=

UD
i=1 n

nu este unic; Aria unei mulimi elementare E nu depinde de reprezentarea


i

ii)

E=

UD
i=1

iii ) Dac E, F sunt mulimi elementare cu E F = atunci

aria(EF) = ariaE + ariaF;

- 159 -

iv)

Dac E, F sunt mulimi elementare i EF atunci aria E ariaF i aria (F\E) = ariaF ariaE.

Definiia 7.3.2. Fie A R2 o mulime mrginit. Mulimea A are arie dac

exist dou iruri (En)nN, (Fn)nN de mulime elementare astfel nct


i) ii)

En A Fn, () nN; irurile de numere reale pozitive (aria En)nN i (aria Fn)nN sunt convergente i lim ariaE n = lim ariaFn
n n

Prin definiie aria A = lim ariaE n = lim ariaFn .


n n

Observaii. i) Definiia ariei lui A este independent de alegerea irurilor

(En) i (Fn);
ii) iii) iv)

Dac A, B au arie atunci A B, A B i A \ B au arie; Dac A, B au arie i A B = atunci Exist funcii al cror subgrafic nu au arie. n acest sens fie

aria (AB) = ariaA +ariaB;


1, dac x [0,1] Q funcia f:[0, 1] R, f(x) = 0, dac x [0,1] \ Q

Teorema 7.3.1. Dac f:[a, b] R+ este continu atunci f are arie i

aria f = f (x )dx .
b a
n Demonstraie. Fie ( n ) D ([a, b]) , n = (a = x n < x 1 < ... < x n = b ) astfel 0 p
n

nct n 0 . Fie mn = inf { f (x ) : x [x n 1, x n ] }, Mn = sup{ f (x ) : x [x n 1, x n ] } k = 1, p n k k k k k k , Cum f este continu () u n [x n1, x n ] i v n [x n1, x n ] astfel nct k k k k k k
m n = f un , Mn = f v n . k k k k

( )

( )

- 160 -

Fie

Dn = xn 1, xn 0, mn , k k k k

Gn k

[ = [x

n k 1

] [ ] , x ] [0, M ], k = 1, p
n k n k
pn

, dreptunghiurile de baz x n x n1 k k

i nlimile m n i respectiv Mn . k k Mulimile elementare E n = U D n , Fn = U Gn verific incluziunile k k


k =1 k =1 pn

En f Fn, () n 1 i aria En = m (x
pn k =1 n k

n k

n k 1

) = f (u )(x
pn k =1 n k

n k

x n1 = n f , u n . k k

Analog aria Fn = (f , v n ). k
n

Cum f este continu, este integrabil i din propoziia 7.2.2,


n

lim n f , un = lim n f , v n = f (x )dx , de unde va rezulta c k k


b n a b b n a a

( )

lim ariaEn = lim ariaFn = f (x )dx , deci f are arie i aria f = f (x )dx .

Corolarul 7.3.1. Dac f,g:[a, b] R sunt dou funcii continue astfel nct

f(x) g(x), ()x[a,b] atunci mulimea f ,g = {(x, y ) R 2 : a x b, f (x ) y g(x ) }, cuprins ntre graficele funciilor f, g i dreptele de ecuaii x = a, x = b are arie i
ariaf ,g = [g(x ) f (x )] dx .
b a

Exemplu. S se calculeze aria mulimii cuprinse ntre parabolele de ecuaii

y2 = x, y = x2

Fig.2

- 161 -

Rezolvnd sistemul format de cele dou ecuaii obinem punctele de intersecie dintre cele dou parabole O(0,0), M(1,1).
ariaf ,g = 3 x 2 x3 1 2 1 1 = = . x x 2 dx = 3 0 3 3 3 3 2

[g(x ) f (x )]dx =
1 0

1 0

Volumul corpurilor de rotaie


Pornind de la volumul cilindrului vom defini ce nseamn c un corp de rotaie (corp obinut prin rotirea subgraficului unei funcii pozitive f n jurul axei Ox) are volum i vom da o formul de calcul al volumului unor astfel de corpuri. Dac f:[a, b] R, f(x) = r > 0, () x [a, b] atunci corpul de rotaie este un cilindru de raz r i nlime b a. Aceast mulime se poate scrie sub forma C r = (x, y, z ) R 3 : y 2 + z 2 r, a x b . Volumul acestui cilindru este volC r = r 2 (b a ) .
Definiia 7.3.3. Fie f:[a, b] R+.

Mulimea C f = (x, y, z ) R 3 : y 2 + z 2 f (x ), a x b se numete corpul de rotaie determinat de funcia f sau corpul obinut prin rotirea subgraficului funciei f n jurul axei Ox. Vom numi mulime cilindric elementar orice mulime care se obine prin rotirea subgraficului unei funcii constante pe poriuni n jurul axei Ox.
Definiia 7.3.4. Fie f:[a, b] R+ i Cf corpul de rotaie determinat de

funcia f. Corpul Cf are volum dac exist dou iruri (En) i (Fn) de mulimi cilindrice elementare astfel nct
i)

En Cf Fn, () n N
n n

ii) lim vol E n = lim vol Fn .

n acest caz volumul lui Cf se definete prin vol Cf = lim vol E n = lim vol Fn .
n n

- 162 -

Observaii. i)

Definiia volumului corpului de rotaie Cf nu depinde de

irurile (En) i (Fn).


ii)

Exist funcii al cror corp de rotaie nu au volum. n acest sens fie

1, dac x [0,1] Q funcia f : [0, 1] R, f(x) = 0, dac x [0,1] \ Q

Analog cu teorema 7.3.1 obinem


Teorema 7.3.2. Dac f:[a, b] R+ este continu, atunci corpul de rotaie

Cf determinat de f are volum i volC f = f 2 (x )dx .


b a

Demonstraie. Analog cu demonstraia teoremei 7.3.1.

Lungimea graficului unei funcii


Fie f:[a, b] R i D ([a, b]) , = (a = x 0 < x1 < ... < x n = b ) .
Definiia 7.3.4. Se numete funcie poligonal asociat lui f i funcia
f (x k ) f (x k 1 ) (x xk 1 ), pentru x [xk -1, xk ], 1 k n . x k x k 1

f:[a, b] R, f (x ) = f (x k 1 ) +

Observaie. Funcia f se obine pe fiecare interval [x k -1, x k ] scriind ecuaia

dreptei care trece prin punctele din plan A k 1(xk 1, f (xk 1 )) i A k (xk , f (x k )) . Distana dintre punctele Ak-1 i Ak este d(Ak-1,Ak)=
n

(x k x k 1 )2 + (f (x k ) f (x k 1 ))2 i prin definiie l(f ) = d(A k -1, A k )


k =1

se numete lungimea graficului funciei poligonale f.


Definiia 7.3.5. Graficul unei funcii continue f:[a, b] R are lungime finit

dac mulimea { l(f ) : D ([a, b]) } este mrginit. n acest caz numrul real pozitiv l(f ) = sup{ l(f ) : D ([a, b]) } se numete lungimea graficului funciei f.

- 163 -

Analog cu teorema 7.3.1 se obine


Teorema 7.3.3. Dac funcia f:[a, b] R este derivabil cu derivata

continu atunci graficul lui f are lungime finit i l(f ) =

b a

1 + (f ' (x )) dx .
2

Aria suprafeelor de rotaie


Definiia 7.3.6. Fie funcia f : [a, b] R+.

Mulimea

S f = (x, y, z ) R 3 : y 2 + z 2 = f (x ), ()x [a, b]

se

numete

suprafaa de rotaie determinat de funcia f sau suprafaa obinut prin rotirea graficului funciei f n jurul axei Ox. Fie = (a = x 0 < x1 < ... < x n = b ) D ([a, b]) . Fie S(f) suprafaa de rotaie determinat de funcia f. Aria lateral a trunchiului de con Tk de raze f (x k 1 ), f (x k ) i generatoare Ak-1 Ak este (f (x k 1 ) + f (x k )) d( Ak - 1Ak) i atunci aria lateral a suprafeei S(f) este
A (f ) = (f (x k 1 ) + f (x k )) (x k x k 1 ) + (f (x k ) f (x k 1 )) .
2 2 k =1 n

Definiia 7.3.7. Fie funcia continu f:[a, b] R+.

Suprafaa de rotaie S(f) are arie dac () ( n ) D([a,b]) cu

n 0 irul

(A (f )) este convergent n R.
n

n acest caz numrul real pozitiv A(f) = lim (A (f


n

)) se numete aria lateral

a suprafeei de rotaie S(f). Dac f:[a, b] R+ este derivabil cu derivat continu se arat c suprafaa de rotaie determinat de f are arie i A(f) = 2 f (x ) 1 + (f ' (x ))2 dx .
b a

- 164 -

Probleme propuse
1. S se calculeze: a) ln xdx, x > 0 ; c) e) b) x 2 e ax dx, x R, a 0 ; d) e 2 x cos 3 xdx, x R ; f) xe 2 x sin 2 xdx, x R.

x 2 + 1dx, x R ;

1
2

ln3 xdx, x > 0 ;

2. S se determine o relaie de recuren pentru calculul integralelor: a) In = x n e x dx, xR, nN; c) In = tgn xdx, x - , , nN;
2 2

b) In = lnn xdx, x > 0, n N ; d) In =


1 dx, x (0, ) , nN. sinn x

3. S se calculeze: a) c) e)
1+ 5 ; dx, x 0, (1 + x ) 1 + x x 2 2 1

b)

dx, xR; x x +1
2

x +1

x5 x a
2 2

dx, x > a, a > 0 ;

d) x 3 x 2 + a 2 dx, xR; f) h)

2+ x x + 3 x + x +1

dx, x > 0;

13 1 x dx, x (0,1) ; 1+ x

g) x 2 x 2 + 4 x + 5dx, x (- 1,5 ) ; i)

x3 1 x 2

dx, x(-1,1);

x 1+ 3 x 2

dx, xR;

j)

1+ 4 x dx, x > 0 . x

4. S se calculeze: a) c)

1 + sin x + cos xdx, sin


3

x (0, ) ;

b)

sin

dx , x 0, ; 4 x cos x 2

sin x dx, x 0, ; 3 x + cos x 2

d) sin x sin 2x sin 3 xdx, x R ;

- 165 -

e)

sin x(2 + cos x )dx,

x (0, ) .

5. S se calculeze: a) c)

2 1

dx , a R; x a +1 e x 1dx ;

b) d)

ln(5 + 4 cos x )dx ;

ln 2 0

4 0

ln(1 + tgx )dx .

6. Fie In = (1 x 2 ) dx, n N. S se arate c In este convergent i


1 n 0 n

lim In = 0 . S se calculeze lim n In .


n

7. Interiorul elipsei de ecuaie

x2 + y 2 = 1 este desprit de hiperbola de 4

ecuaie

x2 y 2 = 1 n trei regiuni. S se calculeze aria fiecreia din ele. 2

8. S se calculeze volumul corpului de rotaie determinat de funciile: a) f : [0, 1] R, f(x) = ex +e-x b) f : [a, b] R, f (x ) =

(x a)(b x )
x

, unde a > 0 .

9. S se calculeze lungimile graficelor urmtoarelor funcii: a) f : [0,


1 ] R, f(x) = ln(1-x2); 2

b) f : [0, 1] R, f(x) = e2x 10.

S se calculeze ariile suprafeelor de rotaie determinate de ] R, f(x) = cosx; 2


x 1 x 2 . 4

funciile:
a) f : [0,

b) f : [0, 1]R, f(x)=

- 166 -

CAPITOLUL 8

INTEGRALE IMPROPRII
8.1. Integrale improprii de spea nti
Fie f : [a, ) R, integrabil pe orice compact [a, u] [a, ). Definiia 8.1.1. Funcia f este integrabil (n sens impropriu) pe [a, ) dac urmtoarea limit lim f (x ) dx exist i este finit.
u u a

n acest caz integrala

f (x ) dx
a a

este convergent i
u

f (x ) dx
n caz contrar integrala
a

= l = lim f (x ) dx .
u a

f (x ) dx

este divergent.

Analog definim

f (x ) dx , pentru f :(-, a] R.
a

Pentru f : R R, integrabil pe orice compact [a, b] R definim

f (x ) dx

= lim

a b

f (x ) dx
b a

(n ipoteza c aceast limit exist).

Prin natura unei integrale improprii a fi convergent sau divergent. Exemple.1. Fie integrala

f (x ) dx
a

nelegem proprietatea sa de

1 dx, unde a > 0, R. x

- 167 -

Fie u > a; vom avea


1 +1 a +1 , dac 1 - + 1 u u 1 a x dx = u ln , dac = 1 a

, si

lim

u a

1 +1 a , daca > 1 1 dx = 1 x + , daca 1, deci

1 dx este convergent pentru > 1 i divergent pentru 1. x

n cazul convergenei

1 1 +1 dx = a . 1 x

2. Integrala

cos 2xdx este divergent deoarece


1 sin 2x 2
u 0

u 0

cos 2xdx =

1 1 sin 2u i lim sin 2u nu exist. u 2 2

3. Integrala

e x dx este divergent deoarece


1 u

e x dx = e x

1 1 + e u i lim + e u = +. u e e

Observaie. Dac atunci

f (x ) dx
a
x

este convergent i f are primitive pe [a, )

f (x ) dx
a

= F(x )

= lim F(x ) F(a ) , unde F este o primitiv a lui f.

Teorema 8.1.1. (Criteriul lui Cauchy). Fie f:[a, ) R, integrabil pe orice compact [a, u] [a, ). O condiie necesar i suficient pentru convergena integralei

f (x ) dx
a

este ca () > 0 s existe > a astfel nct

() x' , x' ' > s avem

f (t )dt
x '' x'

< .

- 168 -

Demonstraie. Fie funcia F:[a,)R, F(x)= f (t ) dt i atunci


x a

f (t ) dt = F(x' ') F(x') .


x '' x'

Din criteriul lui Cauchy Bolzano o condiie necesar i suficient ca F s aib limit finit la + este ca () >0 s existe > a astfel nct () x,x> s avem F(x' ') F(x') < i demonstraia este ncheiat.

Definiia 8.1.2. Integrala integrala

f (x ) dx

se numete absolut convergent dac

f (x ) dx este convergent.

Propoziia 8.1.1. O integral absolut convergent este convergent. Demonstraie. Rezult imediat folosind teorema 2.1.1 i inegalitatea

f (t ) dt
x '' x'

x '' x'

f (t ) dt .

Observaie. Dac R, 0, atunci integralele au aceeai natur i n cazul convergenei

f (x ) dx
a

(f )(x ) dx

(f )(x ) dx = f (x ) dx .

De asemenea, dac f:[a, ) R, integrabil pe orice compact [a, u] [a, ) i a > a atunci integralele

f (x ) dx

f (x ) dx

a'

au aceeai natur.

Propoziia 8.1.2. (Criteriul de comparaie cu inegaliti). Fie f,g:[a, ) R, integrabile pe orice compact [a, u] [a, ). Presupunem c 0 f(x) g(x), () x a. Atunci i) Dac integrala este convergent.

g(x ) dx

este convergent rezult c i integrala

f (x ) dx

- 169 -

ii)

Dac integrala este divergent.

f (x ) dx

este divergent rezult c i integrala

g(x ) dx

Demonstraie. Fie funciile F,G:[a, ) R, F(u) =

u a

f (x ) dx , G(u) =

g(x ) dx .
u a

Cum f, g 0 rezult c F, G sunt monoton cresctoare pe [a, ) deci () lim F(u) , lim G(u) i cum f g vom avea F G.
u u

i)
u

Dac integrala

g(x ) dx
a

este convergent, conform definiiei


u

() lim G(u) i este finit. Cum F G rezult c i lim F(u) este finit, deci integrala

f (x ) dx
a

este convergent. Prin reducere la absurd, folosind i).

ii)

Propoziia 8.1.3. (Criteriul comparaiei la limit). Fie f,g:[a, ) R+, integrabile pe orice compact [a, u] [a, ). Presupunem c () lim
f (x ) = l, 0 l . Atunci x g(x )

i)

Dac l < i

g(x ) dx este convergent rezult c

f (x ) dx
a

este

convergent; ii) divergent. Demonstraie. i) Fie l < A < . Din definiia limitei () > a astfel nct
f (x ) A, () x , adic f (x ) Ag(x ), () x . g(x )

Dac l > 0 i

g(x ) dx
a

este divergent rezult c

f (x ) dx
a

este

Cum

g(x ) dx
a

este convergent rezult c

g(x ) dx

este convergent i

folosind propoziia 8.1.2 va rezulta c

f (x ) dx

este convergent i atunci

f (x) dx
a

este convergent.

ii) Fie l > B > 0. Folosim din nou definiia limitei i raionm ca la i).

- 170 -

Consecin. Fie f : [a, ) R+, a > 0, integrabil pe orice compact [a, u] [a, ). Presupunem c exist lim x f (x ) = l, 0 l . Atunci
x

i) ii)

Dac > 1 i l < rezult c Dac 1 i l > 0 rezult c

f (x ) dx
a

este convergent. este divergent.


1 i din exemplul 1. x

f (x ) dx
a

Demonstraia rezult din propoziia 8.1.3, lund g(x ) =

Exemplu. S se studieze natura integralei Avem f (x ) =


x
3

1 3 4

xdx 2x 7 + x + 1

.
4 > 1 deci 3

2x 7 + x + 1

0, () x 1 i lim x 3 f (x ) =
x

1
3

< , =

integrala este convergent. Teorema 8.1.2. (Criteriul lui Dirichlet). Fie f,g:[a, ) R, f continu, g monoton pe [a, ) i lim g(x ) = 0 .
x

Presupunem c funcia F : [a, ) R, F(u) = Atunci integrala

f (x )dx
u a

este mrginit.

f (x )g(x )dx
a

este convergent.

Demonstraie. Fie > 0 i M > 0 astfel nct F(u) M, () u a . Cum


x

lim g(x ) = 0 , exist > a astfel nct g(x )

, ( ) x . 4M

Fie u, u> , u< u. Din teorema a doua de medie () c (u, u) astfel nct

f (x )g(x ) dx = g(u') f (x ) dx + g(u' ') f (x ) dx. .


u' ' c u'' u' u' c

Cum

f (x ) dx
c u'

= F(c ) F(u') 2M , = F(u' ') F(c ) 2M ,va rezulta c

f (x ) dx
u' ' c

- 171 -

u u

f (x )g(x )dx g uI

( ) f (x )dx + g(u ) f (x )dx 4 2M + 4 2M = M M


c II uII uI c

Folosind teorema 8.1.1. rezult c integrala

f (x )g(x )dx
a

este convergent.

Observaie. Concluzia teoremei este adevrat i n cazul n care nlocuim ipoteza f continu cu ipoteza (evident mai slab) f integrabil.
sin x x

Exemplu. Fie integrala

dx .

Lund f(x) = sinx, g(x) = dat este convergent.

1 i aplicnd teorema 81.2. rezult c integrala x

Teorema 8.1.3. (Criteriul lui Abel). Fie f, g: [a, ) R , g monoton i mrginit, f integrabil pe orice compact [a, u] [a, ) i convergent. Atunci

f (x )dx este
a

f (x )g(x )dx este convergent.


a

S observm c exist o asemnare ntre criteriile de convergen de la serii de numere reale i cele de la integrale improprii. Criteriul integral al lui Cauchy stabilete o legtur ntre integralele improprii de spea nti i o anumit serie numeric ce poate fi construit cu ajutorul funciei de sub integral. Acest criteriu afirm c, dac f: [a, ) R + este o funcie descresctoare i n0 N este cel mai mic numr natural cu proprietatea c n0 a atunci

integrala

f (x ) dx i seria

n =n0

f (n)

au aceeai natur.(vezi [10]).

Propoziia 8.1.4. (formula de integrare prin pri) Fie f, g: [a, ) R , derivabile cu derivate continue pe [a, ) . Presupunem c exist lim f (x )g(x ) i este finit. Atunci, dac una dintre integralele
x

- 172 -

f (x ) g(x ) dx , f (x ) g(x ) dx
a a

este convergent rezult c i cealalt este

convergent i

f (x ) g(x ) dx = lim f (x ) g(x ) f (a) g(a) f (x ) g(x ) dx


a x a

(8.1)

Demonstraie. Fie u > a. Din formula de integrare prin pri pentru integrale definite avem

f (x ) g(x ) dx = f (u) g(u) f (a) g(a) f (x ) g(x ) dx.


u u a a

S
u

presupunem

f (x ) g(x ) dx este
a
u

convergent.

Atunci

exist

lim f (x ) g(x ) dx i este finit i cum lim f (u) g(u) exist i este finit rezult c
a

lim f (x ) g(x ) dx exist i este finit deci


u a

f (x )g(x ) dx
a

este convergent i prin

trecere la limit cu u obinem (8.1). Propoziia 8.1.5. (formula de schimbare de variabil)

Fie f: [a, ) R , continu, : [, ) [a, ) , strict cresctoare, derivabil cu derivata continu. Presupunem c
a

( ) = a,

lim (t ) = + .
t t <

Atunci, dac una dintre

integralele

f (x ) dx, f ((t )) ' (t ) dt este convergent rezult c i cealalt este

convergent i

f (x ) dx = f ((t )) ' (t ) dt
a

( este finit sau infinit).

Demonstraie. n aceeai manier cu demonstraia propoziiei 8.1.4.

- 173 -

8.2. Integrale improprii de spea a doua


Fie f : [a, b ) R , integrabil pe orice compact [a, u] [a, b ), b R, lim f (x ) = + .
x b x <b

Definiia 8.2.1. Funcia f este integrabil (n sens impropriu) pe [a, b) dac


u b u <b

lim f (x ) dx exist i este finit. n acest caz spunem c integrala


u a

f (x ) dx
b a

este

convergent i divergent.

f (x ) dx = lim f (x ) dx.
b u a u b u<b a b

n caz contrar integrala

f (x ) dx este
b a

Analog definim

f (x ) dx , n cazul f : (a,b] R.
a

Dac f : [a, b] \ {c} R , unde c (a, b ) , lim f (x ) = + , f integrabil pe orice


x c

compact [u, v ] [a, b ] \ {c }, spunem c integrala integralele

f (x ) dx este convergent dac


b a

c a

f (x ) dx i

f (x ) dx
b c c a

sunt convergente i n acest caz


b u b

f (x ) dx = f (x ) dx + f (x ) dx = lim f (x ) dx + lim f (x ) dx.


b a c u c u<c a uc u>c u

Exemple.1. Fie integrala integral este improprie

(b x )
a

dx, R . S observm c aceast

numai

pentru

>0.

acest

caz

f : [a, b ) R, f (x ) =

(b x )
1

, lim f (x ) = + . Pentru u (a, b ) vom avea


x b x <b

(b x )
a

1 +1 u a , daca 1 + 1 (b x ) dx = ln b x u , daca = 1 a

, si

u b u<b

lim

(b x )
a

1 (b a) +1, daca < 1 dx = 1 , deci + , daca 1

(b x )
a

dx este convergent

pentru <1 i divergent pentru 1. n cazul convergenei avem

(b x )
a

dx =

1 (b a ) +1. 1

- 174 -

2. Integrala
u (0,1) ) i lim
u0 u>0 1 u

1 0

1 dx este divergent deoarece x

1 u

1 dx = ln x 1 = ln u (pentru u x

1 dx = + . x

Observaie. Dac atunci

f (x ) dx este
b a u b u<b

convergent i f are primitive pe [a, b)

f (x ) dx = F(x )
b a

b a

= lim F(u) F(a ) , unde F : [a, b ) R este o primitiv a

funciei f. Pornind de la rezultatele obinute pentru integralele improprii de spea nti de forma

f (x ) dx ,
a

unde f : [a, ) R se obin rezultate asemntoare pentru

integralele improprii de spea a doua, de forma nlocuind pe + cu b.

f (x ) dx ,
b a

unde f : [a, b ) R ,

Demonstraiile fiind identice ne vom rezuma doar la prezentarea acestor rezultate. Teorema 8.2.1. (Criteriul lui Cauchy). Fie f : [a, b ) R , integrabil pe orice compact [a,u] [a,b). O condiie necesar i suficient pentru convergena integralei

f (x ) dx
b a

este ca ( ) > 0 s existe > 0, < b a astel nct

()x, x (b ,b) s avem

f (t )dx
x b a

< .

Definiia 8.2.2. Integrala

f (x ) dx

se numete absolut convergent dac

integrala

b a

f (x ) dx este convergent.

Propoziia 8.2.1. O integral absolut convergent este convergent.

- 175 -

Propoziia 8.2.2. (Criteriul de comparaie cu inegaliti).

Fie f,g: [a, b ) R, integrabile pe orice compact, [a, u] [a, b ), astfel nct
0 f (x ) g(x ), ()x [a, b ). Atunci

i)

Dac integrala este convergent. Dac integrala este divergent.

g(x ) dx
b a

este convergent rezult c i integrala

f (x ) dx
b a

ii)

f (x ) dx
b a

este divergent rezult c i integrala

g(x ) dx
b a

Propoziia 8.2.3. (Criteriul comparaiei la limit).

Fie f , g : [a, b ) R + , integrabile pe orice compact [a, u] [a, b ). Presupunem c () lim


i)
f (x ) = l , 0 l . Atunci: x b g(x ) x <b

Dac l < i

g(x ) dx
b a

este convergent rezult c

f (x ) dx
b a b

este

convergent.
ii)

Dac l > 0 i

g(x ) dx
b a

este divergent rezult c

f (x ) dx
a

este

divergent.
Consecin. Fie f : [a, b ) R + , integrabil pe orice compact [a,u] [a,b).

Presupunem c exist lim (b x ) f (x ) = l, 0 l . Atunci:


x b x <b

i) ii)

Dac < 1 i l < rezult c Dac 1 i l > 0 rezult c

f (x ) dx
b a

este convergent. este divergent.

f (x ) dx
b a

Exemplu. Fie integrala

1 0 3

1 1 x4
3

dx . 1 , lim f(x) = +
x 1 x <1

Avem

f : [0,1) R+, f(x) =

1 x4

- 176 -

lim(1 x )3
x 1 x <1

1 3

(1 x )(1 + x + x 2 + x 3 )

1 . 4

Rezult = convergent.

1 1 < 1 , l = 3 < i conform consecinei integrala dat este 3 4

Observaie. Dac f : (a, b] R + atunci consecina propoziiei 8.2.3 se

modific uor astfel: Presupunem c exist lim (x a ) f (x ) = l , 0 l . Atunci :


x a x >a

i) ii)

Dac < 1 i l < rezult

f (x ) dx
b a b a

este convergent. este divergent.

Dac 1 i l > 0 rezult c

f (x ) dx

Teorema 8.2.2. (Criteriul lui Dirichlet). Fie f,g:[a,b) R, f continu,

g monoton i lim g (x ) = 0. Presupunem c funcia F:[a,b) R, F(u) =


x b x <b

f (x )dx
u a

este mrginit. Atunci integrala

f (x ) g(x ) dx
b a

este convergent.

Teorema 8.2.3. (Criteriul lui Abel). Fie f , g : [a, b ) R , g monoton i

mrginit, f integrabil pe orice compact convergent. Atunci

[a,u] [a,b) i

f (x )dx
b a

este

f (x ) g(x ) dx
b a

este convergent.

Propoziia 8.2.4. (formula de integrare prin pri).

Fie f , g : [a, b ) R , derivabile cu derivate continue pe [a, b). Presupunem c exist lim f (x )g(x ) i este finit. Atunci, dac una dintre integralele
x b x <b

f (x ) g(x ) dx, f (x ) g(x ) dx


b b a a

este convergent rezult c i cealalt este


b

convergent i

f (x ) g(x ) dx = lim f (x ) g(x ) f (a) g(a) f (x )g(x ) dx. .


b a x b x <b a

- 177 -

Propoziia 8.2.5. (formula de schimbare de variabil).

Fie f : [a, b ) R , continu, : [, ) [a, b ) strict cresctoare, derivabil cu derivata continu. Presupunem c ( ) = a i lim (t ) = b . Atunci, dac una dintre integralele
t t <

b a

f (x ) dx ,

f ((t )) ' (t ) dt

este convergent rezult c i cealalt este


convergent i

b a

f (x ) dx = f ((t )) ' (t ) dt , ( poate fi finit sau infinit).

Exemplu. Fie integrala

2 1

dx x x 1

.
lim f (x ) = +
x 1 x >1 1

Fie

f : (1,2] R, f (x ) =

1 . Cum x x 1
1 x 1 x >1

avem o integral
1 = 1, rezult c x x 1

improprie de spea a doua. Cum lim(x 1)2 f (x ) = lim(x 1)2


x 1 x >1

1 integrala este convergent = < 1, l = 1 < + . Pentru calcul vom folosi 2

schimbarea de variabil dat de


x 1 = t x 1 = t 2 x = t 2 + 1 = (t ), : (0,1] (1,2] ,

este strict cresctoare derivabil cu ' (t ) = 2t , deci continu, lim (t ) = 1 .


t 0 t >0

Conform propoziiei 8.2.5,

2 1

dx x x 1

(
0

1 1 1 2tdt = 2 2 dt = 2arctgt 1 = 2 = . 0 2 0 t +1 t +1 t 4 2

Observaie. Exist integrale improprii care sunt i de spea nti i de

spea a doua, de exemplu

x x2 + 1

dx

. Acestea se vor numi integrale improprii

de spea a treia. Integrala dat va fi convergent dac () a > 0 astfel nct integralele I1 =
a 0

x x +1
2

dx

I2 =

x x2 + 1 dx

dx

sunt convergente. n acest caz

x x2 + 1

= I1 + I2 .

- 178 -

8.3. Integralele lui Euler


Pentru p, q > 0 definim integralele
(p ) = xp 1e x dx,

(p, q) = xp 1(1 x ) dx ,
1 q 1 0

numite integralele lui Euler (sau funciile , ). Vom arta c aceste integrale sunt convergente pentru ( ) p, q > 0. S observm mai nti c integrala (p) este o integral improprie de spea nti iar pentru p (0,1) este i de spea a doua iar (p,q) este o integral improprie de spea a doua pentru p (0,1) sau q (0,1) (altfel, dac p 1 i q 1 este o integral definit). Fie p,q > 0, I1 = xp 1e x dx, I2 = xp 1e x dx . Fie f : [ 1, ) R, f ( x ) = xp 1e x .
1

Cum lim x 2 f (x ) = lim x 2 x p 1e x = lim


x x

x p 1 = 0 rezult c integrala I2 este x e x

convergent (conform propoziiei 8.1.3, = 2 > 1, l = 0 < ). Dac p 1 integrala I1 este o integral definit, deci convergent iar dac
p (0,1) , cum

lim x f (x ) = lim x x p 1e x = 1 > 0 , pentru


x 0 x >0 x 0 x >0

= 1 p < 1, conform

propoziiei 8.2.3 rezult c I1 este convergent. n concluzie (p) = I1+ I2 este convergent. Artm n continuare convergena integralei (p,q). Vom trata cazul
p (0,1) sau q (0,1) .

Fie f : (0,1) R, f (x ) = x p 1 (1 x ) .
q 1

Dac p (0,1) , cum lim x f (x ) = lim x x p 1e x = 1 > 0 , pentru = 1 p < 1,


x 0 x >0 x 0 x >0

conform propoziiei 8.2.3 rezult c (p,q) este convergent n raport cu limita inferioar. Dac q (0,1) , cum lim(1 x ) f (x ) = lim(1 x ) x p 1 (1 x )
x 1 x <1 x 1 x <1 q 1

= 1 > 0 , pentru

= 1 q < 1, rezult c (p,q) este convergent n raport cu limita superioar.

- 179 -

n concluzie (p,q) este convergent.


Propoziia 8.3.1. (proprieti ale funciilor , ). i) ii) iii) iv)
(p + 1) = p(p ), ()p > 0 ;
(p )(1 p ) = , ( ) p (0,1) ; sin p

(p, q) = (q, p ), ()p, q > 0 ;


(p, q) = (p ) (q) , ( ) p, q > 0 . (p + q)

Demonstraie. i) Folosind integrarea prin pri obinem


(p + 1) = x pe x dx = x p e x dx = x pe x 0 0

( )

+ px p 1e x dx = p(p ) ,
0

deoarece lim x p e x = 0 .
x

Cum (1) = e x dx = e x
0

= 1 rezult c pentru nN,

(n + 1) = n(n ) = n(n 1 + 1) = n(n 1)(n 1) = K = n(n 1)K 2 1 (1) = n! .

iii) Folosind schimbarea de variabil x = 1-t obinem


(p, q) = xp 1(1 x ) dx = (1 t ) t q 1dt = t q1(1 t ) dt = (q, p ) .
1 q 1 0 p 1 1 p 1 0 1 0

Aplicaii.
1. Din (ii), pentru p =
1 1 1 1 obinem = = , deci 2 2 2 2

x e x dx = . Folosind schimbarea de variabil x = t2 obinem


1
2

1 2

= x 2 e x dx = te t 2tdt = 2 e t dt , de unde
2

e x dx =

. 2

Folosind schimbarea de variabil x = - t din ultima integral obinem

e x dx =

i n concluzie 2

e x dx = .

2. S se arate c integrala

1 dx este convergent i s se 1+ x6

calculeze.

- 180 -

Fie f : [ 0, ) R, f (x ) =

1 0, () x 0 . 1+ x6

Cum lim x 6 f (x ) = lim x 6


x x

1 = 1 < pentru = 6 > 1 , rezult c integrala 1+ x 6

dat este convergent. Vom folosi schimbarea de variabil dat de x6 = t, deci


1 5 x = t dx = t 6 dt i integrala devine 6
I=
0 1 6

1 dx = 1+ x6

1 1 1 t t dt = dt . 0 1+ t 1+ t 6 6

5 6

5 6

Vom folosi pentru ultima integral schimbarea dat de


t y 1 dy . = y t = y + ty t = dt = 1+ t 1 y (1 y )2 Obinem astfel
1 1 I= 6 0 y y 1 y 1+ 1 y 1
5 6

(1 y )2

1 1 1 1 1 5 dy = y 6 (1 y ) 6 dy = , 6 0 6 6 6

Folosind propoziia 8.3.1. rezult


1 1 5 1 1 sin 1 1 5 1 6 6 1 6 6 1 6 = 1 =. = I = , = = 6 6 6 6 1 5 6 1 6 1 3 6 (1) + 2 6 6

3. S se arate c integrala I =

1 0 n

1 1 xn

dx, n 2 este convergent i s se

calculeze. Fie f : [ 0,1) R, f (x ) = Cum

1
n

1 xn

0, () x [0,1) .

lim (1 x ) f (x ) = lim (1 x )
x 1 x <1 x 1 x <1

(1 x )

1 n n

=
n 1

1
n

1+ x + K + x

< ,

pentru

1 < 1, rezult c integrala dat este convergent. n Pentru calculul integralei vom folosi schimbarea de variabil dat de
1 n

1 1 n 1 x = t x = t dx = t dt . n n

- 181 -

Astfel obinem
I= 1 1 1 t n 0 n 1 t
1 1 n

dt =

1 t n

1 1 1 n 0

(1 t )

1 n

1 1 1 1 1 1 1 n n 1 . = dt = ,1 = n (1) n n n n sin n

Probleme propuse
1) Folosind definiia s se studieze natura integralelor: a) c) e)

0 2 1

arctgx dx; 1+ x2

b) d)

cos 3 xdx ;
1 dx ; x + x +1
2

dx x

2 0

dx ; x ln x

f)

(x

dx
2

+1

2) Folosind definiia s se arate c urmtoarele integrale sunt convergente i

s se calculeze:
a) b)

e 2 x cos axdx , unde a R ;


1
2

(x
0

+1

dx .

3) Folosind criteriile de convergen s se studieze natura integralelor: a) c) e) g)

0 1

x 2dx ; 1 + x5
dx 1 x
2

b)

ln x dx, R ; x sin x dx ; x2 + 1

1 3

d) f) h)

cos x 2dx ; sin x + x 2 dx, > 0 ; x

0 b

sin x 2dx ; xdx

(x a )(b x ) , a < b .
a

- 182 -

4) S se arate c urmtoarele integrale sunt convergente i s se calculeze: a) c) e)

1 dx, n N, n 2 ; 1 + xn

b) d) f)

1 0 3

dx 1 x6
4

;
dx ; dx .

(1 + x )
0

x2

dx ;

(1 + x )
0

x e

x2

dx ;

2 0

1 sin x 3 cos x

- 183 -

CAPITOLUL 9

IRURI I SERII DE FUNCII


9.1. iruri de funcii
Definiia 9.1.1. Se numete ir de funcii un ir (fn), unde

fn: E R R, () n N , sunt funcii reale. Dac x E, irul (fn(x)) este un ir de numere reale. Dac irul (fn(x)) este convergent spunem c x este punct de convergen pentru irul (fn). Vom nota A = {x E : (fn(x)) convergent} i o vom numi mulimea de convergen a irului de funcii (fn). Definiia 9.1.2. Fie fn : E R un ir de funcii i A E mulimea sa de convergen. irul (fn) converge simplu sau punctual pe mulimea A ctre funcia
f : A R i scriem fn s f (pe A) (sau fn A f ) dac lim fn (x ) = f (x ), () x A .
n

Observaie. Conform definiiei convergenei unui ir de numere reale,


fn s f pe A dac i numai dac () x A i () > 0, ()n , x N astfel nct

()n n,x

avem

fn (x ) f (x ) < , () x A .

Exemplu. Fie fn (x ) = x n , pentru x [0,1] .


0, pentru x [0,1) Cum lim x n = rezult c fn s f pe [0,1], unde n 1, pentru x = 1

f(x) =

0, pentru x [0,1) . , 1 pentru x = 1

- 184 -

Observaie. Dac rangul n,x din definiia 9.1.2 este independent de x obinem un alt concept de convergen numit convergen uniform. Definiia 9.1.3. irul de funcii (fn) este uniform convergent pe A ctre
funcia f i scriem fn u f (pe A) sau ( fn A f ) dac () > o, ()n N astfel
u

nct ()n n avem fn (x ) f (x ) < , () x A .

Observaie. Din aceast definiie rezult c, dac fn u f (pe A) atunci


fn s f (pe A).

Exemple. 1. Fie fn (x ) =
n

x , pentru x [0, ) . Observm c 1 + nx

lim fn (x ) = 0, () x 0, deci fn s f , pe [0, ), unde f(x) = 0, () x [0, ) .

Pe de alt parte s observm c


fn (x ) f (x ) = x 1 , () x [o, ), () n N . 1 + nx n

Cum lim

1 = 0 , pentru n

() > 0, ()n N

astfel nct

1 < , ()n n i n

atunci

fn (x ) f (x )

1 < , () x [0, ) , deci fn u f , pe [0,). n

2. Fie fn(x) = xn, pentru x [0,1] . Dup cum am vzut n exemplul 9.1.1.,
0, pentru x [0,1) fn s f pe [0,1], unde f (x ) = . 1, pentru x = 1

S artm c fn u f , pe [0,1]. ntr-adevr, presupunem contrariul, deci

() > 0, ()n N astfel nct ()n n


Lund
= 1 2

avem fn (x ) f (x ) < , () x [0,1] . astfel nct

rezult

()n0 N

()n n0

avem

1 x n < , () x [0,1) . Trecnd n aceast inegalitate la limit cu x 1, x < 1 obinem 2

o contradicie.

- 185 -

Criterii de convergen uniform


Teorema 9.1.1. Fie fn : E R un ir de funcii. irul (fn) converge uniform

pe A E ctre funcia f dac i numai dac


n

lim sup fn (x ) f (x ) = 0 xA

(9.1)
Demonstraie. Dac fn u f pe A atunci conform definiiei, pentru

() > 0, ()n N
de unde

astfel nct () n n fn (x ) f (x ) <

, () x A , 2

() n n sup
x A

fn (x ) f (x )

< , adic lim sup fn (x ) f (x ) = 0 . 2 n x A

Reciproc, dac (9.1) are loc rezult c pentru orice > 0, ()n N astfel

nct ()n n avem sup fn (x ) f (x ) < , de unde


xA

() x A

fn (x ) f (x ) sup fn (x ) f (x ) < , adic fn u f pe A.


x A

Exemplu. Fie fn (x ) =
n

x , pentru x R . 1 + nx 2

Cum lim fn (x ) = 0, () x R rezult c fn s f , pe R, unde f (x ) = 0, () x R . Evident fn este derivabil i fn (x ) = punctul x =


1 n

(1 + nx )

1 nx 2

2 2

1 , deci fn (x ) = 0 x = ; n

este punct de maxim local i x =

1 este punct de minim local n

pentru fn. Cum lim fn (x ) = 0 , aceste puncte sunt puncte de extrem global.
x

1 1 rezult c Cum fn = n 2 n
1 lim sup fn (x ) 0 = lim = 0 , adic fn u 0 , pe R. xR n 2 n n

- 186 -

Teorema 9.1.2. (Criteriul lui Cauchy). irul de funcii (fn) este uniform

convergent pe A dac i numai dac pentru

() > 0, ()n N

astfel nct

()m, n n

avem
fm (x ) fn (x ) < , () x A .

(9.2)

Demonstraie. S presupunem c fn u f pe A i fie > 0 . Conform

definiiei ()n N astfel nct ()n n avem Pentru m,n n vom avea

fn (x ) f (x ) <

, () x A . 2

fm (x ) fn (x ) fm (x ) f (x ) + f (x ) fn (x ) <

+ = , () x A. , 2 2

adic (9.2) este ndeplinit.


Reciproc, s presupunem c pentru () > 0, ()n N astfel nct

()m,n n
deci convergent. Fie f : A R,

avem fm (x ) fn (x ) <

, () x A 2

(9.3)

Atunci, pentru orice x A , fixat, irul numeric (fn(x)) este ir Cauchy n R,


f (x ) = lim fn (x ) ; deci fn s f , pe A.
n

Fixnd n (9.3) n n i fcnd m obinem


fn (x ) f (x ) < , () n n i () x A , deci fn u f pe A. 2

Teorema 9.1.3. (Criteriul majorrii). Fie f , fn : A R . Dac exist un ir de

numere pozitive (an), convergent ctre zero, astfel nct


fn (x ) f (x ) an , () n N i () x A ,

(9.4)

atunci fn u f pe A.

Demonstraie. Fie > 0 ; cum a n 0, ()n N astfel nct a n < , ()n n

i atunci, folosind (9.4) obinem


fn u f pe A.

fn (x ) f (x ) an < , () n n i () x A , adic

- 187 -

Exemplu. Fie fn (x ) =
fn (x ) 0 =

cos nx , pentru x R . Evident avem n +1


1 n +1 , () n N, () x R .

cos nx n +1

Cum lim
n

1 n +1

= 0 rezult c fn u 0 , pe R.

Proprieti ale irurilor uniform convergente


n continuare vom arta c uniform convergena pstreaz la limit o serie de proprieti ale funciilor irului cum ar fi: mrginirea, continuitatea, derivabilitatea, integrabilitatea, etc.
Teorema 9.1.4. (Transfer de mrginire). Fie fn : A R un ir de funcii
mrginite pe A i f : A R . Dac fn u f pe A atunci f este mrginit pe A.

Demonstraie. Cum fn u f pe A, conform definiiei, pentru = 1,

()n0 N

astfel nct

()n n0

avem fn (x ) f (x ) < 1, ()x A i n particular,

pentru n = n0 obinem
fn 0 (x ) f (x ) < 1 , ()x A.
0 0

(9.5)

Cum fn este mrginit pe A, () M > 0 astfel nct fn (x ) M, () x A i atunci, folosind (9.5) obinem
f (x ) f (x ) fn 0 (x ) + fn 0 (x ) < 1 + M, () x A, adic f este mrginit pe A.

Observaie. Dac irul de funcii (fn) nu este uniform convergent pe A,

concluzia teoremei nu este n general adevrat. n acest sens considerm irul de funcii fn (x ) =
f (x ) = nx , pentru x (0,1) . Avem fn s f pe (0,1), unde 2 1 + nx
fn (x ) n , () x (0,1) , pentru n N , fixat, 2

1 , ( )x (0,1) . Evident avem x

deci funciile fn sunt mrginite. S observm c f nu este mrginit pe (0,1).

- 188 -

Teorema 9.1.5. (Transfer de continuitate). Fie fn : A R un ir de funcii


continue ntr-un punct x 0 A . Dac fn u f pe A atunci funcia f este continu

n x0.
Demonstraie. Fie > 0 . Cum fn u f pe A,

() n N astfel nct
avem

()n n i ()x A
fn (x ) f (x ) <

avem

fn (x ) f (x ) <

. n particular, pentru n=n 3

, () x A . 3

Cum fn este continu n x0, () > 0 astfel inct ()x A cu x x 0 < avem fn (x ) fn (x 0 ) <

. Pentru ()x A cu x x 0 < vom avea 3 + + = , 3 3 3

f (x ) f (x 0 ) f (x ) fn (x ) + fn (x ) fn (x 0 ) + fn (x 0 ) f (x 0 ) <

deci f este continu n x0.

Observaie. Din teorema 9.1.5 rezult c, dac fn u f pe A i funciile

fn sunt continue pe A atunci funcia f este continu pe A. Dac irul de funcii (fn) nu este uniform convergent, concluzia nu este n general adevrat. n acest sens considerm irul de funcii fn (x ) = xn , pentru x [0,1] .
Evident funciile fn sunt continue pe [0,1], fn s f

pe [0,1], unde

0, daca x [0,1) , dar funcia f nu este continu pe [0,1]. f (x ) = 1, daca x = 1

Teorema 9.1.6. (Transfer de integrabilitate).


Dac fn : [a, b] R este un ir de funcii integrabile pe [a,b] i fn u f pe

[a,b], atunci f este integrabil pe [a,b] i lim fn (x ) dx = f (x ) dx .


b b n a a

Demonstraie. Fie > 0 . Cum fn u f , ()n N astfel nct ()n n i

()x [a, b] avem

fn (x ) f (x ) <

. n particular, pentru n = n avem 4(b a )

- 189 -

fn (x )

< f (x ) < fn (x ) + , () x [a, b] 4(b a ) 4(b a )

(9.6)

Fie D ([a,b]), = (a = x 0 < x 1 < K < x n = b ) . Cum


fn

este

integrabil

rezult

fn

este

mrginit

fie

Mk = sup fn (x ) : x [xk 1, x k ] ,

mk = inf fn (x ) : x [xk 1, xk ] , k = 1, n .

Din criteriul lui Darboux de integrabilitate () > 0 astfel nct ()D adic

([a, b]) , = (a = x0 < x1 < K < xn = b) cu

< avem S (fn ) s (fn ) <


, 2

(M
n k =1

mk (xk xk 1 ) <

. 2

(9.7)

Cum irul (fn) este uniform convergent pe [a,b] ctre funcia f, din teorema 9.1.4 rezult c f este mrginit pe [a,b]. Fie Mk = sup{ f (x ) : x [x k 1, x k ] }, m k = inf {f (x ) : x [x k 1, x k ] }, k = 1, n . Trecnd n relaia (9.6) la supremum apoi la infimum, pentru x [x k 1, x k ], obinem
Mk

Mk Mk + i 4(b a ) 4(b a ) mk mk + 4(b a ) 4(b a )

mk

Scznd aceste relaii membru cu membru i nmulindu-le cu x k x k 1 obinem

(Mk mk ) (xk xk 1 ) (Mk mk ) (xk xk 1 ) +


de unde, prin nsumare i folosind (9.7) rezult

(xk xk 1 ) , 2(b a )

S (f ) s (f ) = (Mk mk ) (xk xk 1 ) Mk mk (xk xk 1 ) + k =1 k =1

n (xk xk 1 ) = S fn s fn + 2 < 2 + 2 = 2(b a ) k =1

( )

( )

adic f este integrabil pe [a,b].

- 190 -

Cum

f (x ) dx f (x ) dx = (f (x ) f (x )) dx
b b b a n a a n

f (x ) f (x ) dx 4(b a) (b a) = 4 < , () n n
b a n

rezult c

lim fn (x ) dx = f (x ) dx .
b b a a

Aplicaie. Fie {r1,r2 ,K,rn ,K} mulimea numerelor raionale din [0,1] i sirul

de funcii fn : [0,1] R, fn (x ) =

1, pentru x { r1, r2 ,K, rn } , () n N .S se arate c 0 , pentru x { r1, r2 ,K, rn }

irul de funcii (fn) nu este uniform convergent pe [0,1].


1, pentru x [0,1] Q S observm c lim fn (x ) = , deci fn s f pe [0,1], n 0, pentru x [0,1] \ Q 1, pentru x [0,1] Q unde f (x ) = . S presupunem prin absurd c fn u f pe [0,1] \ Q 0, pentru x

[0,1]. Atunci conform teoremei 9.1.6 funcia f este integrabil pe [0,1], contradicie.
Teorema 9.1.7. (Transfer de derivabilitate).

Fie I R un interval mrginit i fn : I R un ir de funcii derivabile pe I . Dac exist un punct x 0 I astfel nct irul (fn(x0)) este convergent i
exist o funcie g : I R astfel nct fn u g pe I , atunci

i) ii)

exist o funcie f : I R astfel nct fn u f pe I;

f este derivabil pe I i f = g, adic lim fn = lim fn .


n n

Pentru demonstraie se poate consulta [10].


Observaii. 1. Dac pentru un ir de funcii derivabile fn : I R ,

convergent pe I ctre o funcie f, irul (fn) converge ctre f vom spune c irul (fn) se poate deriva termen cu termen.

- 191 -

2. Exist iruri de funcii care pot fi derivate termen cu termen fr ca irul

derivatelor s convearg uniform. n acest sens considerm irul de funcii fn (x ) =


n

ln(1 + n 4 x 2 ) , pentru x [0,1] . 2n

S observm c lim fn (x ) = 0, ()x [0,1], deci fn s 0 pe [0,1].

Funciile fn sunt derivabile pe [0,1],


lim fn (x ) = 0 , () x [0,1] , deci

n3 x fn (x ) = , 1 + n4 x 2

() x [0,1] i

fn ' s 0 pe [0,1], adic

fn s f pe [0,1].

Prin urmare, irul (fn) se poate deriva termen cu termen. S observm totui c irul (fn) nu converge uniform la 0.

9.2.Serii de funcii
Definiia 9.2.1. Se numete serie de funcii o serie
fn : E R R, ()n 1.

f
n =1

, unde

Fie S n : E R, S n = fk , numit i sirul sumelor pariale. Fie A E


k =1

muimea de convergen a irului de funcii (Sn), numit i mulimea de convergen a seriei de funcii.
Definiia 9.2.2. Spunem c seria de funcii

f
n =1

converge simplu sau

punctual pe A dac irul sumelor pariale (Sn) este simplu convergent pe A. Dac
f = lim S n atunci f se va numi suma seriei de funcii
n

f
n =1 s

n sensul convergenei

punctuale i scriem

fn = f (punctual pe A) sau
n =1

fn = f .
n =1

Definiia 9.2.3. Seria de funcii

f
n =1

este uniform convergent pe A dac

irul sumelor pariale (Sn) este uniform convergent pe A. Dac S n u f pe A,

- 192 -

atunci f se va numi suma seriei de funcii vom scrie

f
n =1 u

n sensul convergenei uniforme i

fn = f (uniform pe A) sau
n =1

fn = f.
n =1

Criterii de convergen uniform


Teorema 9.2.1. (Criteriul lui Cauchy). Fie fn : A R R . Seria de funcii

f
n =1

converge uniform pe A dac i numai dac pentru () > 0, () n N astfel

nct ()n n i ()p 1 avem fn +1(x ) + fn + 2 (x ) + K + fn +p (x ) < , () x A .


Demonstraie. Fie (Sn) irul sumelor pariale ale seriei

f
n =1

. Conform

teoremei 9.1.2, (Sn) converge uniform pe A dac i numai dac pentru

() > 0, () n N astfel nct () n n i () p 1 avem


Sn + p (x ) Sn (x ) < , () x A ,

sau echivalent
fn +1(x ) + fn + 2 (x ) + K + fn + p (x ) < , () x A ,

i demonstraia este ncheiat.

Teorema 9.2.2. (Criteriul lui Weierstrass). Fie fn : A R R . Dac exist

o serie numeric convergent

a
n =1

, cu termeni pozitivi astfel nct (9.8)

fn (x ) an , () n N, () x A .

atunci seria de funcii

f
n =1

este uniform i absolut convergent pe A.

Demonstraie. Fie > 0. Cum

a
n =1

este convergent, din criteriul lui

Cauchy de la serii numerice () n N astfel nct () n n i ( ) p 1 avem


a n +1 + a n + 2 + K + a n +p < , i folosind (9.8) rezult c pentru () n n i

() p 1 avem

- 193 -

fn +1(x ) + fn + 2 (x ) + K + fn + p (x ) fn +1(x ) + fn + 2 (x ) + K + fn + p (x ) an +1 + an + 2 + K + an + p < , () x A.

(9.9)

Conform teoremei 9.2.1 rezult c seria de funcii convergent pe A. Absolut convergenta rezult din (9.9).

f
n =1

este uniform

Exemplu.
cos nx n +x
3 2

Fie
1 n

seria

de

funcii

n =1

cos nx n3 + x 2

,x R.

Observm

c este

1 n +x
3 2

, ( ) n N i ()x R iar seria

numeric

n =1

1 n
3 2

convergent. Folosind criteriul lui Weierstrass rezult c seria dat este uniform convergent pe R .
Teorema 9.2.3. (Criteriul lui Dirichlet). Fie fn , gn : A R R dou iruri de

funcii astfel nct seria de funcii

f
n =1

are irul sumelor pariale uniform mrginit

iar irul (gn) este monoton descresctor i uniform convergent la 0. Atunci seria

f g
n =1 n

este uniform convergent.

Demonstraie. Cum irul sumelor pariale (Sn) asociat seriei

f
n =1

este

uniform mrginit () M > 0 astfel nct

Sn (x ) M, () n N i ()x A .

Fie > 0 . Cum gn u 0 pe A, () n N astfel nct () n n avem


gn (x ) , ()x A . 2M

Pentru n n , p 1 i x A vom avea

- 194 = gn +1(x )(Sn +1(x ) Sn (x )) + gn + 2 ( x )(Sn + 2 (x ) Sn +1(x )) + K + gn + p ( x )(Sn + p (x ) Sn + p 1(x )) = fn +1(x )gn +1(x ) + fn + 2 (x )gn + 2 (x ) + K + fn + p (x )gn + p (x ) =

+ Sn + p 1(x ) (gn + p 1(x ) gn + p (x )) + gn + p (x )Sn + p (x ) gn +1(x

= gn +1(x )Sn (x ) + Sn +1(x )(gn +1(x ) gn + 2 (x )) + K +

) Sn (x ) + Sn +1(x ) gn +1(x ) gn + 2 (x ) + K + Sn +p 1(x ) gn +p 1(x ) gn +p (x ) + + gn + p (x ) Sn + p (x ) M gn +1(x ) + M gn +1(x ) gn + 2 (x ) + K + M gn + p 1(x ) gn + p (x ) + + M gn + p (x ) = M gn +1(x ) + M (gn +1(x ) gn + 2 (x )) + K + M (gn + p 1(x ) gn + p (x )) + M gn + p (x ) =
= 2M gn + p (x ) 2M = , 2M

ceea ce arat c seria

f g
n =1 n

este uniform convergent pe A.

Proprieti ale seriilor uniform convergente


Pornind de la proprietile irurilor uniform convergente se obin cu

uurin urmtoarele rezultate:

Teorema 9.2.4. Fie fn : A R R un ir de funcii. Dac seria

f
n =1

este

uniform convergent pe A ctre f i fn sunt funcii mrginite atunci f este mrginit pe A.


Demonstraie. Fie S n = fk . Cum fn este mrginit rezult c Sn este
k =1 n

mginit,

()n 1.

Cum seria

f
n =1

este uniform convergent rezult c

S n u f pe A i atunci conform teoremei 9.1.4 rezult c f este mrginit

pe A.

Teorema 9.2.5. Fie fn : A R R un ir de funcii astfel nct seria

f
n =1

s fie uniform convergent la f. Dac funciile fn sunt continue ntr-un punct x 0 A (respectiv pe A) atunci f este continu n x0 (respectiv pe A).

- 195 -

Demonstraie. Rezult imediat din teorema 9.1.5.

Teorema 9.2.6. Dac fn : [a, b] R este un ir de funcii integrabile pe [a,b]

iar seria

f
n =1

converge uniform la f atunci f este integrabil pe [a,b] i


b fn dx = fndx a n =1 a n =1 b

(9.10)

Demonstraie. Fie Sn = fk ; atunci S n u f pe A. Cum fk este


k =1

integrabil pe [a,b], ()k 1 rezult c Sn este integrabil pe [a,b], ()n 1 i conform teoremei 9.1.6 funcia f este integrabil pe [a,b] i
n b n b lim fk dx = fdx . a n a k =1

lim Sndx = fdx sau


a a

Cum

n b n fk dx = fdx , rezult imediat (9.10). a k =1 a k =1 b

Observaie. n ipotezele teoremei 9.2.6 avem egalitatea (9.10) i n acest

caz spunem c seria de funcii

f
n =1

poate fi integrat termen cu termen.

Teorema 9.2.7. Fie I R un interval mrginit, fn : I R un ir de funcii

derivabile pe I nct seria


x 0 I astfel nct seria

f'
n =1

este uniform convergent la g i exist un punct

fn (x 0 ) este convergent. Atunci seria


n =1

f
n =1

este uniform

convergent pe I ctre o funcie f derivabil pe I iar f = g, adic


fn = fn . n =1 n =1
n

(9.11)

Demonstraie. Fie Sn = fk , Tn = fk, () n N . Cum irul de funcii (Sn)


k =1 k =1

satisface ipotezele teoremei 9.1.7, exist o funcie f : I R astfel nct


S n u f pe I, f este derivabil pe I i f = g, adic

- 196 -

fn = f i fn = fn n =1 n =1 n =1
u

Observaie. n ipotezele teoremei 9.2.7 avem egalitatea (9.11) i n acest

caz spunem c seria de funcii

f
n =1

poate fi derivat termen cu termen.

9.3. Serii de puteri


Definia 9.3.1. Se numete serie de puteri o serie de funcii
n

f
n =1

, unde

fn (x ) = an xn , x R, an R, () n N sau fn (x ) = a n (x a ) , unde a R . n continuare

considerm fn (x ) = a n x n i astfel o serie de puteri va fi de forma

n =0

a nxn = a0 +a1x ++ anxn +, unde (an) este un ir de numere reale.


Numerele an se vor numi coeficienii seriei de puteri. S observm c mulimea de convergen A a unei serii de puteri este

nevid deoarece 0 A .
Exemplu. Fie seria de puteri

n=0

n ! = 1 + 1! + 2 ! + K + n ! + K

xn

x2

xn

Fie x 0 R , arbitrar. Considerm seria numeric


xn xn 0 , cu termenul general xn = 0 n! n =0 n !

Cum lim

x n +1 xn

= lim

(n + 1)!

x n +1 0

n! x0
n

= 0 < 1, din criteriul raportului rezult c seria

xn n0! este absolut convergent, deci convergent. n =0

Cum x 0 R este arbitrar rezult c mulimea de convergen a seriei este A = R.

- 197 -

Teorema 9.3.1. (Teorema I a lui Abel).

Fie seria de puteri


i) ii)

n=0

x n , a n R . Atunci exist r [ 0, ) astfel nct

Seria este absolut convergent pe (-r,r); Seria este divergent n () x cu x > r .

Pentru () 0 < r0 < r seria este uniform convergent pe [-r0, r0].


Demonstraie. Dac seria de puteri este convergent numai n punctul

x = 0, lund r = 0, teorema este demonstrat. Pesupunem c exist x 0 0 punct de convergen pentru seria de puteri, adic seria numeric convergent. Vom arta c convergen. ntr-adevr, fie x R cu x < x 0 . Cum seria
n

n =0

x n este 0

()

x cu x < x 0

rezult c x este punct de

n =0

an x 0

este convergent

rezult c lim an x 0 = 0 i atunci irul (anx0n) este mrginit, deci ()M > 0 astfel
n

nct a x M, () n N . Vom avea an x


n n 0

= ax
n

n n 0

x x0

x M , () n N i cum x0

x < 1 rezult ca seria geometric x0

x M x n=0 0

este convergent. Folosind criteriul

de comparaie cu inegaliti rezult ca i seria seria

a x
n=0 n

este convergent, deci

a x
n =0 n

este absolut convergent.

Astfel, dac x 0 0 este un punct de convergen al seriei atunci orice punct x R cu x < x 0 este punct de convergen absolut al seriei. Rezult c mulimea de convergen conine ntreg intervalul ( x 0 , x 0 ). De aici deducem c, dac x1 este un punct de divergen al seriei, atunci orice punct x cu x > x 1 este punct de divergen al seriei.

- 198 -

ntr-adevr, dac ar exista un punct x0 cu x 0 > x 1 n care seria este convergent, atunci, din prima parte a demonstraiei seria ar fi convergent i n x1 (deoarece x 1 < x 0 ), contradicie. Fie A mulimea de convergen a seriei de puteri. Evident avem 0 A , deci A . Fie r = sup A i evident r > 0 (deoarece suntem n cazul n care exist x 0 0 punct de convergen). Vom arta c r satisface concluziile teoremei.
i)

Fie x ( r, r ) ; avem deci x < r . Fie x 0 R astfel nct

x < x 0 < r . Cum x0 este punct de convergen al seriei, rezult c seria este

absolut convergent n x, deoarece x < x 0 .


ii)

Dac r = + inegalitatea x > r nu are sens, deci n acest caz nu

avem ce demonstra. S presupunem c r < + i fie x un punct astfel nct x < r . Dac x ar fi punct de convergen atunci orice punct y cu r < y < x convergen, care contrazice faptul c r = sup A.. Rmne s demonstrm ultima parte a teoremei. Fie 0 < r0 < r . Atunci r0 este punct de absolut convergen al seriei, deoarece seria este convergent. Pentru
x [ r0 , r0 ] avem x r0 , deci
n an xn an r0 , () n N . Folosind

este punct de

a r
n =0

n n 0

= a n r0n
n=0

criteriul lui Weierstrass rezult c seria [-r0, r0].

a x
n =0 n

este uniform convergent pe

Numr real r se numete raz de convergen iar I = ( r, r ) se numete interval de convergen. Vom da n continuare o metod de calcul al razei de convergen.

- 199 -

Teorema 9.3.2. Fie seria de puteri

a x
n=0 n

i r raza sa de convergen. Atunci


r= 1

Presupunem c exist
= 0 r = +, = + r = 0 ).

= lim n an .
n

(cu conveniile

Demonstraie. Fie x 0 R i seria numeric


x n = an x n . Evident avem lim 0
n n

n=0

an xn , cu termenul general 0

xn = lim

an x 0 = x 0 .

Vom folosi criteriul lui Cauchy de la serii de numere reale. Dac = 0 , atunci lim n x n = 0 , deci seria
n

a x
n =0 n

n o

este absolut convergent, ()x 0 , deci

r = + . Dac = + i x 0 0 , cum lim n x n = rezult c seria


n

a x
n =0 n

n o

este

divergent pentru orice x 0 0 , adic r = 0. Fie 0 < < + . Dac x 0 < absolut convergent. Dac x 0 > Atunci x 0 > 1 i deci seria

1 avem x 0 < 1, deci seria

a x
n =0 n

n o

este

1 1 , lum un punct x1 astfel ca x 1 > x 0 > .


an x n o

n =0

este divergent. Din prima parte a

demonstraiei teoremei 9.3.1 rezult c seria concluzie r =


1 .

a x
n =0 n

n 1

este divergent. n

Corolarul 9.3.1. Fie seria de puteri


lim a an 0 . Atunci r = lim n . n an + 1 n an + 1

a x
n=0 n

. Presupunem c exist

- 200 -

Exemplu. Fie seria de puteri


lim
n

n x ( 1) n=1

. Avem

a n = ( 1)

1 n

n +1 an = lim = 1, deci raza de convergen este r = 1 i intervalul de n n an + 1

convergen I = ( 1,1) . Pentru x = -1 obinem seria numeric


divergent. Pentru x = 1 obinem seria numeric Leibniz este convergent. n concluzie mulimea de converge este A = (-1, 1].

( 1)n = 1 , ( 1) n n
n n =1 n =1 n

care este

( 1)
n =1

1 , care conform criteriului lui n

Proprieti ale seriilor de puteri


Fie seria de puteri

a x
n =0 n

, r raza de convergen, A mulimea de


n

convergen i f : A R , f(x) = convergen.

a x
n=0 n

, suma seriei de puteri pe mulimea de

Dac 0 < r0 < r , conform teoremei 9.3.1 seria de puteri

a x
n=0 n

este

uniform convergent pe [-r0, r0] i folosind teorema 9.2.5 rezult c funcia f, suma seriei, este continu pe [-r0, r0]. Cum r0 este arbitrar n (0,r) rezult c f este continu pe (-r,r).
Propoziia 9.3.1. Dac seria de puteri
r > 0 , suma f pe (-r,r), atunci

a x
n =0 n

are raza de convergen

i)

Seria derivatelor

na x
n =1 n

n 1

are aceeai raz de convergen r ;

- 201 -

ii)

Funcia f este derivabil pe (-r,r) i f = g, unde g(x) =

na x
n =1 n

n 1

()x ( r, r ) , adic f (x) = nan x n1 ()x ( r, r ) . n=1


Spunem c seria de puteri poate fi derivat termen cu termen.
Demonstraie. i) ii)

Rezult din faptul c lim

nan = lim n
n

an .

Rezult din faptul c seriile

an x n i
n =1

na x
n =1 n

n 1

sunt uniform

convergente pe ()[ r0 , r0 ] ( r, r ) i din teorema 9.2.7.


Corolarul 9.3.1. Dac seria de puteri

a x
n =1 n

are raza de convergen

r > 0 , suma f pe (-r,r) atunci f este indefinit derivabil pe (-r,r) i f (k ) (x ) = n(n 1)K (n k + 1)a n x n k , ()x ( r, r ) .
n =k

Propoziia 9.3.2. Dac seria de puteri


r > 0 i suma f pe (-r,r) atunci

n =0

an x n

are raza de convergen

i) ii)

Seria
x

n=0

n + 1x

an

n +1

are aceeai raz de convergen r ;

0 f (t )dt =

a n n +1 x () x < r , n=0 n + 1

adic seria de puteri poate fi integrat termen cu termen pe orice interval [0,x], unde x ( r, r ) .
Demonstraie. i) ii)

Rezult din faptul c lim n


n

an = lim n an . n n +1

Rezult din uniform convergena seriei de puteri pe

() [ r0 , r0 ] ( r, r ) i teorema 9.2.6.

- 202 -

Propoziia 9.3.3. i) Dac seria de puteri

n =0

a x
n

are raza de convergen

r i 0 atunci seria de puteri


ii)

n=0

(a ) x
n n =0

are aceeai raz de convergen.

Dac seriile de puteri

an x n ,

n =0

b x
n

au razele de convergen r1

i respectiv r2 atunci seria

n=0

(a

+ b n ) x n are raza de convergen r min{ r1, r2 }.

Demonstraie. Rezult imediat folosind proprietile de la operaiile cu

serii numerice.

xn 1 n . Avem an = n , nlim n an = 1, deci r =1 i n =1

Exemplu. Fie seria

I = ( 1,1) , intervalul de convergen.

Cum pentru x=1 seria este divergent iar pentru x=-1 este convergent rezult c mulimea de convergen este A=[-1,1). Fie f : [ 1,1) R suma seriei pe mulimea de convergen, deci
xn f ( x ) = , () x [ 1, 1) . n =1 n

Conform propoziiei (9.3.1) funcia f este derivabil pe (-1,1) i f (x) = 1+x+x2++xn+=


f (x) =

1 , () x ( 1, 1), de unde 1 x

1 dx = ln (1 x ) + C , () x ( 1, 1), unde C R. . 1 x

Cum f(0) = 0 rezult C = 0, deci f(x) = -ln (1-x), () x ( 1,1) .

Seria binomial
Fie seria de puteri 1 + unde R .
( 1) 2 ( 1)...( n + 1) n x+ x + ... + x + ... , n! 1! 2!

(9.12)

S observm c pentru N se obine un polinom de grad . Vom presupune deci R \ N .

- 203 -

Cum

lim

an ( 1)...( n + 1) (n + 1) ! n +1 = lim = lim =1 an + 1 n! ( 1)...( n) n n n

rezult c raza de convergen a seriei este r = 1. Fie f : (-1, 1) R suma seriei. Conform propoziiei (9.3.1) funcia f este derivabil pe (-1, 1) i
f ' (x) = + ( 1) ( 1)...( n + 1) n1 x + ... + x + ... , () x ( 1, 1) 1! (n 1) !

(9.13)

nmulind n (9.13) cu x obinem


x f ' ( x) = x + ( 1) 2 ( 1)...( n + 1) n x + ... + x + ... , () x ( 1, 1) (9.14) 1! (n 1 ) !

Din (9.13) i (9.14) se obine (1+x) f (x) = f(x), () x ( 1,1) de unde


f ' ( x) = , () x ( 1, 1) f(x) 1 + x

(9.15)

(s observm c f ( x ) 0, ()x ( 1, 1) , deoarece dac ar exista x 0 ( 1,1) cu f(x0) = 0 ar rezulta f(k)(x0) = 0, () k N i atunci f = 0, contradicie, cci f(0) = 1) Din (9.15) obinem ln f(x) = ln (1+x)+lnc deci f(x) = c (1+x), () x ( 1,1) unde c > 0. Cum f(0) =1 rezult c =1 i atunci f(x)=(1+x), () x ( 1,1) . Obinem deci
(1 + x ) = 1 + ( 1) 2 ( 1)...( n + 1) n x+ x + ... + x + ..., () x ( 1,1) 1! 2! n!

(9.16) care generalizeaz formula binomului lui Newton, adevrat pentru N. Pentru diferite valori particulare pentru din (9.16) se obin sumele unor serii importante. Pentru = -1 obinem
1 = 1 x + x 2 x 3 + ... + ( 1)n x n + ..., () x ( 1, 1) , 1+ x

(9.17)

de unde
1 = 1 + x + x 2 + ... + x n + ..., () x ( 1, 1) . 1 x

(9.18)

- 204 -

Pentru =
1+ x = 1+

1 obinem 2
1 1 1 3 5...( 2n 3) n x 2 x 2 + ... + ( 1)n 1 x + ... , 2 1! 2 2! 2n n ! () x ( 1, 1),

(9.19)

Pentru =

1 obinem 2

1 1 1 3 2 1 3 5...( 2n 1) n = 1 x+ 2 x + ... + ( 1)n x + ... , 2 1! 2 2! 2n n ! 1+ x () x ( 1, 1),

(9.20)

de unde
1 1 x2 = 1+ x2 1 3 4 (2n 1) ! ! 2n + x + ... + x + ... , 2 1! 2 4 (2n )! ! () x ( 1, 1),

(9.21)

Prin integrare termen cu termen obinem


arcsin x = x + (2n 1) ! ! x3 1 3 5 + x + ... + x 2n +1 + ... , 32 524 (2n + 1) (2n)! ! () x ( 1, 1)

(9.22)

De asemenea din (9.17), nlocuind pe x cu x2 obinem:


1 = 1 x 2 + x 4 x 6 + ... + ( 1)n x 2n + ..., () x ( 1, 1) i prin integrare termen 2 1+ x

cu termen gsim c arctg x = x


x3 x5 x7 x 2n +1 + + ... + ( 1)n + ..., () x ( 1, 1) . 3 5 7 2n + 1

(9.23)

Serii Taylor
Fie seria de puteri

a
n =0

x n ,r > 0 raza sa de convergen i f suma sa pe

intervalul de convergen, deci f (x) = a n x n , () x ( r, r ) .


n =0

Din propoziia (9.3.1) rezult c f este indefinit derivabil pe (-r, r) i f(n)(0) = n!an, () n N , deci a n =
f ( n ) (0 ) i atunci n!

- 205 -

f (x) =

f ( n ) (0 ) n x , () x ( r, r ) . n! n=0

(9.24)

S observm c seria de puteri din (9.24) poate fi asociat oricrei funcii indefinit derivabile n origine.
Definiia 9.3.2. Fie I R un interval astfel nct oInt I i f:I R, indefinit

derivabil n x = 0. Se numete serie Taylor ataat funciei f n x = 0 seria de puteri


f ( n ) (0 ) n n! x , x I n =0

(9.25)

Fie r 0 raza de convergen a seriei (9.25). Se pune problema n ce condiii suma seriei (9.25) coincide cu funcia iniial f pe intervalul de convergen (-r, r). S reamintim formula lui Mac Laurin de la funcii reale de variabil real. Dac I R este un inteval deschis astfel nct 0 I i f :I R este o funcie de n ori derivabil pe I, unde n N* atunci
x n (n) x f (0) + R n ( x ), () x I . f ( x ) = f (0) + f ' (0) + ... + n! 1!

(9.26)

Dac f este de (n+1) ori derivabil pe I atunci restul de ordin n, Rn, se


x n+1 ( n+1) poate scrie sub forma R n ( x ) = f () , unde este ntre 0 i x (forma (n + 1)!

Lagrange a restului).
Teorema 9.3.3. Fie I R un interval astfel nct 0 Int I, f :I R, indefinit

derivabil pe I i A mulimea de convergen a seriei (9.25). Atunci suma seriei Taylor ataat funciei f n x = 0 coincide cu funcia f pe A I dac i numai dac lim Rn ( x ) = 0, () x A I, unde Rn este restul de ordin n
n

din formula lui Mac-Laurin.


Demonstraie. Fie S n ( x ) = f (0) +
f ' (0 ) f ( n ) (0 ) n x + ... + x , pentru x I i atunci, 1! n!

din formula Mac-Laurin avem


f ( x ) = S n ( x ) + R n ( x ), () x I .

(9.27)

- 206 -

S observm c Sn(x) coincide cu polinomul Taylor


f (0 ) f n (0 ) n Tn (x ) = f (0 ) + x + ... + x . 1! n!

Din (9.27) rezult imediat c lim Sn (x ) = f (x ) lim Rn (x ) = 0, () x A I .


n n

n acest caz spunem c f este dezvoltabil n serie de puteri pe A I i


f (x ) = f (0 ) + f (0 ) f (0 ) 2 f (n ) (0 ) n x+ x + ... + x + ..., () x A I, se numete dezvoltarea n 1! 2! n!

serie de puteri a funciei f pe A I.


Corolarul 9.2.3.

Dac

f C (I), 0 Int I i

exist

M>0

astfel

nct

f (n ) (x ) M, () x I, () n atunci f este dezvoltabil n serie de puteri pe I i


f (x ) = f (n ) (0 ) n x , () x I. n! n =0

Demonstraie. Fie x I . Atunci ()n * , exist n ntre 0 i x astfel nct


R n (x ) = x n+1 (n+1) ( n ) . f (n + 1)!

Conform ipotezei R n (x ) Cum


lim x
n +1

(n + 1)!

n +1

M, () x I, ()n . .

(n + 1) !

= 0, () x I rezult

lim R n ( x ) = 0, () x I

i aplicm

teorema 9.3.3.

Exemple. 1. Fie funcia f:R R, f(x) = ex. Avem f(n)(x) = ex, () xR,

() nN, f(n)(0) = 1,() nN iar seria Taylor asociat funciei f n x = 0 este

xn n! . n =0

Raza de convergen a acestei serii de puteri este r =+ deci intervalul de convergen este I = ( , + ) = R i coincide cu mulimea de convergen.

- 207 -

Pentru () M>0 avem f (n ) ( x ) = e x e M , () x(-M, M), () nN, i atunci conform corolarului 9.3.2 funcia f este dezvoltabil n serie de puteri pe(-M, M), deci pe R i
xn x x2 xn e = = 1+ + + ... + + ..., () x R . 1! 2! n! n =0 n!
x

(9.28)
n ), () xR, 2

2. Fie funcia f:RR, f(x)=cos x. Avem f(n)(x) = cos (x+

() nN, f(n)(0) = cos

n , () nN i f (n ) ( x ) 1 , () xR, () nN. 2

Conform corolarului 9.3.2. funcia f este dezvoltabil n serie de puteri pe R i


cos x = xn n x2 x4 x6 x 2n cos = 1 + + ... + ( 1)n + ... , () xR . 2 2! 4 ! 6 ! (2n)! n =0 n!

(9.29)

Analog obinem:
x 2n+1 x3 x5 x7 n + + ... + ( 1) + ..., () xR. sin x = x (2n + 1)! 3! 5! 7!

(9.30)

Observaie. nlocuind n (9.28) pe x cu ix, unde iC, i2=-1 i apoi cu ix i

folosind (9.29) i (9.30) obinem:


eix = 1 + x x2 2 xn i+ i + ... + in + ... = 1! 2 ! n!

x2 x4 x6 x3 x5 x7 + + ... = cos x+i sin x, () xR . = 1 + + ... + i x 1! 4 ! 6 ! 3! 5! 7!

(9.31)

Analog obinem e-ix = cos x i sin x Din (9.31) i (9.32) obinem formulele lui Euler: (9.32)

- 208 -

cos x =

e ix + e ix e ix e ix , sin x = , () xR. 2 2i

Definiia 9.3.3. Fie I R un interval, x0Int I, f :I R, indefinit derivabil

n x0. Seria de puteri f n x0.

f (n) ( x 0 ) n n ! (x x 0 ) se numete seria Taylor ataat funciei n =0

Rezultatele stabilite pentru serii Taylor ataate unei funcii f n punctul x=0 se transfer la seriile Taylor definite mai sus.

Probleme propuse
1. S se studieze convergena simpl i uniform a urmtoarelor iruri de

funcii pe intervalele indicate:


a) fn(x)=
1 , x [0, ) ; 1 + xn x , x (0, ) ; x+n

b) fn(x)=

cos nx , x R ; n2 + 1

c) fn(x)=

n 1 d) fn(x)= e kx , x [0, ) ; k =1 k

e) fn(x)=

x2 , x [1, ) ; n2 + x 4

f) fn(x)=

k x
k =1

2 k

, x[-1,1].

2. Fie fn : R + R, fn (x ) =

1 arctg x n . S se arate c (fn) converge uniform iar n

f converge neuniform. n

3. Fie fn : R R, fn (x ) =

sin kx , unde >2. k k =1


n

S se arate c (fn ) este uniform convergent pe R, iar limita sa este o funcie derivabil cu derivata continu pe R.

- 209 -

4. Fie fn : [0,1] R , fn (x ) =

nx . 1 + n2 x 2

S se arate c (fn ) converge neuniform pe [0,1], dar


lim
1 0

fn (x ) dx =

0 n

lim fn (x ) dx .

5. S se determine mulimea de convergen pentru urmtoarele serii de

funcii:
a)

n =1

(n + 1)n
nn+ x

;
1 ; n2

b)

n =1

2n sinn x ; n +1 2n x . 3n

c)

n =1

x(x 1)... (x n + 1) n!

d)

n =0

6. S se arate c urmtoarele serii sunt uniform convergente pe intervalele

indicate:
a)

n =1

1 , x (0, ) ; 2 n + x2

b)

n =1

xn , x [0, a] , unde a > 0 ; enx

c)

(x
n =1

xn 1 , x [0 , a] , a (0 , 1) ; d)

n ( 1) n =1

x2 , x R. 1 + n3 x 4

7. S se arate c seria

n =1

nx 2 este uniform convergent pe [0 , a] , unde n3 + x 3

a > 0 i lim
x 1

n =1

n x2 = n3 + x 3

n =1

n . n +1
3

8. S se arate c seria de funcii

n =1

sin n x n4 + x 2

este uniform convergent pe

R iar suma sa este continu pe R.


9. S se determine raza de convergen, intervalul de convergen i

mulimea de convergen pentru urmtoarele serii de puteri:

- 210 -

a)

n =1

1 1 + n nn

n2 + n

xn ;

b)

n =0


n =1

xn ; 2n + 3n

c)

(n !)
n =1

d)

( 1)n
n2

2n +1

xn ;

e)

(n !)2 (x 1)n ; (2n)! n =1


f)

n =1

( 2)n n ! ( x + 1)n . n n

10. S se arate c seria 1 +

( 1)n x 3n (2 3)(5 6) ...[(3n 1) 3n] n =1

este

convergent pe R iar suma sa f verific ecuaia


f ' ' (x ) + x f (x ) = 0 , x R .

11. S se determine intervalele de convergen i sumele corespunztoare

pentru urmtoarele serii de puteri:


x 3n+1 a) ( 1) ; 3n + 1 n =1

b)

n =0

(n + 1) x

( 1)n x 2n ; c) n = 0 (n + 1)(n + 3 )

d)

n(n + 1) x
n =1

12. S se arate c urmtoarele funcii sunt dezvoltabile n serii de puteri i

s se valabile

gseasc aceste dezvoltri, specificndu-se intervalul pe care sunt

a) f : R R,

f (x ) = sin3 x ;
f (x ) = ln (1 + x ) ; f (x ) = 3x ; x + 5x + 6
2

b) f : ( 1, ) R,

c) f : R \ { 2,3 } R, d) f : ( a, ) R,

f (x ) = x + a .

- 211 -

CAPITOLUL 10
INTEGRALE CU PARAMETRU

Fie R un interval, J R o mulime arbitrar de numere reale,


f : J R o funcie integrabil n raport cu x pe orice compact din , () t J.

Fie , : J . O integral de forma


F(t ) =
(t ) (t )

f (x, t ) dx ,

(10.1)

se numete integral cu parametru. Ne intereseaz s stabilim condiiile n care proprietile funciei f ( de continuitate, derivabilitate, etc.) se transmit funciei F. Fie t0J (deci punct de acumulare pentru J). Presupunem c urmtoarea limit exist i este finit :
tt0

lim f (x, t ) = (x ) , ()x I .

(10.2)

Aceasta nseamn c ( )x I i () >0 exist o vecintate V, x ( t 0 ) astfel nct () tV ,x J, t t 0 avem


f (x, t ) (x ) < .

Dac aceast vecintate este independent de x, adic () > 0, () V (t 0 ) astfel nct () t V J, t t 0 i ( ) x I avem
f (x, t ) (x ) < , spunem c

funcia f tinde uniform n t0 ctre funcia sau limita (10.2) este uniform.

- 212 -

S observm c, dac limita (10.2) este uniform i tnJ, tnt0 atunci irul de funcii (fn), unde fn(x) = f(x, tn), () n 1, converge uniform pe I ctre . Teorema 10.1. Fie f:I J R, continu n raport cu x pe I, () tJ i fie I= [, ] , F(t)= f ( x, t ) dx i t0J. Presupunem c limita (10.2) exist i este

uniform. Atunci este continu pe [, ] i


tt0

lim f ( x, t ) dx =

tt0

lim f ( x, t ) dx = ( x ) dx .

Demonstraie. Fie (tn) J, tn t 0 . Din observaia de mai sus irul de

funcii (fn), unde fn(x) = f(x,tn), converge uniform pe [, ] ctre . Cum funciile fn, n 1, sunt continue i fn u , din proprietatea de transfer de continuitate rezult c este continu. Fie > 0; cum limita (10.2) este uniform, exist > 0 astfel nct () tJ,
t t 0 , t t 0 < i () xI avem

f (x, t ) ( x ) <

Vom avea

f ( x, t ) dx (x )dx f (x, t ) (x ) dx

dx = ,

() t J,

t t 0 , cu t t 0

< , i demonstraia este ncheiata.

n continuare presupunem c I = [a, b], J = [c, d].


Teorema 10.2. Fie f:[a, b] x [c, d] R, continu, , : [c, d] [a, b] , continue.

Atunci funcia F definit de (4.1) este continu pe [c, d].


Demonstraie. Fie t0[c, d], fixat i t[c, d] arbitrar.

Evident avem
F( t ) = ( t ) f ( x, t )dx = ( t ) f ( x, t )dx + ( t ) f ( x, t )dx + ( t ) f ( x, t )dx ,de unde
0 0

( t )

( t 0 )

( t 0 )

( t )

(10.3)

F( t ) F( t 0 ) =

( t 0 ) ( t )

f ( x, t )dx +

( t 0 ) ( t 0 )

f ( x, t )dx +

( t ) ( t 0 ) ( t 0 ) ( t 0 )

f ( x, t )dx

( t 0 ) ( t 0 )

f ( x, t 0 )dx

( t 0 ) ( t )

f ( x, t )dx +

( t ) ( t 0 )

f ( x, t )dx +

( f ( x, t ) f ( x, t 0 ))dx .

(10.4)

- 213 -

Cum f este continu pe [a, b] x [c, d] ( =compact ) rezult c f este mrginit pe [a, b] x [c, d] , deci ()M > 0 astfel nct
f ( x, t ) M, ()( x, t ) [a, b] [c, d] .

Fie > 0 . Cum i sunt continue pe [c,d] exist ' > 0 astfel nct

() t [c, d]

cu t t 0 < ' avem

( t ) ( t 0 ) <

, 3M

( t ) ( t 0 ) <

. 3M

Cum f este continu pe [a, b] [c, d] din teorema lui Cantor rezult c f este uniform continu pe [a, b] [c, d] i atunci () '' > 0 astfel nct () x [a, b] i

() t [c, d] cu

t t 0 < '' avem


f ( x, t ) f ( x, t 0 ) ( t 0 ) ( t 0 ) < . 3

Din (10.4) vom avea


F( t ) F( t 0 ) M ( t ) ( t 0 ) + M ( t ) ( t 0 ) +
< M

dx < ( t 0 ) 3 ( t ) ( t ) 0 0
( t 0 )

+ M + = , () t [c, d] , cu t t 0 < , unde = min ( ' , '' ) . 3M 3M 3

Rezult c F este continu n t0 i cum t0 este fixat dar arbitrar luat rezult c F este continu pe [c,d].

Teorema 10.3. Fie f : [a, b] [c, d] R , continu, , : [c, d] [a, b] , derivabile.

Presupunem c f admite pe [a, b] x [c, d] derivat parial n raport cu t, continu pe [a, b] [c, d] . Atunci F este derivabil pe [c,d] i pentru ( ) t [c, d] avem
F' ( t ) = ( t )
( t )

f ( x, t )dx + f (( t ), t )' ( t ) f (( t ), t )' ( t ) . t

Demonstraie. Fie t0[c,d], fixat i t[c,d], t t0.

Folosind descompunerea din (10.3) vom avea


( t ) f ( x, t ) f ( x, t ) F( t ) F( t 0 ) 1 0 dx + = ( t ) t t0 t t0 t t0
0 0

( t ) ( t 0

f ( x, t )dx )

1 t t0

( t ) ( t 0 )

f ( x, t )dx (10.5)

- 214 -

Fie funcia : [a, b] [c, d] R


f ( x, t ) f ( x, t 0 ) , t t0 t t0 ( x, t ) = f ( x, t 0 ), t = t 0 t

Din ipotezele teoremei rezult c funcia este continu pe [a, b] x [c, d]. Folosind teorema 10.2 rezult c funcia G : [c, d ] R , G( t ) = ( t ) ( x, t )dx este
0

( t 0 )

continu pe [c,d], deci


lim ( t ) ( x, t )dx = lim G( t ) = G( t 0 ) = ( t ) ( x, t 0 )dx = ( t
t t 0
0

( t 0 )

( t 0 )
0

( t 0 )
0)

t t 0

f ( x, t 0 )dx . t

Cum pentru t t0, ( x, t ) =


lim
( t 0 )

f ( x, t ) f ( x, t 0 ) , rezult c exist t t0

tt0

f ( x, t ) f ( x, t 0 ) dx = ( t 0 ) t t0

f ( x, t 0 )dx ( t 0 ) t
( t 0 )

(10.6)

Ne ocupm n continuare de urmtoarele dou integrale din (10.5). Din teorema de medie exist un punct ct ntre (t0) i (t) astfel nct
1 t t0 ( t ) ( t 0 ) f (c t , t ) i folosind derivabilitatea funciei n t0 i t t0

( t ) ( t 0 )

f ( x, t )dx =

continuitatea lui f rezult c exist


lim
tt0

1 t t0 1 t t0

( t ) ( t 0 )

f ( x, t )dx = ' ( t 0 )f (( t 0 ), t 0 )

(10.7)

Raionnd analog gsim c


lim
tt0 ( t ) ( t 0 )

f ( x, t )dx = ' ( t 0 )f (( t 0 ), t 0 )

(10.8)

Din (10.6), (10.7), (10.8) rezult derivabilitatea funciei F i egalitatea din enun.
Exemplu. S se calculeze integrala I =
1 0

ln(1 + x ) dx , folosind integrala cu 1+ x2

parametru I ( ) =

ln(1 + x ) dx . 1+ x2

Evident avem I = I(1). Fie f : [0,1] [0,1] R, f ( x, ) =


ln(1 + x ) i atunci I( ) = 1+ x2

f ( x, )dx .

- 215 -

Evident f satisface ipotezele teoremei 10.3, deci funcia I este derivabil pe


f ( x, )dx + f (, ) = [0,1] i I' ( ) = 0

x ln(1 + 2 ) 0 (1 + x )(1 + x 2 ) dx + 1 + 2 .

Calculnd prima integral, obinem


1 ln(1 + 2 ) I ' ( ) = + arctg , de unde, prin integrare obinem 2 2 1+ 1+ 2
I ( ) = I ' ( )d = 1 arctg ln(1 + 2 ) + c . 2

Cum I(0) = 0 rezult c = 0, deci


I ( ) =

1 1 arctg ln(1 + 2 ) i atunci I = I(1) = arctg1 ln 2 = ln 2 . 2 2 8

n continuare considerm integrale cu parametru pe intervale necompacte. Fie integrala


F( t ) = a f ( x, t )dx , unde f : [a, b) x [c, d] R.
b

(10.9)

Presupunem c integrala (10.9) este convergent,

() t [c, d] .

Ne

propunem s dm condiii de continuitate i derivabilitate pentru funcia F. Un rol important n formularea acestor condiii l are noiunea de convergen uniform a unei integrale cu parametru pe un interval necompact. Convergena integralei (10.9), pentru

() t [c, d ] revine la urmtoarea

condiie: pentru () t [c, d ] i () > 0, () c ,t (a,b) astfel nct () u (c ,t , b) avem


F( t ) f ( x, t )dx < .
a u

(10.10)

Dac n formularea de mai sus c,t este independent de t, adic

() > 0, () c (a , b)

astfel nct () u (c , b) i () t [c, d ] avem (10.10), spunem

c integrala (10.9) converge pe [a,b) uniform n raport cu t[c,d].


Teorema 10.4. Dac integrala (10.9) converge uniform pentru t[c,d] atunci

irul de funcii
Fn ( t ) = a f ( x, t )dx , unde
a < bn < b,
n

bn

(10.11)

lim bn = b , converge uniform pe [c,d].

- 216 -

Demonstraie. Fie > 0. Conform ipotezei

() c (a, b)

astfel nct

() u (c , b)
Cum vom avea

i () t [c, d ] avem (4.10).

lim bn = b , () n N astfel nct

()n n

avem c < b n < b i atunci

F( t )

bn a

f ( x, t )dx < , () t [c, d ], () n n , adic

Fn ( t ) F( t ) < , () n n , () t [c, d] , deci Fn u F pe [c,d].

Teorema 10.5. Dac f : [a, b) [c, d] R este continu i integrala (10.9)

converge uniform pentru t[c,d] atunci funcia F definit de (10.9) este continu pe [c,d].
Demonstraie. Fie b n < b, b n b . Din teorema 10.4 irul (Fn) definit de

(4.11) este uniform convergent pe [c,d] ctre F. Din teorema 10.2 aplicat funciei f pe [a, b n ] [c, d] i folosind faptul c f este continu rezult c Fn este continu pe [c,d]. Cum Fn u F pe [c,d], din teorema 10.1.5 rezult c funcia F este continu pe [c,d].

Teorema 10.6. Fie f : [a, b) [c, d] R . Presupunem c f este continu pe


[a, b) [c, d ] , derivabil parial n raport cu t pe [a, b) x [c, d] cu

f continu pe t

[a, b) [c, d] .

Presupunem c integrala iar integrala

b a

f ( x, t )dx este convergent pentru () t [c, d] ,

b a

f ( x, t )dx este uniform convergent pentru t[c,d]. t

Atunci F definit de (10.9) este derivabil pe [c,d] i


F' ( t ) =

b a

f ( x, t )dx, () t [c, d] . t

n plus F este continu pe [c,d].

- 217 -

Demonstraie. Fie b n < b, b n b . Avem [a, b n ] [c, d] [a, b) [c, d] . Fie


Fn ( t ) = a f ( x, t )dx .
bn

Folosind teorema 10.3 rezult c Fn este derivabil pe [c,d] i


F' n ( t ) = a
bn

f ( x, t ) dx, ()t [c, d] . t f este continu pe [a, b n ] [c, d] , din teorema 10.2 rezult c Fn este t
b a

Cum

continu pe [c,d]. Din convergena integralei

f ( x, t )dx rezult Fn F pe [c,d].

Din uniform convergena integralei a irului (Fn) pe [c,d].

f ( x, t )dt rezult convergena uniform a t


b

Conform teoremei 9.1.7 funcia F este derivabil pe [c,d] i F' ( t ) = lim F' n ( t ) = lim a
n n bn b f f ( x, t )dx = a ( x, t )dx . t t

Continuitatea funciei F rezult din teorema de transfer de continuitate de la iruri de funcii.

Vom da n continuare dou criterii de convergen uniform pentru integrale cu parametru pe un interval necompact.
Teorema 10.7. Fie f : [a,b) [c, d ] R . Presupunem c integrala
F( t ) = a f ( x, t )dx este convergent, () t [c, d] .
b

O condiie necesar i suficient pentru ca integrala

f ( x, t )dx s convearg

uniform n raport cu t [c, d] este ca () > 0 s existe c (a, b) astfel nct

()u, v (c , b),u < v

s avem

v u

f ( x, t )dx < , () t [c, d ] .

Demonstraie. Se folosete un raionament asemntor cu cel din

demonstraia criteriului lui Cauchy (vezi teorema 8.1.1)

- 218 -

Teorema 10.8. Fie f : [a, b) [c, d] R . Presupunem c integrala


F( t ) = a f ( x, t )dx este convergent, () t [c, d] i exist o funcie : [a, b) R
b

astfel nct
i) ii)
f ( x, t ) ( x ) , () x [a, b), () t [c, d ] ;

integrala

( x )dx este convergent.

Atunci integrala bf ( x, t )dx converge uniform pe [a,b) n raport cu t[c,d]. a


Demonstraie. Fie > 0 . Din convergena integralei

( x )dx , folosind

criteriul lui Cauchy,

()c (a,b)

astfel nct

()u, v (c ,b),u < v

avem

v u

( x )dx < .

Pe de alt parte, din i), pentru () t [c, d] vom avea

v u

f ( x, t )dx f ( x, t ) dx ( x )dx < , i demonstraia este ncheiat folosind


u u

teorema 10.7.
Teorema 10.9. (Criteriul lui Dirichlet). Fie

f : [a, b) [c, d] R continu,

g : [a, b) [c, d] R , monoton descresctoare n raport cu x pe [a,b), () t [c, d] .

Presupunem c exist M > 0 astfel nct

u a

f ( x, t )dx M, ()t [c, d ] , ()u [a, b) i lim g( x, t ) = 0 , uniform n raport cu


x b x <b

t [c, d] . Atunci integrala t [c, d] .

f ( x, t )g( x, t )dx este uniform convergent n raport cu

Demonstraie. Analog cu demonstraia teoremei 10.1.2.

Teorema 10.10. (Criteriul lui Abel). Fie f, g : [a, b) [c, d] R , f continu, g

monoton n raport cu x pe [a,b), () t [c, d ] i exist M > 0 astfel nct


g( x, t ) M , () x [a, b) , () t [c, d] .

Presupunem c integrala
t [c, d] .

f ( x, t )dx este uniform convergent n raport cu

- 219 -

Atunci integrala
t [c, d] .

f ( x, t )g( x, t )dx este uniform convergent n raport cu

Exemple. 1. S se arate c integrala

arctg x dx, > 0 este convergent x(1 + x 2 )

i s se calculeze. Fie F( ) =
0

arctg x dx , > 0 . x(1 + x 2 )

Avem F( ) = 0 f (x, ) dx , unde f ( x, ) = Cum


f ( x, )

arctg x . x (1 + x 2 )

x = , () x > 0 , > 0 2 x (1 + x ) 1 + x 2

dx 1+ x2

este este

convergent rezult, conform propoziiei 10.1.2, c integrala convergent. Evident avem Cum
f 1 = i 2 (1 + x )(1 + 2 x 2 )

f (x, ) dx
0

f 1 ( x, ) , () x > 0, > 0 . 1+ x2

1 dx este convergent rezult c 1+ x2

f ( x, )dx converge

uniform pe [0, ) n raport cu > 0. Folosind teorema 10.6 funcia F este derivabil i
F' ( ) = 0
f dx ( x, )dx = 0 = 2 (1 + x )(1 + 2 x 2 )

2 dx 1 = 2 2 0 2 2 1 1 + x

dx = 2 1 + x 2( + 1)

de unde F( ) =

ln( + 1) + c . 2
ln( + 1) , () > 0 . 2

Pentru 0 avem F() 0 , deci c = 0 i atunci F( ) =


2. S se calculeze integrala F( t ) = 0 e tx

sin3 x dx i s se deduc apoi x

valoarea integralei

sin 3 x dx . x

- 220 -

Avem F( t ) = f ( x, t )dx , unde f ( x, t ) = e tx 0 Se arat imediat c integralele

sin 3 x , pentru x > 0 i t 0. x

tx

sin 3 x dx , x

f ( x, t )dx = 0 e tx sin3 xdx t

sunt uniform convergente pe (0,) n raport cu t[0,) i atunci F este derivabil pe [0,) i F' ( t ) = 0 e tx sin3 xdx . Calculnd ultima integral obinem F' ( t ) = 1 t 3 1 1 3 1 , de unde F( t ) = arctg arctgt + c . 2 2 12 3 4 4 t + 9 4 t +1
t 3 1 , deci F( t ) = i arctg arctgt + 3 3 4 3 12

Cum lim F( t ) = 0 deducem c = t atunci

sin 3 x dx = . x 3

Probleme propuse
1. S se calculeze F(t), unde F( t ) = f ( x t, x + t ) dx , iar f : R 2 R este de at
bt

clas C1 pe R2.
2. S se calculeze urmtoarele integrale folosind derivarea sub integral: a)

2 0

1 1 + y cos x ln dx , y ( 1,1) ; cos x a y cos x

b)

1 0

ln(1 y 2 x 2 ) x2 1 x2

dx , y < 1.

3. Fie funcia : [a, b] R , continu i : [a, b] R ,


( t ) = 1 t a ( x ) sin k(t x )dx , unde k R . k

S se arate c

' ' ( t ) + k 2 ( t ) = ( t ), ()t [a, b] .

4. Fie g: R R, derivabil, f : R R de dou ori derivabil i


2 2 x + ay 1 [f ( x ay ) + f ( x + ay )] + 1 x ay g( t )dt , a 0 . Atunci F a 2 F = 0 . F( x, y ) = 2 2a y 2 x 2

- 221 -

5. S se calculeze urmtoarele integrale folosind derivarea sub integral: a) c) e)

2 0 2 0

ln(cos 2 x + y 2 sin 2 x )dx, y > 0 ; arctg( y sin x ) dx ; sin x cos ax cos bx dx, a > 0, b > 0 . x

b) d)

2 0

1 ln(1 + y cos x )dx, y < 1 ; cos x sin tx dx ; x

- 222 -

CAPITOLUL 11
INTEGRALE CURBILINII

11.1. Integrale curbilinii de spea nti


Definiia 11.1.1. Se numete drum parametrizat (sau curb parametrizat, arc de curb parametrizat) n spaiul R3 orice funcie continu : [a, b] R 3 ,
( t ) = ( x( t ), y( t ), z(t )) , t[a,b].

Punctele A( x(a), y(a), z(a)), B( x(b), y(b), z(b)) se numesc extremitile curbei . Pentru curba vom mai folosi notaia AB . Vom nota cu ( ) = Im = { ( x( t ), y( t ), z( t )) R3 : t [a, b]}, imaginea curbei .

Fig.1 Ecuaiile x = x( t ), y = y( t ), z = z( t ) reprezint ecuaiile parametrice ale curbei


i scriem

x = x( t ) ( ) : y = y( t ), t [a, b] z = z( t )

(11.1)

- 223 -

Dac B = { i, j, k } este o baz ortonormat n reperul cartezian Oxyz, atunci curba poate fi dat i astfel
( ) : r = r ( t ), t [a, b]

(11.2) de poziie al punctului

unde

r ( t ) = x( t )i + y( t ) j + z( t )k

este

vectorul

M( x( t ), y( t ), z( t )) .

Reprezentarea (11.2) se numete ecuaia vectorial parametric a curbei . Dac z( t ) = 0, ()t [a, b] , curba se numete curb plan, deci
x = x( t ) () : , t [a, b] . y = y( t )

n ipotezele n care poate fi eliminat parametrul t obinem


( ) : F( x, y ) = 0, ( x, y ) D R2 ,

forma implicit. forma explicit.

Dac pe D sunt ndeplinite ipotezele teoremei funciilor implicite rezult:


( ) : y = y( x ), xJ R,

x = x( t ) Fie ( ) : y = y( t ) , t [a, b] , o curb oarecare n spaiu. z = z( t )

Definiia 11.1.2. Curba


x, y, z C1 ([a, b])

se numete neted (sau regulat) dac sau echivalent

x'2 ( t ) + y'2 ( t ) + z'2 ( t ) 0, ()t [a, b] ,

r ' ( t ) , () t [a, b] .

Curba se numete neted pe poriuni dac este o reuniune finit de arce netede, adic dac ()A1, A 2 ,..., A n 1 ( ) astfel nct AA 1, A 1A 2 ,..., A n 1B s fie netede. Curba se numete simpl dac funcia t r ( t ) este injectiv pe [a.b) . Curba se numete nchis dac A = B , adic r (a) = r (b) . Fie D ([a,b]), = (a = t 0 < t1 < ... < t n = b) i
A k ( x( t k ), y( t k ), z( t k )) ( ), k = 0, n, A 0 = A, A n = B , punctele corespunztoare de pe

curb.

- 224 -

Punctele A, A1,..., A n 1, B determin o linie poligonal ale crei vrfuri sunt situate pe ( ) . Lungimea acestei linii poligonale este
L = ( x( t k ) x( t k 1 ))2 + ( y( t k ) y( t k 1 ))2 + ( z( t k ) z( t k 1 ))2 .
k =1 n

Definiia 11.1.3. Curba se numete rectificabil (sau spunem c are

lungime) dac mulimea { L : D([a,b])} este majorat. n acest caz numrul notat L( ) = sup L se numete lungimea curbei .

Observaie. n cazul curbelor plane aceast definiie coincide cu definiia

7.3.5 dat pentru lungimea graficului unei funcii f : [a.b] R .


Teorema 11.1.1. Fie o curb neted dat prin ecuaiile parametrice

(11.1). Atunci curba este rectificabil i


L( ) =
b a

( x' ( t ))2 + ( y' ( t ))2 + z' ( t ))2 dt .

(11.3)

Demonstraie. Fie D([a,b]), = (a = t 0 < t1... < t n = b) .

Atunci L = ( x( tk ) x( t k 1 ))2 + ( y( tk ) y( tk 1 ))2 + ( z( tk ) z( tk 1 ))2 .


k =1

Din teorema lui Lagrange aplicat funciilor x, y, z pe intervalul [ t k 1,t k ] rezult c exist k , k , k ( t k 1, t k ) astfel nct:
L = x'2 (k ) + y'2 (k ) + z'2 ( k ) ( t k t k 1 )
k =1 n

Cum x, y, z C1 ([a, b]) , funciile x' , y' , z' sunt continue pe [a,b], deci mrginite i fie M > 0 astfel nct x' ( t ) M,
n

y' ( t ) M,

z' ( t ) M, ()t [a, b] .

Rezult L M 3 ( tk t k 1 ) = M 3 (b a) , deci curba este rectificabil.


k =1

S artm n continuare (11.3).

- 225 -

Vom scrie L astfel:


L = x ' 2 ( t k ) + y ' 2 ( t k ) + z' 2 ( t k ) ( t k t k 1 ) +
n k =1

[ x' ( ) + y' ( ) + z' ( ) x' (t ) + y' (t ) + z' (t ) ](t t ) = = (, t ) + [ x' ( ) + y' ( ) + z' ( ) x' ( t ) + y' ( t ) + z' ( t ) ]( t t
+
k =1 n 2 2 2 2 2 2 k k k k k k k k 1 n 2 2 2 2 2 2 k k =1 k k k k k k k

k 1

unde : [a, b] R, ( t ) = x' 2 ( t ) + y' 2 ( t ) + z' 2 ( t ) este o funcie continu pe [a,b].


Fie I = a x' 2 ( t ) + y' 2 ( t ) + z' 2 ( t )dt . Vom avea
L I ( , t k ) I + L ( , t k ) ( , t k ) I + +
n

k =1

x '2 ( k ) + y '2 ( k ) + z'2 ( k ) x '2 ( t k ) + y '2 ( t k ) + z'2 ( t k ) ( t k t k 1 )

Folosind inegalitatea
a2 + b2 + c 2 m2 + n2 + p2 a m + b n + c p , ()a, b, c, m, n, p R ,

obinem
L I (, t k ) I + + ( x' (k ) x' ( t k ) + y' (k ) y' ( t k ) + z' ( k ) z' ( t k ) )( t k t k 1 ).
k =1 n

Fie > 0. Cum este continu pe [a,b] este integrabil i atunci


() ' > 0 astfel nct () D([a,b]), = (a = t 0 < t1 < ... < t n = b) cu

< ' i

()c k [ t k 1, t k ] , k = 1, n avem

(, c k ) I <

i n particular, pentru c k = t k , 2

k = 1, n vom avea
(, t k ) I < 2

(11.4)

Cum x' , y' , z' sunt continue pe [a,b] rezult c sunt uniform continue i atunci
() '' > 0

astfel

nct

() t' , t' ' [a, b]

cu

t' t' ' < ''

avem (11.5)

x' ( t ' ) x' ( t' ' ) <

, y' ( t ' ) y' ( t' ' ) < , z' ( t ' ) z' ( t ' ' ) < 6(b a) 6(b a) 6(b a)

- 226 -

Fie = min( ' , '' ) i D([a,b]), = (a = t 0 < t1 < ... < t n = b) cu < . Evident avem
k t k < ,

k t k < , k t k < , k = 1, n i

folosind condiiile (11.5) vom avea

(
n k =1 n

x' (k ) x' ( t k ) + y' (k ) y' ( t k ) + z' ( k ) z' ( t k )

) (t

t k 1 ) <

< 6(b a) + 6(b a) + 6(b a) (t k t k 1 ) = 2 k =1

(11.6)

Folosind (11.4) i (11.5) obinem


L I < , () D([a,b]) cu < , deci

L( ) = I =

b a

x'2 ( t ) + y'2 ( t ) + z'2 ( t )dt .

Exemplu. S se calculeze lungimea arcului de elice


x = a cos t ( ) : y = a sin t , t [0,2] , unde a > 0 z=t

Vom avea
L( ) = =
2 0

x'2 ( t ) + y'2 ( t ) + z'2 ( t )dt =

2 0

a2 sin2 t + a2 cos2 t + 1dt =

2 0

a2 + 1dt = 2 a2 + 1.

Observaie. Fie o curb neted dat prin ecuaiile parametrice (11.1).

Dac M( x( t ), y( t ), z( t )) ( ) cu t [a, b] este un punct arbitrar de pe curba i notm cu s( t ) = L( AM) , conform teoremei 11.1.1,
s( t ) = a x' 2 ( ) + y' 2 ( ) + z' 2 ( )d .
t

S observm c funcia s este cresctoare, derivabil cu derivata continu pe [a,b] i s' ( t ) = x'2 ( t ) + y'2 ( t ) + z'2 ( t ), ()t [a, b] , de unde
ds = x'2 ( t ) + y'2 ( t ) + z'2 ( t )dt , care se mai numete i element de arc.

- 227 -

Vom defini n continuare integrala curbilinie de spea nti sau n raport cu arcul. Fie o curb rectificabil dat prin ecuaiile parametrice (11.1). Fie funcia f : D R , unde D R 3 astfel nct ( ) D . Fie D([a,b]), = (a = t 0 < t1 < ... < t n = b) i A k ( x( t k ), y( t k ), z( t k )), k = 0, n , A = A 0 , B = A n , punctele corespunztoare de pe curba . Fie sk = L( AA k ), k = 0, n , numit i coordonata curbilinie a punctului A k . S observm c sn = L( AB) = L( ) = L, s0 = 0 . Fie Mk ( k , k , k ) A k 1A k , k = 1, n , unde k = x( k ), k = y( k ), k = z( k ) , cu
k [ t k 1, t k ], k = 1, n .

Fie k = L( AMk ), k = 1, n , coordonata curbilinie a lui Mk . Astfel un punct oarecare Mk A k 1A k poate fi precizat fie prin coordonata sa curbilinie k , fie prin coordonatele carteziene k ,k , k . Evident avem k [s k 1, s k ], k = 1, n . Pentru diviziunea = (a = t 0 < t1 < ... < t n = b) a intervalului [a, b] obinem o diviziune s = (0 = s0 < s1 < ... < sn = L ) a intervalului [0, L] . Considerm suma ( f , Mk ) = f (Mk )(sk sk 1 ) , numit sum integral
k =1 n

curbilinie a funciei f corespunztoare diviziunii s i punctelor intermediare Mk (prin f (Mk ) nelegem f ( k , k , k ) ).


Definiia 11.1.4. Funcia f este integrabil pe dac ()I R astfel nct

() > 0, () > 0 astfel nct

() D([a,b]),

= (a = t 0 < t1 < ... < t n = b) cu

< i ()Mk A k 1A k , unde A k ( x( t k ), y( t k ), z( t k )), k = 1, n avem ( f, Mk ) I < .

- 228 -

Observaie. Ca i n cazul integralelor definite (capitolul 7) se arat c

numrul real I din definiie, n ipoteza c exist, este unic i prin definiie I se numete integrala curbilinie de spea nti a funciei f pe curba i se noteaz:
I = f ( x, y, z )ds (sau

f ( x, y, z)ds , fds )
AB

S observm c f ( x, y, z )ds = lim ( f ,Mk ) .


0

Proprieti ale integralei curbilinii de spea nti


Pornind de la proprietile integralei Riemann, se obin cu uurin urmtoarele proprieti ale integralei curbilinii de spea nti.
1. Dac f i g sunt integrabile pe i , R atunci f + g este integrabil

pe i

(f + g)ds = fds + gds


2. Fie () = AB i C AB . Dac f este integrabil pe AC i pe CB atunci f

este integrabil pe AB i

f ds = f ds + f ds .
AB AC CB

3. Dac f este integrabil pe i f 0 atunci

f ds 0 .

Consecin. Dac f, g sunt integrabile pe i f g atunci

f ds g ds .

Vom da n continuare o formul de calcul al integralelor curbilinii de spea nti.


Teorema 11.1.2. Fie o curb neted dat prin ecuaiile parametrice

(11.1) i fie f: DR, continu, unde D R3 astfel nct () D. Atunci funcia f este integrabil pe i

- 229 -

f ( x, y, z) ds =

b a

f ( x( t ), y( t ), y( t )) x' 2 ( t ) + y' 2 ( t ) + z' 2 ( t ) dt .


b

Demonstraie.

Fie I = a f ( x( t ), y( t ), z( t )) x' 2 ( t ) + y' 2 ( t ) + z' 2 ( t ) dt ,

D([a,b]), = (a = t 0 < t 1 < L < t n = b ) , k [t k 1, t k ], k = 1,n , A k (x( t k ), y( t k ), z( t k )) , k = 0,n , Mk (x( k ), y( k ), z( k )) A k 1A k , k = 1,n , sk = L(AAk). Avem ( f ,Mk ) = f (x( k ), y( k ), z( k ))(s k s k 1 ) .
k =1 n

Cum funciile x, y, z sunt continue, din teorema de medie ()k [t k 1, t k ],


k = 1 n astfel nct ,
sk sk 1 =

tk t k 1

x'2 ( ) + y'2 ( ) + z'2 ( ) d =

= x'2 (k ) + y'2 (k ) + z'2 (k ) ( t k t k 1 ).

Fie : [a,b]R, ( t ) = f (x( t ), y( t ), z( t )) , care evident este o funcie continu. Vom avea
( f , Mk ) = (k ) x'2 (k ) + y'2 (k ) + z'2 (k ) ( t k t k 1 ) =
k =1 n

= (k ) x'2 (k ) + y'2 (k ) + z'2 (k ) ( t k t k 1 ) +


k =1 n

(11.7)

+ (k )
k =1

[ x ' ( ) + y ' ( ) + z' ( )


2 2 2 k k k

x ' 2 ( k ) + y ' 2 ( k ) + z ' 2 ( k ) ( t k t k 1 )

Prima sum din ultimul membru al egalitilor (11.7) reprezint o sum Riemann corespunztoare funciei x' 2 + y' 2 + z' 2 , diviziunii i punctelor intermediare k. Cum funcia x' 2 + y' 2 + z' 2 este continu, va fi integrabil pe [a,b] i

atunci, dac nD([a,b]) este un ir de diviziuni cu n 0 va rezulta c


n

lim n ( f , n ) = lim (n ) x'2 (n ) + y'2 (n ) + z'2 (n ) ( t n t n 1 ) = I . k k k k k k k


n k =1

pn

Folosind uniform continuitatea funciei x' 2 + y ' 2 + z ' 2 va rezulta c a doua sum din ultimul membru al egalitilor (11.7) are limita 0 pentru n 0 .

- 230 -

n concluzie, dac nD([a,b]) cu n 0 rezult c lim ( f ,Mn ) = I , deci k


n
n

f este integrabil pe i

f (x, y, z )ds = I = f (x( t ), y(t ), z( t ))


b a

x' 2 ( t ) + y' 2 ( t ) + z' 2 ( t ) dt

x = x( t ) Observaie. n cazul curbelor plane (): , t [a,b] vom avea y = y( t )

f ( x, y ) ds = a f (x(t ), y(t ))
b

x' 2 ( t ) + y' 2 ( t ) dt

iar n cazul unei reprezentri explicite (): y = y(x), x [,],

f ( x, y ) ds = f (x, y( x ))

1 + y' 2 ( x ) dx .

Exemple. 1. S se calculeze integrala


x = a cos t 1 I= 2 ds , unde ( ) : y = a sin t , t [0,]. 2 2 x + y + z z = b t

S observm c ipotezele teoremei 11.1.2 sunt ndeplinite i atunci


I=
0

1 a cos t + a2 sin2 t + b2 t 2
2 2 2

a2 sin2 t + a2 cos2 t + b2 dt =

1 = a +b 2 dt = 0 a + b2 t 2
2

bt a2 + b2 = arctg a 0 ab

b a2 + b2 . arctg a ab

2. S se calculeze integrala
x2 y2 I = x y ds , unde ( ) : 2 + 2 = 1, x 0, y 0, iar a, b > 0, a b . a b

Curba () este un arc de elips care se scrie parametric astfel:


x = a cos t () : y = b sin t, t 0, 2

i atunci:

- 231 -

I = 02 a b sin t cos t a 2 sin 2 t + b 2 cos 2 t dt =


ab 2 1 + cos 2t 2 1 cos 2t dt = = + b2 0 sin 2t a 2 2 2 ab 2 b2 a2 a2 + b2 sin 2t cos 2t + dt = = 2 0 2 2 1 2 2 2 2 2 2 2 2 2 b a ab a +b b a a +b 2 dt = cos 2t + = 2 2 0 2 cos 2t + 2(b a ) 2 2 2

b2 a2 a2 + b2 2 cos 2t + 2 ab = 2 2 3 2(b a ) 2

3 2

=
0

3 b2 a2 a2 + b2 2 b2 a2 a2 + b2 ab = + + 2 2 3 (a b ) 2 2 2 2 ab a b (a 2 + a b + b 2 ) 3 3 (a b ) = = 3 (a 2 b 2 ) 3 (a + b )

3 2 =

Aplicaii ale integralei curbilinii de spea nti 1. Lungimea unei curbe rectificabile
n definiia integralei curbilinii de spea nti, lund f(x,y,z)=1 obinem
(f , Mk ) = (sk sk 1 ) = L( ) , de unde prin trecere la limit cu
k =1 n

obinem L( ) = ds .

2. Aplicaii mecanice
Considerm un fir material de grosime neglijabil, care este imaginea unei curbe netede din R3. Considerm firul neomogen, la care densitatea = (x,y,z) > 0 este o funcie continu de coordonatele punctului de pe curb.

- 232 -

Din considerente de mecanic se poate arta c masa M i coordonatele centrului de greutate G(xG,yG,zG) sunt date de
M = (x, y, z ) ds , x G =

1 x (x, y, z ) ds , M

yG =

1 1 y (x, y, z ) ds , z G = M z (x, y, z ) ds . M

Momentele de inerie ale firului fa de planele de coordonate, axe i respectiv pol vor fi date de
IxOy = z 2 (x, y, z ) ds , IyOz = x 2 (x, y, z ) ds , IxOz = y 2 (x, y, z ) ds ,

IOx = (y 2 + z 2 )(x, y, z ) ds , IOy = (x 2 + z 2 ) (x, y, z ) ds ,


IOz = (x 2 + y 2 )(x, y, z ) ds , IO = (x 2 + y 2 + z 2 )(x, y, z ) ds .


11.2. Integrale curbilinii de spea a doua (sau n raport cu coordonatele)


x = x( t ) Fie ( ) = AB : y = y( t ) , t [a,b] , z = z( t )

(11.8)

o curb simpl, rectificabil, de extremiti A (x(a), y(a), z(a)) i B(x(b), y(b), z(b)) . Fie F : D V3 o funcie vectorial de componente P,Q,R: D R, unde D R3 astfel nct ( ) D . Presupunem c F este mrginit pe D, adic P,Q,R sunt mrginite pe D. Fie D([a,b]), = (a = t 0 < t 1 < L < t n = b ) i punctele corespunztoare de pe curb A k (x k , y k , z k ) , k = 0,n , A0 = A, An = B, unde x k = x(t k ) , y k = y(t k ) , z k = z(t k ) , k = 0,n . Fie Mk ( k , k , k ) A k 1A k ,
k [t k 1, t k ]. k = 1, n unde k = x( k ) , k = y (k ) , k = z( k ) ,

- 233 -

Considerm suma
F, Mk = [ P(Mk )(xk xk 1 ) + Q(Mk )(y k yk 1 ) + R(Mk )(zk zk 1 ) ] ,
k =1

numit sum integral curbilinie a funciei F corespunztoare diviziunii i punctelor intermediare M k.


Definiia 11.2.1. Funcia F este integrabil pe curba dac ()I R astfel

nct () > 0, () > 0 astfel nct ( ) D([a,b]), = (a = t 0 < t 1 < L < t n = b ) , cu < i ()Mk A k 1A k , unde A k (x(tk ) , y(t k ), z(tk )), k = 1, n avem
F, Mk I < .

Observaie. Se arat ca i n cazul integralei definite (capitolul 7) c

numrul real I din definiie, n ipotez c exist, este unic i prin definiie I se numete integrala curbilinie de spea a doua a funciei F pe curba i se noteaz:
I = P( x, y, z ) dx + Q( x, y, z ) dy + R( x, y, z ) dz (sau P dx + Q dy + R dz )

S observm c I = lim F, Mk .
0

Dac r = x i + y j + zk este vectorul de poziie al punctului curent M (x,y,z) din spaiu i dr = dx i + dy j + dz k atunci I = F(x, y, z ) dr

sau F dr .

Observaii. 1. Din punct de vedere fizic I reprezint lucrul mecanic

efectuat de fora variabil F de-a lungul arcului AB . Astfel numete circulaia vectorului F de-a lungul arcului AB .

F(x, y, z )dr

se mai

2. Dac curba este nchis, adic A = B atunci se mai numete contur i

integrala se mai noteaz


I = F(x, y, z ) dr .

- 234 -

3. Un punct M(x (t ), y (t ), z(t )) ( ) parcurge curba ntr-un sens pe care l

numim direct atunci cnd t parcurge continuu intervalul [a,b] de la a la b. Cnd t parcurge intervalul [a,b] de la b la a, punctul corespunztor M parcurge n sens invers. O curb mpreun cu unul din sensurile de parcurgere se numete curb orientat. Curba mpreun cu sensul direct de parcurgere se noteaz cu + i n mod asemntor se definete . Dac = AB scriem + = AB, = BA . n plan se consider de obicei sensul direct cel trigonometric. Din definiie se observ c, dac se schimb sensul de parcurs pe arcul
AB atunci diferenele x k x k 1, y k y k 1, z k z k 1 i schimb semnul deci:
AB

F(x, y, z ) dr = F(x, y, z ) dr .
BA

Alte proprieti ale integralei curbilinii de spea a doua care se obin cu uurin pornind de la proprietile integralei Riemann sunt date de
Propoziia 11.2.1. i) Dac F, G sunt integrabile pe i , R atunci F + G este

integrabil pe i

(F + G)dr = F dr + G dr ;

ii) Dac C AB i F este integrabil pe arcele AC i pe CB atunci F

este integrabil pe AB i
AB

F dr = F dr + F dr .
AC CB

- 235 -

Vom da n continuare o formul de calcul pentru integralele curbilinii de spea a doua.


Teorema 11.2.1.

Fie o curb neted dat prin ecuaiile parametrice

(11.8) i fie F : D V3 , F = (P, Q,R ) , continu, unde D R3 astfel nct () D. Atunci funcia F este integrabil pe i

P(x, y, z )dx + Q(x, y, z )dy + R(x, y, z ) dz =

= a [P(x (t ), y(t ), z(t ))x' (t ) + Q(x(t ), y (t ), z(t ))y' (t ) + R(x(t ), y(t ), z(t ))z' (t )]dt .
b

Demonstraie. Notm cu I integrala din membrul drept i fie D([a,b]),

= (a = t 0 < t 1 < L < t n = b ) , c k [t k 1, t k ] , k = 1,n ,


Mk (x (c k ), y(c k ), z(c k )) A k 1A k , unde A k (x k , y k , z k ) , x k = x(t k ) , y k = y (t k ), z k = z(t k ), k = 0,n .

Vom avea
F, Mk = [P(Mk )(xk xk 1 ) + Q(Mk )(y k yk 1 ) + R(Mk )(zk zk 1 ) ]
k =1

Cum este neted, funciile x, y, z sunt de clas C1 pe [a,b] i din teorema lui Lagrange () k , k , k (t k 1, t k ), k = 1, n astfel nct
x k x k 1 = x (t k ) x(t k 1 ) = x' (k )(t k t k 1 ) y k y k 1 = y (t k ) y (t k 1 ) = y' (k )(t k t k 1 ) zk zk 1 = z(t k ) z(t k 1 ) = z' (k )(t k t k 1 ) .

Obinem astfel
F, Mk = [P(x(c k ), y(c k ), z(c k )) x' (k ) +
k =1

+ Q(x(c k ), y(c k ), z(c k )) y' (k ) + R(x(c k ), y(c k ), z(c k )) z' (k ) ] (t k t k 1 )

Scriem F,Mk

astfel

- 236 F, Mk = [P(x(c k ), y(c k ), z(c k )) x' (c k ) + Q(x(c k ), y(c k ), z(c k )) y' (c k ) +


k =1

+ R(x(c k ), y(c k ), z(c k )) z' (c k )] (t k t k 1 ) + + [ P(x(c k ), y(c k ), z(c k )) (x' (k ) x' (c k )) +


k =1 n

+ Q(x(c k ), y(c k ), z(c k )) (y' (k ) y' (c k )) + + R(x(c k ), y(c k ), z(c k )) (z' (k ) z' (c k )) ] (t k t k 1 ).

i demonstraia se continu n mod asemntor cu demonstraia teoremei 11.1.2.


Exemple. 1. S se calculeze circulaia vectorului
v = (2x y )i + z j + (x + 3z )k de-a lungul arcului de elice

x = a cos t ( ) : y = a sin t , t [0, ], unde a R*. z = a t

Vom avea:
=

v dr = (2x y ) dx + z dy + (x + 3z ) dz =

[(2a cos t a sin t )( a sin t ) + at a cos t + (a cos t + 3a t ) a] dt =


0 0

= a2 2 sin t cos t + sin2 t + t cos t + cos t + 3t dt =

a2 3 2 + 4 . 2

2. S se calculeze integrala
I = 1 x 2 dx + x dy , unde (): x2 + y2 = 1, y 0, este parcurs n sens direct.

x = cos t Scriem curba sub forma parametric ( ) : y = sin t, t [0, ]

Vom avea
I=
0 0

[ 1 cos t( sin t) + cost(cost)]dt =


2

= sin2 t + cos2 t dt = cos 2t dt = 0


0

- 237 -

Integrale curbilinii independente de drum

Fie integrala

AB

P( x, y ) dx + Q( x, y ) dy ,

(11.9)

unde = AB este curb simpl, neted, P,Q: D R, continue, D R2, deschis astfel nct () D. Ne propunem s gsim condiiile n care integrala (11.9) nu depinde de drum, adic s nu depind de , ci numai de extremitile A i B ale curbei.
Teorema 11.2.2. Condiia necesar i suficient ca integrala (11.9) s nu

depind de drum n D este ca s existe o funcie F: D R, difereniabil pe D astfel nct dF(x, y ) = P(x, y )dx + Q(x, y ) dy, ()(x, y ) D. n acest caz expresia diferenial P(x, y ) dx + Q(x, y ) dy se numete diferenial total exact iar F primitiv a expresiei difereniale.
Demonstraie. Suficiena. Presupunem c exist F: D R, difereniabil pe D astfel

nct dF(x, y ) = P(x, y ) dx + Q(x, y ) dy, ()(x, y ) D.


x = x( t ) , de extremiti A(x1,y2), B(x2,y2), Fie ( ) : y = y( t ), t [a, b]

unde x1 = x(a), y1 = y(a), x2 = x(b), y2 = y(b). Vom avea

P(x, y ) dx + Q(x, y ) dy = [P(x(t ), y(t )) x' (t ) + Q(x(t ), y(t )) y' (t )]dt =


b a b

= a

d F(x (t ), y(t )) dt = F(x 2 , y 2 ) F(x 1, y 1 ) dt

Necesitatea. Presupunem c integrala (11.9) este independent de drum.

Fie A(x0,y0) D, fixat i M(x,y) D arbitrar.

- 238 -

Fig.2 Definim funcia F: D R astfel F(x, y ) = P( x, y ) dx + Q(x, y )dy , ()(x, y ) D .


AM

Fie h R cu h suficient de mic astfel nct M(x+h,y)D (acest lucru este posibil deoarece D este deschis). Folosind teorema de medie de la integrala Riemann, exist (0,1) astfel nct
F(x + h, y ) F(x, y ) = =

P(x, y ) dx + Q(x, y ) dy P(x, y ) dx + Q(x, y ) dy =


AM' AM x +h x

P(x, y ) dx + Q(x, y ) dy =
MM'

P(t, y ) dt = h P(x + h, y ) .

Astfel, pentru h0 vom avea F F(x + h, y ) F(x, y ) = P(x + h, y ) P(x, y ) , pentru h0, deci () (x, y ) = P(x, y ) x h Analog

() F (x, y ) = Q(x, y )
y

i atunci F este difereniabil n (x,y) i

dF(x,y) = P(x,y)dx+Q(x,y)dy.
Observaie. O integral independent de drum

P(x, y ) dx + Q(x, y )dy

se

mai noteaz i astfel extremitile curbei.

(x 2 ,y 2 )
x1,y1 )

P(x, y ) dx + Q(x, y ) dy , unde A(x1,y1), B(x2,y2) sunt

- 239 -

Consecine: 1. Dac integrala P(x, y ) dx + Q(x, y ) dy este independent de drum atunci

() F: D R, difereniabil pe D astfel nct dF(x, y ) = P(x, y ) dx + Q(x, y ) dy,

()(x, y ) D , adic

F F = P, = Q , n D. x y P 2F Q 2F = = , , y xy x yx

n plus, dac P, Q C1(D) atunci FC2(D) i cum folosind teorema lui Schwarz rezult
P Q = . y x

(11.10)

Dac D este un interval bidimensional (adic D=IJ, unde I,JR sunt intervale) sau mai general, D domeniu simplu conex atunci (11.10) reprezint o condiie necesar i suficient ca integrala (11.9) i s fie independent de drum.
2. Dac integrala (11.9) este independent de drum i este o curb

simpl, neted, nchis atunci

P(x, y ) dx + Q(x, y ) dy = 0 .

ntr-adevr, fie A,B() i m, n ca n figur:

Fig.3 Vom avea:

P(x, y ) dx + Q(x, y ) dy = P(x, y ) dx + Q(x, y ) dy + P(x, y ) dx + Q(x, y ) dy =

AmB

BnA

AmB

P(x, y ) dx + Q(x, y ) dy P(x, y ) dx + Q(x, y ) dy = 0


AnB

- 240 -

Observaie. Se arat c, dac integrala (11.9) este nul pe orice curb

nchis situat n D atunci ea este independent de drum.

Calculul integralelor curbilinii independente de drum


Presupunem c integrala I = P(x, y ) dx + Q(x, y ) dy este independent de

drum, unde ( ) = AB , A(x1,y1), B(x2,y2), P,Q: D R, continue, D R2, () D. Presupunem c [AC] , [CB] D , unde C are coordonatele C(x2,y1).

Fig. 4
x = t x = x 2 Vom avea AC : , t [x1, x 2 ] , CB : , t [y 1, y 2 ] i atunci y = y1 y=t
I= =

P(x, y ) dx + Q(x, y ) dy + P(x, y ) dx + Q(x, y ) dy =


AC CB y2 y1 1 x2 x1

P(t, y ) dt +

Q(x 2 , t ) dt,
x2
1

deci
y2
1

( x 2 ,y 2 ) ( x1 ,y1 )

P(x, y ) dx + Q(x, y ) dy = x P(t, y 1 ) dt + y Q(x 2 , t ) dt .

Rezult c o primitiv a expresiei difereniale P(x, y ) dx + Q(x, y ) dy este

F(x, y ) = x P(t, y 0 ) dt + y Q(x, t ) dt , unde M0 (x 0 , y 0 ) D este un punct fixat.


x y
0 0

- 241 -

Observaie. Rezultatele obinute pn acum n cazul bidimensional se

extind n cazul tridimensional astfel: Dac P,Q,R C1(D), unde D R3 este un domeniu simplu conex, () D, curb neted, simpl atunci o condiie necesar i suficient ca integrala

P(x, y, z ) dx + Q(x, y, z ) dy + R(x, y, z ) dz

s fie independent de drum este

ca
P Q Q R R P = = = , n D. , , y x z y x z

Exemplu. Fie o curb plan simpl, neted. S se arate c integrala

(2xy + y )dx + (x
2

+ 2xy )dy este independent de drum i s se determine o

primitiv a expresiei de sub integral. Avem P(x,y)= 2xy+y2, Q(x,y)= x2+2xy, P,QC1(R2) ,
P Q = 2x + 2y , deci integrala dat este independent de drum i atunci = y x

exist F: R2 R, difereniabil cu dF(x, y ) = (2xy + y 2 )dx + (x 2 + 2xy )dy . Funcia F se determin prin F(x, y ) = ( x
( x, y )
0 ,y 0

P(x, y ) dx + Q(x, y ) dy = x P(t, y 0 ) dt + y Q(x, t ) dt , )


x y
0 0

unde (x 0 , y 0 ) R 2 este un punct fixat. Lund (x0,y0)=(0,0) obinem:


F(x, y ) = 0 P(t,0 ) dt + 0 Q(x, t ) dt = 0 0 dt + 0 (x 2 + 2xt )dt = x 2 y + xy2
x y x y

Probleme propuse
1. S se calculeze: a) b)

x ds , unde ( ) : y = ln x,

x [1,2] ;

y ds , unde ( ) : x + y = 1;

- 242 -

c) d) e)

x = t xy ds , unde ( ) : ; y = 1 t 2 , t [ 1,1]

xyz ds , unde ( ) = [ AB], A (2,1,1),B(3,0,2) ;

2y 2 + z 2 ds , unde ( ) : x 2 + y 2 + z 2 = a 2 , x = y .

2. S se calculeze coordonatele centrului de greutate G al firului material

omogen

( ) :
3. S se calculeze a)

x = a(t sin t ) , t [0,2], a > 0 . x = a(1 cos t )

y dx x dy , unde ( ) : x 2 + y 2 = a 2 i este parcurs n sens direct ; 2 2 x +y

x = a cos b cos t b) y dx + z dy + x dz , unde ( ) : y = a cos b sin t, t [0,2] ; z = a sin b

c)

(x y )dx y dy ,

unde este frontiera mulimii din R2 mrginit de

curbele de ecuaii y = x 2 , y 2 = x i parcurs n sens invers acelor de ceasornic ;


d)

x dy + y dz + z dx ,

unde este curba obinut prin intersecia suprafeei

sferice de ecuaie x 2 + y 2 + z 2 = a 2 cu planele de coordonate, situat n primul octant i parcurs n sens direct.
2x (1 e y )

4. Fie P,Q: R2R, P(x, y ) =

(1 + x )

2 2

, Q(x, y ) =

ey . 1+ x2 pe R2 astfel nct

S se arate c

()F : R 2 R,

difereniabil

dF(x, y ) = P(x, y ) dx + Q(x, y ) dy, ()(x, y ) R 2 i s se determine F.

- 243 -

5. S se calculeze

( 2,2 ) (1,1)

2xy dx + 1 x 2 dy
2 2

( (1 x )

+y

i s se determine o primitiv a

expresiei de sub integral.


6. Fie f: D R, fC2(D), unde DR2 i fie v = grad f =
f f j. i+ x y

S se arate c, dac este o curb simpl neted astfel nct () D atunci

v dr = 0 .

7. Fie o curb simpl, nchis, neted sau neted pe poriuni ce limiteaz

un domeniu D. Atunci D are arie i Aria D =

1 x dy y dx . 2

- 244 -

CAPITOLUL 12

INTEGRALE MULTIPLE

12.1. Integrale duble


Fie f :D R o funcie mrginit, unde D R2 este un domeniu compact cu frontiera = FrD, o curb pe care o considerm simpl, nchis, neted sau neted pe poriuni. Dac presupunem f 0 atunci graficul su
f ={(x,y,z)R3 : (x,y)D, z = f(xy)} este o suprafa situat deasupra

planului x0y i proiecia ortogonal a ei pe planul x0y este domeniul D. Ne propunem s determinm volumul cilindrului care se sprijin pe D, are generatoarele paralele cu axa 0z i este limitat superior de suprafaa . Definiia 12.1.1. Fie A R2. Definim d(A) = sup d(P1,P2), numit i
P1 , P2 A

diametrul mulimii A, unde d(P1,P2) reprezint distana dintre punctele P1 i P2. Definia 12.1.2. Spunem c = ( D1, D2 , ..., Dn ) este o diviziune a domeniului D dac D1,D2,, Dn sunt domenii compacte fr puncte interioare comune astfel nct D = U D k i k = FrDk, k = 1, n , este o curb simpl, nchis, neted sau
k =1 n

neted pe poriuni. Vom nota cu D(D) mulimea tuturor diviziunilor lui D i pentru D(D), = (D1,D2,,Dn), vom nota cu
= max d(Dk ) , norma diviziunii .
k=1,n

- 245 -

Fie

Pk ( k , k ) Dk , k = 1, n

(f , Pk ) = f (Pk ) ariaDk,
k =1

numit

sum

integral dubl a funciei f, corespunztoare diviziunii i punctelor Pk. S observm c Dk are arie, cum k = FrDk este o curb simpl, nchis, neted sau neted pe poriuni. Fie mk = inf f , Mk = sup f , k = 1, n i
Dk Dk

s ( f ) = mk aria Dk , S ( f ) = Mk aria Dk , sumele Darboux inferioar, i


k =1 k =1

respectiv superioar a funciei f corespunztoare diviziunii . S observm c s (f) (f , Pk ) S (f ) , () D (D), = (D1, D2, ,Dn) i () Pk ( k , k ) Dk , k = 1, n . Dac f 0 suma (f, Pk) reprezint volumul corpului obinut prin reuniunea a n cilindri avnd ca baze pe D1,D2,,Dn, generatoarele paralele cu axa Oz i nlimile egale, respectiv, cu f(P1), f(P2), ,f(Pn). Volumul astfel obinut aproximeaz volumul cilindrului din introducere. Sumele Darboux adaus volumul cilindrului.
Definiia 12.1.3. Funcia f este integrabil pe D dac
s (f), S (f ) aproximeaz prin lips i respectiv prin

proprietatea > 0,

()

> 0 astfel ncat ()D(D),


(f , Pk ) I < .

( ) IR = ( 1, 2 ,..., n ) ,

cu cu

< i () Pk ( k , k )Dk , k = 1, n avem

Ca i n cazul integralei definite se arat c numrul real I din definiie, n ipoteza c exist, este unic i prin definiie I se numete integrala dubl a funciei f pe D i se noteaz I = S observm c
sau f dx dy . D lim (f , Pk ) .
0

f (x, y ) dx dy
D

f (x, y ) dx dy =
D

Observaie. Din definiie rezult c funcia f este integrabil pe D dac i


n numai dac ()nD (D), n = (D1 , Dn , ..., Dn 2 p
n

) cu

n 0 i

- 246 -

() Pkn (n, n ) Dn, k k k


limit.

k = 1, pn irurile

( (f, P ))
n n k

sunt convergente ctre o aceeai

Observaie. Dac f este integrabil pe D i f 0 atunci

f (x, y )dx dy

reprezint volumul cilindrului care se sprijin pe D, are generatoarele paralele cu axa Oz i este limitat superior de suprafaa de ecuaie z = f(x,y).

Criterii de integrabilitate
Teorema 12.1.1. (Criteriul lui Darboux). Fie f : D R 2 R, mrginit.

Atunci funcia f este integrabil pe D dac i numai dac () > 0, () > 0 astfel nct () D(D) cu < avem S(f) s(f) < .
Demonstraie. Necesitatea. Presupunem c f este integrabil pe D i fie
I=

f ( x, y ) dx dy
D

i > 0.

Conform definiiei () > 0 astfel nct () D(D), = (D1,D2,,Dn) cu


< i () Pk ( k , k ) D k , k = 1, n avem (f , Pk ) I <
I
, sau echivalent 3

< (f , Pk ) < I + 3 3

(12.1)

Trecnd n inegalitile (12.1) la infimum i apoi la supremum dup PkDk,


k 1, n i folosind urmtoarele relaii dintre sumele Riemann i Darboux:

s(f) = inf (f, Pk ) : Pk Dk , k = 1, n , S(f) = sup (f, Pk ) : Pk Dk , k = 1, n , obinem I

s ( f ) I + i respectiv I S ( f ) I + , de unde 3 3 3 3
S(f) s(f)
2 < , () D(D) cu < . 3

Suficiena. Fie I = sup {s ( f ) : D(D)}, I = {inf S (f ) : D(D)}. Evident

avem

s ( f ) I I S ( f ) , () D(D).

(12.2)

- 247 -

Fie > 0. Conform ipotezei () > 0 astfel nct () D(D) cu < avem S(f) s(f) < i folosind (12.2) rezult 0 I I S(f) s(f) < , adic
0 I I < .

Cum > 0 este arbitrar rezult I = I . Fie I = I = I . Avem s(f) I S(f) i s(f) (f , Pk ) S(f), () D(D), = (D1,D2,...,Dn) i Pk ( k , k ) D k , k = 1, n , de unde s(f) S(f) (f , Pk ) I S(f) s(f), sau (f , Pk ) I S(f) s(f). Dac < , rezult S(f) s(f) < i atunci (f , Pk ) I < , () Pk ( k , k ) D k , k = 1, n , adic funcia f este integrabil pe D i

f ( x, y ) dx dy = I.
D

Observaie. I se numete integrala Darboux inferioar iar I integrala

Darboux superioar. Din teorema 12.1.1 rezult c o functie mrginit f : D R2 R este integrabil pe D dac i numai dac integralele Darboux corespunztoare, I i I , sunt egale. Valoarea comun a celor dou integrale
I = I = I se numete integrala lui f pe D.

Teorema 12.1.2. Orice funcie f : D R, continu pe compactul D R2,

este integrabil pe D.
Demonstraie. Fie > 0. Cum f este continu pe compactul D, din

teorema lui Cantor este uniform continu i atunci () > 0 astfel nct ()(x,y), (x,y) D cu

(x' , y') (x' ' , y' ')

(x' x' ')2 + (y' y' ')2


. aria D

< , avem

f (x' , y') f (x' ' , y' ') <

Fie D(D), =(D1,D2,...,Dn) cu < .


' ' ' ' Cum f este continu i Dk compact, k = 1, n , () ( k , k ) Dk , ( k' , k' ) Dk ,

k = 1, n astfel nct
' ' ' ' , mk = inf f = f (k , k ), Mk = sup f = f (k' , k' ), k = 1 n . Dk
Dk

- 248 -

Deoarece
' ' ' ' f k , k f k' , k'

( , ) ( , )
' k ' k '' k '' k

, d ( Dk ) < , () k = 1 n

va rezulta c

) (
n k =1

<

n , i atunci S(f) s(f) = (Mk m k )aria Dk = aria D k =1

' ' ' ' = f k' , k' f k , k aria Dk <

((

) (

))

aria Dk = , deci k =1 aria D


n

S(f) s(f) < i conform teoremei 12.1.1 funcia f este integrabil pe D.

Proprieti ale integralei duble


Folosind definiia integrabilitii n cazul integralelor duble i raionnd analog ca la integrala definit se pot demonstra urmtoarele proprieti:
1. Dac f,g sunt integrabile pe D i ,R atunci f + g este integrabil

pe D i

(f + g)(x, y ) dx dy = f (x, y ) dx dy + g(x, y ) dx dy


D D D

(proprietatea de liniaritate).
2. Dac D = D1 D 2 unde D1, D2 sunt domenii compacte fr puncte

interioare comune i f este integrabil pe D1 i pe D2 atunci f este integrabil pe D i

f (x, y ) dx dy = f (x, y ) dx dy + f (x, y ) dx dy


D D1 D2

(proprietatea de aditivitate).
3. Dac f este integrabil pe D i f 0 atunci

f (x, y ) dx dy 0 .
D

4. Dac f i g sunt integrabile pe D i f g atunci

f (x, y )dx dy g(x, y )dx dy


D D

(proprietatea de monotonie).

5. Dac f este integrabil pe D atunci f este integrabil pe D i

f (x, y )dx dy

f (x, y )

dx dy.

- 249 -

6. Dac f este integrabil pe D, m = inf f , M = sup f atunci


D D

m aria D f (x, y ) dx dy M aria D.


D

7. Dac D este un domeniu compact din R 2 cu () = FrD, curb simpl,

neted, sau neted pe poriuni atunci aria D = dx dy .


D

8. Dac f este continu pe D atunci exist un punct (, ) D astfel nct

f (x, y )dx dy = f (, ) aria D


D

(formula de medie).

Calculul integralei duble

Considerm mai nti cazul n care D este un interval bidimensional, adic D = x J, unde = [a, b], J = [c, d] .
Teorema 12.1.3. Fie
f : [a, b] [c, d] R

mrginit i integrabil pe

[a, b] [c, d] astfel nct


i) ii)

() x [a, b] exist integrala F(x) = c f (x, y ) dy ;


d

F este integrabil pe [a,b].


b D a

Atunci

f (x, y ) dx dx = F(x ) dx = ( f (x, y ) dy )dx.


b d a c

Demonstraie. Fie D ([a,b]), = ( a = x 0 < x1 < ... < x n = b

) i

D ([c,d]), = ( c = y0 < y1 <...< ym = d ). Cu ajutorul diviziunilor i definim o diviziune a intervalului bidimensional D = [a,b] x [c,d] n intervale bidimensionale de forma: Dij = [xi-1,xi] x [yj-1,yj], i = 1, n, j = 1, m .

- 250 -

Fig.1 Fie m ij = inf f , Mij = sup f , i = 1, n, j = 1, m .


Dij Dij

Vom avea s (f ) = m ij aria D ij , S (f ) = Mij aria D ij .


i=1 j=1 i=1 j=1

Dac (x,y)Dij atunci mij f(x,y) Mij, de unde


mij (y j y j 1 )

yj y j 1

f (x, y ) dy Mij (y j y j 1 )

(12.3)

Sumnd n (12.3) dup j = 1, m obinem

mij (y j y j1 ) c f (x, y ) dy Mij (y j y j1 ) .


m d m j =1 j =1

Cum funcia F este integrabil pe [a,b], este integrabil pe orice compact [xi-1,xi], i = 1,n , i deci vom avea

mij (y j y j1 )(x i x i1 ) x
m j =1 n m

xi
i1

( f (x, y )dy )dx M (y y


d m c j =1 ij j
b d

j 1

)(x

x i 1 ) .

Sumnd dup i = 1,n obinem

mijaria Dij a
i=1 j=1

( f (x, y)dy )dx M aria D ,adic s (f ) ( f (x, y ) dy )dx S (f ) .


n m
c

i=1 j=1

ij

ij

Cum f este integrabil pe D rezult c


= = a
b d c D

( f (x, y)dy )dx, deci f (x, y)dx dy = ( f (x, y )dy )dx i


b d a c

demonstraia este ncheiat.

- 251 -

Vom da n continuare o formul de calcul al integralelor duble pentru domenii simple n raport cu una din axe. Un domeniu D R 2 se numete simplu n raport cu axa Oy dac este definit de inegalitile
axb D: 1(x ) y 2 (x ) ,

unde 1, 2 : [a, b] R sunt funcii continue. Cu alte cuvinte un domeniu D R 2 este simplu n raport cu axa Oy dac orice paralel la axa Oy dus printr-un punct interior din domeniul D intersecteaz frontiera domeniului n dou puncte. Analog, un domeniu compact D R 2

este simplu n raport cu axa Ox

dac este definit de inegalitile


cyd D: 1(y ) x 2 (y ) ,

unde 1, 2 : [c, d] R sunt funcii continue.


Teorema 12.1.4. Fie D R 2 un domeniu compact simplu n raport cu axa

Oy, adic
axb D: 1(x ) y 2 (x ) ,

(12.4)

unde 1, 2 : [a, b] R sunt continue. Fie f : D R , mrginit i integrabil pe D astfel nct


i) ii)

() x [a, b] exist integrala F(x) = (x ) f (x, y ) dy ;


1

2 (x )

F este integrabil pe [a,b]. Atunci


D

f (x, y ) dx dy = F(x ) dx =
b b a a

2 (x ) 1 ( x )

f (x, y ) dy dx.

Demonstraie. Fie c = inf 1, d = sup 2 i D0 = [a, b] [c, d]. Evident avem


[a,b ] [a,b ]

D D 0 (fig. 2).

- 252 -

Fie funcia f : D0 R , f (x, y ) =

f (x, y ) , daca (x, y ) D 0 , daca (x, y ) D0 \ D .

Fig.2 Funcia f este integrabil pe D0, deoarece f este integrabil pe D i


f = 0 pe D 0 \ D .

Folosind proprietatea de aditivitate a integralei duble va rezulta c

f (x, y ) dx dy = f (x, y ) dx dy = f (x, y ) dx dy


D0 D D

(12.5)

Din teorema 12.1.3 rezult c

D0

f (x, y ) dx dy = ( f (x, y ) dy )dx


b d a c

(12.6)

S observm c, pentru x[a,b], fixat avem

f (x, y ) dy =
d c

1 ( x )
c

f (x, y ) dy +

2 (x ) 1 ( x )

f (x, y ) dy +

2 (x )

f (x, y ) dy =
0

2 (x ) 1 ( x )

f (x, y ) dy =

2 (x ) 1 ( x )

f (x, y ) dy

(deoarece f (x, y ) = 0 pentru (x, y ) D

\D .

(12.7)

Conform ipotezei ultima integral din (12.6) exist pentru () x[a,b]. Cum F este integrabil pe [a,b], din (12.5), (12.6) i (12.7) rezult c
f (x, y ) dx dy =
b D a

2 (x ) 1 ( x )

f (x, y ) dy dx i demonstraia este ncheiat.

Observaii. 1) n particular, dac f este continu pe domeniul compact D

definit de (12.4) rezult c f este integrabil pe D i

f (x, y )dx dy = (
b D a

2 (x ) 1 ( x )

f (x, y ) dy dx .

- 253 -

2) Analog, dac D este simplu n raport cu axa 0x, adic


c yd D: 1(y ) x 2 (y ) ,

unde 1, 2 : [c, d] R sunt continue i f : DR este continu, atunci f este integrabil pe D i

f (x, y )dx dy = c
D

2 (y ) 1 (y )

f (x, y ) dx dy .

3) De obicei se scrie
b 2 (x )
1

a ( (x ) f (x, y ) dy )dx = a dx ( x ) f ( x, y )dy


b 2 ( x )
1

c ( (y ) f (x, y ) dy )dx = c dy ( y ) f ( x, y )dy .


d 2 (y )
1

2 (y)
1

Exemple. 1. S se calculeze integrala


I=

x (1 xy ) dx dy , unde D = [0,1] x [0,2].

Funcia f (x, y ) = x (1 xy) este continu pe D, deci integrabil i conform teoremei 12.1.3 vom avea:
1 3 2 2 x (1 xy ) dy dx = 1 x y x x y 2 dx = 1 2x 2 2x 2 dx = I = 0 0 0 0 20 1 3 2 1 5 2 1

= 2

x 3 2

2
0

x 5 2

=
0

4 4 8 = . 3 5 15

2. S se calculeze integrala
I=

(x 3y ) dx dy ,
D

unde D este domeniul plan limitat de curbele de ecuaii

y = x2, y2 = x. Domeniul D este simplu n raport cu ambele axe. Punctele de intersecie dintre cele dou parabole sunt O(0, 0) i A(1, 1). Scriem domeniul D astfel D :
0 x 1 x2 y x

i conform teoremei 12.1.4 vom avea:

- 254 x 1 y = 0 x (x 3 y ) dy dx = 0 xy 3 2 1
2

1 x x x4 3 2 dx = 0 x x 3 x + 3 dx 2 2 x

5 x 2 3 x2 x4 3 x5 = + 4 2 5 5 2 2 2

=
0

2 3 1 3 3 + = . 5 4 4 10 10

Dac l privim pe D ca domeniu simplu n raport cu axa 0x, adic


0 y 1 D: 2 , atunci y x y
= y 0 y 2 (x 3y ) dx dy =
1

x2 y 0 2 3yx y 2 dy =
1

y y4 3y y + 3 y 3 dx 0 2 2
1

5 1 1 y2 5 4 2 y 1y y 3 = 3 +3 = . 5 2 5 4 10 2 2 2 0

3. S se transforme integrala dubl I =

f (x, y ) dx dy
D

n integrale iterate,

y2 2 2 2 2 1, x 0 . unde D = (x, y ) R : x + y 9, x + 9

Domeniul D este mrginit de cercul cu centrul n origine, de raz 3 i elipsa cu centrul n origine de semiaxe 1 i 3, situat n semiplanul x 0 (fig. 3).

Fig.3

- 255 -

S observm c domeniul D nu este simplu n raport cu axa Oy. Pentru a aplica formula de calcul al integralelor duble pe domenii simple n raport cu axa Oy vom descompune domeniul D n domeniile D1 i D2 prin dreapta y=0. Vom avea = f (x, y ) dx dy + f (x, y ) dx dy = 1 + 2 .
D1 D2

Pentru calculul lui 1 inem seama c a = 0, b = 3,


1(x ) = 3 1 x 2 , pentru x [0,1]

i 1(x ) = 0 pentru x [1, 3], iar 2 (x ) = 9 x 2 , pentru x [0,3]. Vom avea deci
1 =
3 9x2 9x2 f (x, y ) dy dx + f (x, y ) dy dx. 0 3 1 x 2 1 0 1

Similar
2 =
3 0 3 1 x 2 0 9 x 2 f (x, y )dy dx + 1 1 9x2

f (x, y ) dx dy.

n concluzie
=

1 0

dx
1 0

9x2 3 1 x 2

f (x, y ) dy + dx
3 1 3
2

9x2 0 0

f (x, y ) dy + f (x, y ) dy .

+ dx

3 1 x 2 9x

f (x, y ) dy + dx
1

9x2

S observm c D poate fi privit ca domeniu simplu n raport cu axa Ox, deci poate fi scris astfel
3 y 3 D: 1 , i atunci 2 2 3 9y x 9y

= 3 dy 1
3

9y2 9 y2

f (x, y ) dx.

- 256 -

Teorema 12.1.5. (formula lui Green). Fie D R 2 un domeniu compact

mrginit de curba = Fr D , presupus simpl, nchis, neted sau neted pe poriuni. Fie P, Q : D R, P, Q C1 (D). Atunci
Q P P(x, y ) dx + Q(x, y )dy = x y dx dy,

unde sensul de parcurs pe curba este cel direct.


Demonstraie. Vom da o demonstraie pentru cazul n care domeniul D este

simplu n raport cu ambele axe. Pentru cazul general se pot consulta lucrrile [8] sau [14]. Fie D :
axb 1( x ) y 2 ( x )

, unde 1, 2 : [a, b] R sunt continue, deci D este

simplu n raport cu axa Oy.

Fig.4 Fie A(a, 1(a)), B(b, 1(b)), C(b, 2(b)), D(a, 2(a)). Vom avea ( ) = AB [BC] CD [DA ] unde: AB : y = 1 ( x ), x [a, b] ,
x=b [BC] : y = t, t [1(b), 2 (b)] ,

CD : y = 2 ( x ), x [a, b] ,
x=a [DA ] : y = t, t [1(a), 2 (a)] .

- 257 -

innd seama de modul de calcul al integralei curbilinii de spea a doua i de sensul de parcurs pe fiecare arc de curb obinem:

P( x, y)dx = P( x, y)dx + P( x, y)dx + P( x, y)dx + P( x, y )dx =

AB

[ BC ]

CD

[ DA ]

= =

b a b a

P( x, 1( x ))dx P( x, 2 ( x ))dx =
a

(12.8)

[P( x, 1( x )) P( x, 2 ( x ))]dx

(am inut cont c integralele pe segmentele [BC] i [DA] sunt egale cu 0 deoarece x este constant pe aceste segmente). innd cont de formula de calcul al unei integrale duble pe un domeniu simplu obinem:

D
2 ( x ) P b b (x) P dxdy = dx dy = P( x, y ) 2 dx = a a 1 ( x ) y 1( x ) y

(12.9)

= [P( x, 2 ( x )) P( x, 1( x ))]dx
a

Din (12.8) i (12.9) rezult c

P( x, y )dx = y dxdy .

(12.10) (12.11)

Analog obinem

Q( x, y )dy = x dxdy .

Din (12.10) i (12.11), prin adunare, se obine formula lui Green.


Aplicaie. Lund P( x, y ) =
y x 1 1 y x , Q( x, y ) = obinem: 2 2

2 dx + 2 dy = ( 2 + 2 )dxdy = dxdy = aria D ,

deci aria unui domeniu

compact D limitat de o curb simpl, nchis, neted sau neted pe poriuni este aria D =
1 xdy ydx , unde sensul pe curb este cel direct. 2

- 258 -

Schimbarea de variabile n integrala dubl


Considerm integrala dubl I = f ( x, y )dxdy , unde f : D R este o funcie
D

continu, D R2 un domeniu compact mrginit de o curb presupus simpl, nchis, neted sau neted pe poriuni. Considerm transformarea regulat
x = (u, v ) (T ) : , (u, v ) D' R 2 , adic y = (u, v )

(12.12)

, C1 (D),

D(, ) (u, v) 0, () (u, v) D' . D(u, v)

Prin aceast transformare domeniul D mrginit de o curb trece n domeniul D mrginit de curba . Se arat c o transformare regulat este biunivoc, adic fiecrui punct (x,y)D i corespunde un unic punct (u,v)D i reciproc; mai mult, prin aceast transformare D este un domeniu compact, iar frontiera sa = FrD este o curb simpl, nchis, neted sau neted pe poriuni (vezi [7]). Se poate deomonstra c, dac
D( , ) > 0, ()(u, v ) D' , D(u, v )

atunci

transformarea (T) este direct, adic, dac un punct M(u,v)() se deplaseaz n sens direct atunci i punctul corespunztor M(x,y)(), unde x = (u,v), y = (u,v) se deplaseaz tot n sens direct. Ne propunem i calculm aria domeniului D prin transformarea (T) n ipoteza c aceast transformare este regulat i direct, iar funciile i admit derivate pariale mixte de ordinul doi continue pe D. Lund n formula lui Green P(x,y) = 0, Q(x,y) = x, obinem unde sensul pe curba este cel direct. Obinem astfel c Aria D = xdy .

xdy = dxdy ,

Efectund schimbarea de variabile dat de (12.12) rezult:

- 259 -

Aria D = xdy = (u, v ) du + dv = P(u, v )du + Q(u, v )dv , v ' u '

unde P(u, v ) = (u, v )

i Q(u, v ) = (u, v ) . v u Q P

Aplicnd formula lui Green pentru ultima integral obinem:

P(u, v )du + Q(u, v )dv = u v dudv .


'

D'

innd cont de teorema lui Schwarz vom avea


Q P 2 2 = + = u v u v vu v u uv D(, ) = = u v = . u v v u D(u, v ) u v

n concluzie, Aria D =

D(, ) dudv . D ' D(u, v )

Observaie. Dac

D( , ) < 0, ()(u, v ) D' atunci este parcurs n sens D(u, v ) D(, ) dudv . D(u, v )

invers i analog obinem


Aria D =
D'

n general, dac (T) este o transformare regulat se obine


Aria D =

D'

D(, ) dudv . D(u, v )

(12.13)

Teorema 12.1.6. Fie n planul uOv un domeniu compact D cu frontiera

o curb simpl, nchis, neted sau neted pe poriuni i


x = (u, v ) (T ) : , (u, v ) D' , o transformare regulat de la planul uOv la y = (u, v )

planul xOy astfel nct funciile i admit derivate pariale mixte de ordinul doi continue pe D.

- 260 -

Dac D = T(D), = T() i f : D R este o funcie continu atunci

f ( x, y)dxdy = f ((u, v ), (u, v ))


D D'

D(, ) dudv . D(u, v )

(12.14)

Demonstraie. Fie = (D1, D2, , Dn) o diviziune a compactului D. Prin

transformarea regulat (T) fiecrui domeniu Dk, k = 1, n i corespunde un domeniu Dk D. Obinem astfel o diviziune = (D1, D2, , Dn) a domeniului D. Din (12.13) vom avea Aria Dk =
Dk

D(, ) dudv, k = 1, n i din teorema de D(u, v )

medie rezult c pentru fiecare k = 1, n exist un punct (uk, vk)Dk astfel nct
Aria Dk = D(, ) (u k , v k ) aria D'k . D(u, v )

Fie k = (uk,vk), k = (uk,vk), k = 1, n i evident Mk(k,k)Dk. Vom avea:


( f , Mk ) = f (k , k ) aria Dk =
k =1 n

= f ((uk , v k ), (uk , v k ))
k =1 n

D(, ) (uk , v k )aria D'k = D(u, v )

= F(uk , v k ) aria D' k = ' (F,M' k ),


k =1

(12.15)

unde F(u, v ) = f ((u, v ), (u, v ))

D(, ) , M'k (uk , v k ), k = 1, n . D(u, v )

Rezult c orice sum Riemann relativ la funcia F, diviziunea i punctele intermediare Mk este egal cu o sum Riemann relativ la funcia f, diviziunea i punctele intermediare Mk. Din existena celor dou integrale i prin trecere la limit n relaia (12.15) obinem (12.14).
Observaie.
x = cos (T ) : , y = sin
D

n cazul schimbrii n coordonate polare, adic

[0, R], [0,2] , rezult

D( x, y ) = i atunci D(, )

f ( x, y )dxdy = f ( cos , sin )dd .


D'

- 261 -

Exemplu. S se calculeze integrala I =


D

dx dy , unde ( 4 + x 2 + y 2 )3

D:x2 + y2 1, y 0. Efectund schimbarea de variabile x = cos, y = sin, [0,], [0,1] obinem


d 1 = ( 4 + 2 )2 I = 0 d 0 2 3 (4 + ) 4
1 1

1 1 9 = . = 4 16 25 1600

Aplicaii ale integralei duble 1. Calculul volumelor i ariilor


Dac f : D R2 R este o funcie continu i pozitiv atunci integrala

f ( x, y )dxdy
D

reprezint volumul cilindrului care se sprijin pe D, are

generatoarele paralele cu axa Oz i este limitat superior de suprafaa de ecuaie z = f(x,y). Dac f(x,y) = 1, () (x,y)D atunci integrala cilindrului, adic Aria D =

dxdy
D

reprezint aria bazei

dxdy
D

2. Coordonatele centrului de greutate al unei plci plane


Fie D R2 un domeniu compact care este imaginea unei plci plane la care densitatea = (x,y) > 0 este o funcie continu de coordonatele punctului din domeniu. Din considerente de mecanic se poate arta c masa M i coordonatele centrului de greutate G(xG, yG) sunt date de
M= 1 1 x( x, y )dxdy, y G = M y( x, y )dxdy . M D D

( x, y )dxdy,
D

xG =

Dac placa este omogen ( = const > 0) atunci

- 262 -

xG =

xdxdy
D

aria D

, yG =

ydxdy
D

aria D

3. Momentele de inerie ale unei plci plane


Momentele de inerie ale plcii plane D fa de axe i respectiv pol sunt date de
I Ox = y 2 ( x, y )dxdy , IOy = x 2( x, y )dxdy i respectiv
D D

IO = ( x 2 + y 2 )( x, y )dxdy .
D

12.2. Integrale de suprafa


Fie D R2 un domeniu compact.
Definiia 12.2.1.

Se

numete

pnz

parametrizat

sau

suprafa

parametrizat n spaiul R3 orice funcie continu : D R3, (u,v) = (x(u,v), y(u,v), z(u,v)). Vom nota cu () = Im = {( x(u,v), y(u,v), z(u,v))R3: (u,v)D}, imaginea suprafeei . Ecuaiile x = x(u,v), y = y(u,v), z = z(u,v) reprezint ecuaiile parametrice ale suprafeei i scriem: ()
x = x(u, v ) : y = y(u, v ), (u, v ) D z = z(u, v )

(12.16)

Dac B = { i, j, k } este o baz ortonormat n reperul cartezian Oxyz atunci suprafaa poate fi dat i astfel: () : r = r (u, v ), (u, v ) D (12.17)

unde r (u, v ) = x(u, v )i + y(u, v ) j + z(u, v )k , numit i ecuaia vectorial parametric a suprafeei

- 263 -

n ipotezele n care pot fi eliminai parametrii u i v din ecuaiile (12.16) obinem () : F(x,y,z) = 0, (x,y,z)V R3, forma implicit. (12.18)

Dac pe V sunt ndeplinite ipotezele teoremei funciilor implicite, suprafaa se scrie echivalent () : z = z(x,y), (x,y)D0 R2, forma explicit. (12.19)

Definiia 12.2.2. Suprafaa se numete simpl dac funcia (u, v ) r (u, v )

este injectiv. Suprafaa se numete neted (sau regulat) dac x,y,z C1(D) i A2 + B2 + C2 0 pe D, unde
A= D( y, z ) , D(u, v ) B= D( x, y ) D( z, x ) , C= . D(u, v ) D(u, v )

(12.20)

Suprafaa se numete neted pe poriuni dac exist o diviziune = (D1,D2,,Dn) a lui D astfel nct suprafeele
x = x(u, v ) (k ) : y = y(u, v ) , (u, v ) Dk , k = 1 n s fie netede. , z = z(u, v )

Considerm suprafaa

simpl, neted, dat prin ecuaiile parametrice

(12.16) i fie P0(u0,v0)() un punct fixat (fig. 5).

Fig.5

- 264 -

Pentru u = u0 i respectiv v = v0 obinem pe suprafaa curbele


( u ) : r (u, v 0 ) = x(u, v 0 )i + y(u, v 0 ) j + z(u, v 0 )k , ( v ) : r (u0 , v ) = x(u0 , v )i + y(u0 , v ) j + z(u0 , v )k , numite i curbe de coordonate.

Vectorii directori ai tangentelor n P0 la curbele (u) i (v) vor fi


r 'u (u0 , v 0 ) = x'u (u0 , v 0 )i + y'u (u0 , v 0 ) j + z'u (u0 , v 0 )k si
rv ' (u0 , v 0 ) = x'v (u0 , v 0 )i + y'v (u0 , v 0 ) j + z'v (u0 , v 0 )k .

S observm c r 'u r 'v = A i + Bj + C k , deci condiia de regularitate (12.20) exprim faptul c r 'u r 'v , adic unghiul dintre curbele de coordonate u i v este diferit de zero. De asemenea, aceast condiie asigur existena vectorului
N normal la suprafaa n P0, N = r 'u r 'v = A i + Bj + C k .

Versorul vectorului normal N va fi


n= N N = r 'u r 'v A i + Bj + C k . = r 'u r 'v A 2 + B 2 + C2

Dac suprafaa este dat prin forma implicit (12.18) se arat c


N = F' x i + F' y j + F'z k , iar dac este dat prin forma explicit (12.19) atunci N = p i qj + k , unde p = zx, q = zy.

Definiia 12.2.3. Fie o suprafa simpl, neted, dat prin ecuaiile

parametrice (12.16). Se numete element de arie al suprafeei diferenial notat d i definit astfel: d = r 'u r 'v dudv .

forma

Observaie. Din identitatea lui Lagrange


r 'u r 'v
2

= r 'u

r 'v

(r 'u r 'v )2

folosind

notaiile

2 2 2 2 E = r 'u = x'u + y'u + z'u ,

G = r ' 2 = x' 2 + y' 2 + z' 2 , F = r ' u r ' v = x' u x' v + y' u y' v + z' u z' v obinem v v v v
d = EG F2 dudv .

Dac suprafaa este dat sub forma explicit (12.19) atunci


d = 1 + p2 + q2 dxdy .

- 265 -

Definiia 12.2.4. Suprafaa dat prin ecuaiile parametrice (12.16) are arie

dac integrala dubl

EG F 2 dudv exist i este finit. n acest caz, valoarea

integralei duble reprezint aria suprafeei .

Teorema 12.2.1. Dac suprafaa

dat prin ecuaiile parametrice (12.16)


D

este simpl, neted atunci

are arie i Aria = EG F 2 dudv este

independent de reprezentarea parametric (12.16). Pentru demonstraie se pot consulta lucrrile [8], [14] sau [19].

Integrale de suprafa de spea nti (sau n raport cu aria)


Fie

o suprafa simpl, neted, dat prin ecuaiile parametrice (12.16),

unde D R2 este un domeniu compact limitat de o curb simpl, nchis, neted sau neted pe poriuni (deci D are arie). Fie f : V R, mrginit, unde V R3 astfel nct () V. Fie = (D1, D2, , Dn) o diviziune a compactului D i = (1,

2, , n)

x = x(u, v ) diviziunea corespunztoare a suprafeei , ( k ) : y = y(u, v ) , (u, v ) Dk , k = 1, n . z = z(u, v )

Fie Pk(xk, yk, zk)(k), k = 1, n unde xk = x(uk, vk), yk = y(uk, vk), zk = z(uk, vk), (uk, vk)Dk, k = 1, n . Considerm suma ( f ,Pk ) = f (Pk ) aria k .
k =1 n

Definiia 12.2.5. Funcia f este integrabil pe suprafaa dac () IR astfel

nct () > 0, () > 0 cu proprietatea c () D(D), = (D1, D2, , Dn) cu < i () Pk (k), k = 1, n avem ( f, Pk ) I < .

- 266 -

Observaie. Se arat c numrul real I este unic i prin definiie I se

numete integrala de suprafa a funciei F pe i se noteaz I = f ( x, y, z )d .

S observm c

f ( x, y, z)d =

lim ( f , Pk ) .
0

Teorema 12.2.2. Dac suprafaa este simpl, neted, iar funcia f este

continu, atunci f este integrabil pe i

f ( x, y, z)d = f ( x(u, v ), y(u, v ), z(u, v ))

EG F2 dudv .

Demonstraie. Fie = (D1, D2, , Dn) o diviziune a lui D,


x = x(u, v ) Pk (k ) : y = y(u, v ) , (u, v ) Dk , k = 1, n, Pk ( x k , y k , zk ) unde z = z(u, v )

xk = x(uk, vk), yk=y(uk,vk), zk = z(uk, vk), (uk, vk)Dk, k = 1, n . Din teorema 12.2.1 avem Aria = EG F2 dudv i din teorema de medie
D

() (k, k)Dk astfel nct Vom avea:

Dk

EG F2 dudv =

(EG F2 )(k , k ) aria Dk ,

k = 1, n .

( f , Pk ) = f (Pk ) aria k =
k =1

= f ( x(uk , v k ), y(uk , v k ), z(uk , v k )) (EG F2 )(k , k ) aria Dk =


k =1
n

= f ( x(k , k ), y(k , k ), z(k , k )) (EG F2 )(k , k ) aria Dk +


k =1 n

+ [f ( x(uk , v k ), y(uk , v k ), z(uk , v k )) f ( x(k , k ), y(k , k ), z(k , k ))]


k =1

(EG F2 )(k , k ) aria Dk .

Funcia g(u, v ) = f ( x(u, v ), y(u, v ), z(u, v )) EG F2 limita primei sume pentru 0 este
D

este continu i atunci


EG F 2 dudv .

f ( x(u, v ), y(u, v ), z(u, v ))

Folosind uniform continuitatea lui f rezult c a doua sum are limita zero pentru 0 i demonstraia este ncheiat.

- 267 -

Observaie. n cazul unei reprezentri explicite () : z = z(x,y) va rezulta c

f ( x, y, z)d = f ( x, y, z( x, y ))

1 + p2 + q2 dxdy .

Exemplu. S se calculeze integrala I = zd ,

unde () : z = x2 + y2, (x,y)D, iar D = {(x,y)R2 : x2 + y2 a2}. Cum p =


z z = 2x, q = = 2y, rezult I = x y

( x
D

+ y 2 ) 1 + 4 x 2 + 4 y 2 dxdy .

Efectund schimbarea de variabil n coordonate polare


x = cos , y = sin , [0, a], [0,2] obinem
I=
2 0

a 0

1 + 42 3d = 2 3 1 + 42 d ,
0

care

se

calculeaz

folosind

metoda integrrii prin pri.

Integrale de suprafa de spea a doua (sau n raport cu coordonatele)


Fie o suprafa simpl, neted, definit de ecuaiile parametrice (12.16).
Definiia 12.2.6. Suprafaa

se numete bilater (sau cu dou fee) pe D putem considera doi versori ai

dac versorul normalei n este o funcie continu n fiecare punct M(). Deoarece ntr-un punct M de pe suprafaa normalei la suprafa i anume n1 = n =

r 'u r 'v r' r' , n2 = n = u v , r 'u r 'v r 'u r 'v

o suprafa bilater mpreun cu o alegere, cu o fixare a unuia din cei doi versori ai normalei se numete suprafa orientat.
Observaie. Nu orice suprafa are dou fee. Exist suprafee cu o

singur fa, suprafee pe care, printr-o deplasare continu normala i schimb direcia n mod continuu i revine n punctul iniial cu sensul opus sensului iniial. Cel mai simplu exemplu n acest sens este banda lui Mobius.

- 268 -

Pentru a o obine lum o foaie de hrtie dreptunghiular ABCD, o rsucim i o lipim astfel nct A s coincid cu C i B cu D (vezi fig. 6).

Fig.6

Presupunem c suprafaa coordonatele versorului n =

este orientat i fie cos, cos, cos


al normalei la suprafa n punctul M(),

r 'u r 'v r 'u r 'v

numite i cosinui directori ai normalei la suprafa. S observm c , , reprezint unghiurile formate de versorul n cu vectorii i, j i respectiv k . Vom nota cu

+ faa suprafeei definit de versorul normalei - faa suprafeei definit de

n(cos , cos , cos ) i cu

n( cos , cos , cos ) .

Fie v (P, Q, R) un cmp vectorial continuu pe , adic funciile P, Q, R : R sunt continue pe (). Prin definiie, integrala de suprafa de spea a doua a cmpului vectorial v pe suprafaa +, notat cu

v n d

se nelege numrul real definit prin

v n d = [P( x, y, z) cos + Q( x, y, z) cos + R( x, y, z) cos ]d ,


unde n = cos i + cos j + cos k . Notnd cos d = dydz, cos d = dzdx, cos d = dxdy , vom avea

- 269 -

v n d = P( x, y, z)dydz + Q( x, y, z)dzdx + R( x, y, z)dxdy ,


+

notaie frecvent,

unde + este faa suprafeei definit de versorul normalei n(cos , cos , cos ) .

Observaii.
1.

P( x, y, z)dydz + Q( x, y, z)dzdx + R( x, y, z)dxdy = v n d ;


2. Valoarea integralei

v n d

reprezint fluxul cmpului de vectori v prin

suprafaa .
Exemplu . S se calculeze integrala
I = ydydz + zdzdx + 3 xdxdy , unde + este faa exterioar a sferei de
+

ecuaie x2+y2+z2 = a2, a > 0, situat n primul octant. Vectorul director al normalei la suprafa ntr-un punct arbitrar M(x,y,z)() are coordonatele Fx, Fy, Fz (unde F(x,y,z) = x2+y2+z2 - a2), adic 2x, 2y, 2z, iar versorul normalei asociat feei exterioare este:
n= N N = x yr z i+ j+ k. a a a

Rezult c I =

1 ( xy + yz + 3 xz)d . a

O reprezentare parametric a prii de sfer este


x = a cos sin ( ) : y = a sin sin , 0 , 2 z = a cos 0 . 2

Vom avea:
2 2 E = x'2 + y' + z' = a 2 sin2 2 2 G = x ' + y ' 2 + z' = a 2

, de unde EG F 2 = a 4 sin 2 i

F = x ' x ' + y ' y ' + z' z' = 0

- 270 -

I=

1 2 2 2 2 (a sin cos sin + a sin sin cos + 3a cos sin cos ) a D

EG F2 dd = = a3 (sin cos sin3 + sin sin2 cos + 3 cos sin2 cos )dd =
D

1 = a3 2 sin3 + sin2 cos + 3 sin2 cos d = 2a3 . 0 2

Formula lui Stokes


Fie

o suprafa simpl, neted, orientat, definit de ecuaiile

parametrice (12.16), care se sprijin pe conturul nchis, simplu i neted (fig.7).

Fig.7 Dac P, Q, R : V R sunt de clas C1 pe V, unde V R3 astfel nct ()V atunci vom avea:

P( x, y, z)dx + Q( x, y, z)dy + R( x, y, z)dz =

y z dydz + z x dzdx + x y dxdy,


+

unde

+ este faa suprafeei definit de versorul normalei n(cos , cos , cos ) ,

iar sensul de parcurs pe curba este cel asociat feei corespunztoare (curba este parcurs astfel nct un observator ce se deplaseaz pe s lase tot timpul la stnga faa +).

- 271 -

Pentru demonstraie se pot consulta lucrrile [14] sau [19].

Observaie. Fie v cmpul vectorial definit de funciile P, Q, R, adic


v ( x, y, z ) = P( x, y, z )i + Q( x, y, z ) j + R( x, y, z )k , () (x,y,z)V.

Atunci formula lui Stokes se poate scrie vectorial astfel:

vdr = rotv n d

i din punct de vedere fizic aceast formul exprim

faptul c circulaia vectorului v pe bordul al unei suprafee este egal cu fluxul rotorului lui v prin aceast suprafa.

12.3. Integrale triple


Acestea se definesc n mod analog cu integralele duble. Fie V R3 un domeniu compact cu frontiera o suprafa simpl, nchis, neted sau neted pe poriuni. Se arat c un astfel de domeniu are volum (vezi [14]). Spunem c = (V1, V2, , Vn) este o diviziune a domeniului compact V dac V1, V2, , Vn sunt domenii compacte fr puncte interioare comune astfel nct V = U Vk i
k =1 n

k = Fr Vk, k = 1, n este o suprafa simpl, nchis, neted sau

neted pe poriuni. Vom nota cu D(V) mulimea tuturor diviziunilor lui V i pentru D(V), =(V1, V2, , Vn) vom nota cu = max d( Vk ) , norma diviziunii , unde d(Vk)
k =1,n

este diametrul domeniului Vk, d( Vk ) = sup dist(P1, P2 ) .


P1,P2 Vk

S observm c Vk are volum, cum Fr Vk este o suprafa simpl, nchis, neted sau neted pe poriuni. Fie f : V R, mrginit, Pk(k, k, k)Vk, k = 1, n i ( f , Pk ) = f (Pk ) volVk ,
k =1 n

suma integral tripl a funciei f corespunztoare diviziunii i punctelor PkVk.

- 272 -

Fie mk = inf f , Mk = sup f , k = 1, n i s ( f ) = mk volVk , S ( f ) = Mk volVk


Vk Vk k =1 k =1

sumele Darboux inferioar i respectiv superioar a funciei f corespunztoare diviziunii . S observm c s ( f ) ( f , Pk ) S ( f ) , () D(V), = (V1, V2, , Vn) i () PkVk, k = 1, n .
Definiia 12.3.1. Funcia f este integrabil pe V dac () IR astfel nct

() > 0, () > 0 cu proprietatea c () D(V), = (V1, V2, , Vn), cu


< i () PkVk, k = 1, n avem ( f, Pk ) I < .

Se arat c numrul real I este unic i prin definiie I se numete integrala tripl a funciei f pe domeniul V i se noteaz
I = f ( x, y, z )dxdydz (sau
V

fdV ).
V

S observm c

fdV =
V

lim ( f , Pk ) .
0

Ca i n cazul integralelor duble se arat c o funcie mrginit f : V R este integrabil dac i numai dac () > 0, () > 0 astfel nct () D(V), = (V1, V2, , Vn), cu
< i () PkVk

avem

De ( f ) < . S ( f ) sasemenea, dac f este continu pe V atunci f este integrabil pe V, iar dac V este un domeniu simplu n raport cu axa Oz, adic
(x, y ) D , unde D R2 este un domeniu compact care este V: 1( x, y ) z 2 ( x, y )

proiecia domeniului V n planul xOy i 1, 2 : D R sunt continue, atunci

f ( x, y, z)dxdydz = (
V D

2 ( x, y ) 1 ( x , y )

f ( x, y, z )dz dxdy .

Proprieti asemntoare cu cele de la integrale duble se obin i pentru integralele triple.


Exemplu. S se calculeze integrala
I=

zdxdydz , unde V este definit de inegalitile


V

V : x + y + z 1, x 0, y 0, z 0.

- 273 -

Domeniul V este simplu n raport cu axa Oz,


( x, y ) D V: , unde D = {(x,y)R2 : x + y 1, x 0, y 0}. 0 z 1 x y

Vom avea
I = dxdy
D 1 x y 0

zdz =
D

z2 2

1 x y

dx dy =

1 1 2 2 2 (1 x y ) dxdy = 2 ( x + y + 1 2x 2y + 2xy)dxdy . 2 D D

Cum D este un domeniu simplu n raport cu axa Oy, adic D : rezult


I=
1 x 1 1 dx ( x 2 + y 2 + 1 2x 2y + 2xy ) = 0 2 0 y2 y2 1 x 1 1 1 = ( x 2 y + y 3 + y 2xy 2 + 2x ) = 0 2 2 2 0 3

0 x 1 0 y 1 x

1 1 2 1 3 2 2 0 x (1 x ) + 3 (1 x ) + 1 x 2x(1 x ) (1 x ) + x(1 x ) dx = 2

x3 1 1 2 1 3 2 = (x x + x + x + 1 x 2x + 2x 2 1 + 2x x 2 + x 2x 2 + x 3 ) dx = 0 3 3 2 1 1 1 1 1 = x 3 + x 2 x + dx = . 2 0 3 3 24

Schimbarea de variabil la integrala tripl


Considerm integrala tripl I = f ( x, y, z )dxdydz , unde f : V R este o
V

funcie continu, V R3 este un domeniu compact cu frontiera o suprafa simpl, nchis, neted sau neted pe poriuni. Considerm transformarea regulat
x = x(u, v, w ) D( x, y, z ) T : y = y(u, v, w ) , (u, v, w ) V ' , adic x,y,zC1(V), 0 pe V, unde D(u, v, w ) z = z(u, v, z )

V R3 este un domeniu compact. n aceste condiii avem:

- 274 -

f ( x, y, z)dxdydz = f ( x(u, v, w ), y(u, v, w ), z(u, v, z)) D(u, v, w ) dudvdw


V V'

D( x, y, z)

numit

formula schimbrii de variabile la integrala tripl. Expresia diferenial notat dv i definit astfel dv = numete element de volum. Pentru demonstraie se pot consulta lucrrile [14] sau [19]. n cazul coordonatelor sferice transformarea T este
x = cos sin (T ) : y = sin sin , z = cos

D( x, y, z) dudvdz se D(u, v, w )

[0, R], [0,2], [0, ] ,

iar

jacobianul

transformrii este J =

D( x, y, z ) = 2 sin , deci D(, , )


2

f ( x, y, z)dx dy dz = f ( cos sin , sin sin , cos )


V V'

sin d d d .

n cazul coordonatelor cilindrice transformarea T este


x = r cos (T ) : y = r sin , [0,R], [0,2], z [0, h] , iar jacobianul transformrii z=z

este J =

D( x, y, z ) = r , deci D(r, , z )

f ( x, y, z)dxdydz = f (r cos , r sin , z) rdrddz .


V V'

Formula lui Gauss-Ostrogradski


Fie V R3 un domeniu compact cu frontiera o suprafa , simpl, nchis, bilateral, neted sau neted pe poriuni i un cmp de vectori
v ( x, y, z ) = P( x, y, z )i + Q( x, y, z ) j + R( x, y, z )k , de clas C1 pe domeniul V.

n aceste condiii avem:

Pdydz + Qdzdx + Rdxdy = x + y


+
V

R dxdydz , unde + este faa z

suprafeei definit de versorul normalei n(cos , cos , cos ) , numit i formula integral a lui Gauss-Ostrogradski.

- 275 -

Observaie. Formula lui Gauss-Ostrogradski se poate scrie vectorial astfel:

v n d = divvdxdydz .

Exemplu. S se calculeze integrala


I = xdydz + ydzdx + zdxdy , unde
+

+ este faa exterioar a suprafeei ce

limiteaz domeniul V definit de inegalitile : x2 + y2 z 4 - x2 - y2. Vom aplica formula lui Gauss cu P = x, Q = y, R = z i integrala devine I = 3dxdydz = 3 dxdydz = 3 dxdy x
V V D 4x2 y2
2

+y2

dz = 3 2(2 x 2 y 2 )dxdy
D

unde D = {(x,y)R2 : x2+y2 2}. Trecnd la coordonatele polare: x = cos, y = sin, unde [0, 2 ], [0,2] , obinem I = 6 0 d 0 (2 2 )d = 12 .
2 2

Probleme propuse
1. S se calculeze: a) b) c) d) e)

1 + sin x sin y dxdy,


D

cos y

D:0 x

, 0y ; 2 2

(1 + y )
D D

dxdy, D : y x, xy 1, 1 x 2 ; D : x 2 + y 2 2y, y x 2 , x 0 ;

(1 y )dxdy,

x 2 y 2 dxdy, D = OAB, A(1,1), B(1,1) ;

arcsin
D

x + y dxdy, D limitat de x + y = 0, x + y = 1, y = 1, y = -1.

2. Folosind coordonatele polare s se calculeze: a) b)

x 2 + y 2 dxdy, D : x 2 + y 2 a 2 , unde a > 0;

ydxdy,
D

D : x 2 + y 2 a2 , y x , unde a > 0;

- 276 -

c)

( x
D

+ y 2 )dxdy, D : x 2 + y 2 2x .

3. Folosind schimbri de variabile convenabile s se calculeze: a) b) c)

( x + y )dxdy, D : 1 x + y 13,
D

x y 5x ;

xdxdy, D :
D

x2 y2 + 1, y 0 ; a2 b2

ydxdy,
D

D : 1 xy 2, x y 2x .

4. S se calculeze coordonatele centrului de greutate ale plcii omogene

D R2, unde D este limitat de curbele de ecuaii x+ y = 3, xy = 2.


5. Direct i cu formula lui Green s se calculeze

( x y )dx + xdy , unde

este frontiera domeniului D ={(x,y)R2: x2 + y2 2x, y 0} i este parcurs n sens direct.


6. Fie suprafaa ( ) : r = u cos v i + u sin v j + v k , u [0, a], v [0,2] .

S se determine aria i s se calculeze integrala I = x 2 + y 2 d .

7. S se calculeze: a)

( xy + yz + zx)d ,

unde

este poriunea din suprafaa de ecuaie

z = x 2 + y 2 , decupat de suprafaa de ecuaie x 2 + y 2 = 2ax, a > 0 ;

b) c)

( x

+ y 2 )d, ( ) : x 2 + y 2 + z 2 = a2 ;

( x + y + z)d ,

fiind suprafaa cubului definit de

0 x 1, 0 y 1, 0 z 1.
8. S se calculeze urmtoarele integrale de suprafa de spea a doua: a)

xdydz + ydzdx + zdxdy ,

unde

este faa exterioar a tetraedrului

limitat de x = 0, y = 0, z = 0, x + y + z = 1;

- 277 -

b)

( y z)dydz + (z x )dxdz + ( x y )dxdy , unde

este faa interioar a

conului de ecuaie z = x 2 + y 2 , 0 z h ;
c)

zdxdy ,

fiind

faa

exterioar

elipsoidului

de

ecuaie

x 2 y 2 z2 + + 1= 0 . a2 b2 c 2
9. Folosind formula lui Stokes s se calculeze

x 2 + y 2 + z 2 = a 2 ( y + z)dx + (z + x )dy + ( x + y )dz , unde ( ) : x + y + z = 0 parcurs fiind astfel nct domeniul interior s fie lsat la stnga).
10. S se calculeze: a) b) c)

(sensul de

( x
V V

+ y 2 )dxdydz, V : x 2 + y 2 2z, z 2 ; + y 2 + z 2 )dxdydz, V : x 2 + y 2 z 2 , x 2 + y 2 + z 2 a2 , z 0 ;

( x

x 2 + y 2 dx dy dz, V : x 2 + y 2 a 2 , x + y + z 2a, z 0 ;

11. Folosind schimbrile de variabile s se calculeze: a) b) c) d)

x 2 + y 2 + z 2 dx dy dz, V : x 2 + y 2 + z 2 2z ;

x 2 + y 2 dx dy dz, V : x 2 + y 2 2z, x 2 + y 2 + z 2 8 ;
2

( x
V

+ y 2 )dx dy dz, V ::x 2 + y 2 2x 0, y 0, 0 z 1;

x2 y2 z2 x2 y2 z2 1 2 2 2 dx dydz, V : 2 + 2 + 2 1. a b c a b c

12. Direct i cu formula lui Gauss-Ostrogradski s se calculeze

dy dz +x 2 ydz dx + x 2 zdx dy , unde este faa exterioar a suprafeei nchise

a cilindrului x2 + y2 = a2, 0 z h.
13. S se calculeze volumul i coordonatele centrului de greutate ale

corpului omogen mrginit de suprafeele de ecuaii y2 + z2 = 4ax, x2 + y2 = 2ax, unde a > 0.

- 278 -

BIBLIOGRAFIE
1. L.Aram, T.Morozan, Culegere de probleme de calcul diferenial, Ed. Tehnic, Bucureti, 1978. 2. I.Colojoar, Analiz matematic, E.D.P. Bucureti, 1983. 3.J.Cringanu, Analiza matematica, Editura Fundatiei Universitare Dunarea de Jos , Galati, 2006. 4. B.P.Demidovici, Culegere de probleme i exerciii de analiz matematic, Ed. Tehnic, Bucureti, 1956. 5. N.Donciu, D.Flodor, Algebr i analiz matematic (Culegere de probleme), E.D.P. Bucureti, 1979. 6. B.Gelbaum, J.Olmstead, Counterexamples n Analysis, San Francisco, London, Amsterdam, 1964. 7. D.I.Ion, C.Ni, Elemente de aritmetic cu aplicaii n tehnici de calcul, Ed. Tehnic, Bucureti, 1978. 8. M.Niculescu, Analiz matematic, vol. I, II, Ed. Tehnic, Bucureti, 1964. 9. A.Precupanu, Funcii reale i teoria msurii, Univ. Al. I. Cuza, Iai, 1972. 10. A.Precupanu, Analiz matematic, vol. I, II, Univ. Al. I. Cuza, Iai, 1987. 11. A.Precupanu, Bazele analizei matematice, POLIROM, 1998. 12. S.Rdulescu, M.Rdulescu, Teoreme i probleme de Analiz matematic, E.D.P. Bucureti, 1982. 13. R.B.Reisel, Elementary theory of Metric Spaces, Springer Verlag, New York, Heidelberg, Berlin, 1982. 14. M.Rocule, Analiz matematic, vol. I, II, E.D.P. Bucureti, 1979. 15. V.Rudin, Osnovi matematiceskovo analiza, Moscova, 1966. 16. W.Rudin, Real and Complex Analysis, New York, McGraw-Hill, Inc., 1966. 17. L.Schwartz, Cours dAnalyse, vol. I, II, Herman, Paris, 1967. 18. Gh.Sirechi, Calcul diferenial i integral, vol. I, II, Ed. tiinific i Enciclopedic, Bucureti, 1985. 19. O.Stnil, Analiz matematic, E.D.P. Bucureti, 1981.

You might also like